You are on page 1of 592

..

. .
53(075.3)
22.372
19

. .
19 . / . . . .:
: $
, 2009. 592 .: .
ISBN 978$5$488$02030$6 ( )
ISBN 978$5$94666$505$6 ( )
,
, ,
, . $
( 7$ ),
,
.
, ,
.
, $
.
, , ,
( ).
.

53(075.3)
22.372

ISBN 978$5$488$02030$6 ( )
ISBN 978$5$94666$505$6 ( )
. ., 2009
. , 2009

. . . . . . . . . . . . . . . . . . . . . . . . . . . . . . . . . . . 11
1. . . . .. 12
1.1. . . . . . . . . . . . . . . . . . . . . . . .. 12
1.2. -
. . . . . . . . . . . . . . . . . . . . . . . . . . . . . . . . .. 15
1.3. . . . . . . . . . . . . . . . . . . . . . . . .. 17
1.3.1.
. 1.3.2.
: v(t) x(t).
1.4. . . . . . . . . . . . . . . . . . . . . . . . . . . . . . . . . . .. 20
1.4.1. . 1.4.2. -
. 1.4.3. (-
) . 1.4.4. , -
.
1.5. (
) . . . . . . . . . . . . . . . . . . . . . . . . . . . . . . . . . .. 26
1.5.1. . 1.5.2. -
. 1.5.3.
. 1.5.4.
( ).
1.6. .. 32
1.6.1. . 1.6.2. -
. 1.6.3. -
.
. . . . . . . . . . . . . . . .. 40
2.
. . . . . . . . . . . . . . . . . . . . . . . . . . . . . . . . . .. 43
2.1. . . . . . . . . .. 43
2.1.1. ( ). 2.1.2. -
. 2.1.3.
.
2.2. . . . . . . . . . . . . . . . . . . . . . . . . . . . .. 46
2.2.1. , ,
. 2.2.2. (-
). 2.2.3.
. 2.2.4. .
2.2.5. -

3
. 2.2.6. -
. 2.2.7. , -
. 2.2.8. .
2.3. . . . . . . . . . . . . . . . . . . . . . . . . . . . . .. 61
2.3.1. . 2.3.2. -
. 2.3.3. . 2.3.4.
. 2.3.5. -
. 2.3.6.
. 2.3.7. .
2.4. . . . . . . . . . . . . . . . . . . . .. 71
2.4.1. . 2.4.2.
.
2.4.3. . 2.4.4. .
2.4.5. ? 2.4.6.
.
2.5. . .. 86
2.5.1. -
( ). 2.5.2. -
, -
. 2.5.3. .
2.5.4. .
2.5.5.
. 2.5.6. .
2.6. . . . . . . . . . . .. 95
2.6.1. . 2.6.2. -
. 2.6.3. -
. 2.6.4. . 2.6.5. -
. 2.6.6. -
.
. . . . . . . . . . . . . . . .. 104
3. : . . . .. 107
3.1. . . . . . . . . . . . . . . . . . . . . . . . . . . . . . . . .. 108
3.1.1. . 3.1.2. -
. 3.1.3. . 3.1.4. -
.
3.2. . . . . . . . . . . . . . . . . . . . . . .. 114
3.2.1.
. 3.2.2.
. 3.2.3. .
3.2.4.
? 3.2.5. .
3.3. ,
. . . . . . . . . . . . . . . . . . . . . . . . . . . . . . . . . . .. 123
3.3.1. ,
. 3.3.2. , -
. 3.3.3.
.

4
3.4. . . . . . . . .. 129
3.4.1. , -
. 3.4.2. -
. 3.4.3. -
. 3.4.4.
.
3.5. . . . . . . . . . . . . . . . . . . . . . . . . . .. 137
3.5.1. . 3.5.2. . 3.5.3. . 3.5.4.
. 3.5.5. .
. . . . . . . . . . . . . . . .. 148
4. :
. . . . . . . . . . . . . . . . . . . .. 152
4.1. . . . . . . . . . . . . . . . . . . . . .. 152
4.1.1. .
4.1.2. . 4.1.3. -
() . 4.1.4.
.
4.2. . . . . . . . . . . . . . .. 162
4.2.1. .
4.2.2. . 4.2.3. -
. 4.2.4. .
4.3. . . . . . . . . . . . . . . . . . . .. 172
4.3.1. . 4.3.2. -

. 4.3.3.
.
4.4. . . . . . . .. 177
4.4.1.
. 4.4.2. -
. 4.4.3. .
4.5.
. . . . . . . . . . . . . . . . . . . . . . . . . . . . .. 182
4.6. . . . .. 188
4.6.1. , -
. 4.6.2.
. 4.6.3. .
4.6.4. .
4.7. . . . . . . . . . . . . . . . . . . .. 196
4.7.1. . 4.7.2. -
. 4.7.3. -
() .
4.7.4. . 4.7.5. -
.
4.7.6. , -
?.
. . . . . . . . . . . . . . . .. 206

5
5. . . . . . . . . . . . . . . . . . . . . .. 210
5.1. ? . . . . . . . . . . . . . . . . . . . . . . . . . . . .. 210
5.1.1. -
. 5.1.2.
. 5.1.3.
. 5.1.4.
.
5.2. . . . . . . . . . . . . . . . . . . . . . . . . . . . . . . . .. 214
5.2.1.
. 5.2.2. . 5.2.3. -
.
5.2.4.
. 5.2.5. -
. 5.2.6.
. 5.2.7. .
5.3. . . . . . .. 224
5.3.1. -
. 5.3.2.
? 5.3.3. -
.
5.4. . . . . . . . . . . . . . . . . . . . . . . . . .. 229
5.4.1. .
5.4.2. .
5.4.3. . 5.4.4. -
-
. 5.4.5. , -
, . 5.4.6.
. 5.4.7. -
.
5.4.8. ,
.
5.5. . . . . . . . . . . . . . . . . . . . . . . . .. 244
5.5.1. . 5.5.2.
. 5.5.3. -
, . 5.5.4. -
-
. 5.5.5. -
. 5.5.6.
.
5.6. -
. . . . . . . . . . . . . . . . .. 253
5.6.1. . 5.6.2. -
. 5.6.3. ,
. 5.6.4. -
.
5.6.5.
. 5.6.6. -

6
-
.
5.7. -
? . . . . . . . . . . . . . . . . . . . . . . . . . . . . . . . . . . .. 269
5.7.1. ( )
. 5.7.2. -
. 5.7.3. -
. 5.7.4. . 5.7.5.
.
. . . . . . . . . . . . . . . .. 279
6. . . . . . . . . . . . . . . . . . . . . . . . . . . . .. 283
6.1. . . . . . . . . . . . . . . . . . . . . . . . . . . . . .. 283
6.1.1. ( -
). 6.1.2. (-
). 6.1.3.
( -
).
6.2. . . . . . . . . . . . . . . . . .. 287
6.2.1. , -
. 6.2.2. .
6.2.3. . 6.2.4.
. 6.2.5.
. 6.2.6. -
.
6.3. -
. . . . . . . . . . . . . . . . . . . . . . . . . . . . . . . . . . . . . .. 302
6.3.1. . 6.3.2.
. 6.3.3.
. 6.3.4.
.
6.4. . . . . . . . . . .. 309
6.4.1. -
. 6.4.2. .
6.4.3. . 6.4.4.
. 6.4.5. .
6.4.6.
, .
6.5. . . . . . . . . . . . . . . . . . . . . . . .. 328
6.5.1. -
. 6.5.2.
. 6.5.3. -
.
6.6. ? . . . . . . . . . . . . . . . . . . . . . . . . . . .. 336
6.6.1. 6.6.2. -
( ). 6.6.3. .
6.6.4.
.
. . . . . . . . . . . . . . . .. 344

7
7.
() . . . . . . . . . . . . . . . . . . . . . .. 349
7.1. . . . . . . . . . . . . . . . .. 349
7.1.1.
. 7.1.2.
. 7.1.3. . 7.1.4. -
. 7.1.5. . 7.1.6. -
.
7.2. . . . . . . . . . . . .. 373
7.2.1. . 7.2.2. -
. 7.2.3. . 7.2.4. -
. 7.2.5. .
7.3. . . . . . . . . . . . . . . . .. 386
7.3.1. .
7.3.2. . 7.3.3.
. 7.3.4. -
. 7.3.5. .
7.4. . . . . . . . . . . . . . . . . . . .. 402
7.4.1. .
7.4.2. . .
7.4.3. . 7.4.4.
. 7.4.5. . 7.4.6.
.
7.5. . . . . . . . . . . . . . . . . . . . .. 413
7.5.1. ( ). 7.5.2.
. . 7.5.3.
.
7.5.4. . 7.5.5.
.
. . . . . . . . . . . . . . . .. 423
8. . . . . . . . . . . . . .. 427
8.1. . . . . . . . . . . . . . . . . . . . . . . . . . . . . .. 427
8.1.1. . 8.1.2. ()
. 8.1.3. . 8.1.4. -
. 8.1.5. -
.
8.2. . . . . . . . . . . . .. 434
8.2.1. . 8.2.2. -
-
. 8.2.3. , ,
. 8.2.4. -
.
8.3. . . . . . . . . . . . . .. 440
8.3.1. -
. 8.3.2. .
8.3.3. -

8
. 8.3.4.
. 8.3.5.
.
8.4. . . . . .. 452
8.4.1. -
. 8.4.2. -
. 8.4.3.
.
8.5. -
. . . . . . . . . . . . . . . . . . . . . . . . . . . .. 457
8.5.1. -
. 8.5.2.
. 8.5.3. -
,
. 8.5.4. . 8.5.5. .
8.5.6. .
8.6. . . . . . . . . . . . . . .. 467
8.6.1. . 8.6.2. -
(). 8.6.3.
. 8.6.4.
.
. . . . . . . . . . . . . . . .. 474
9. . . . . . . . . . . . . . . . . .. 477
9.1. . . . . . . . . . . . . . . . . . . . . . . . . . . .. 477
9.1.1. . 9.1.2.
. 9.1.3. -
: . 9.1.4. -
. 9.1.5. -
; . 9.1.6.
.
9.2. . . . . . . . . . . . . . . . . . . . . . . .. 491
9.2.1. . 9.2.2. -
-
? 9.2.3.
. 9.2.4. , -
( ).
9.3. . . . . . . . . . . . . . . . . . . . . . . . . . . . . . . .. 500
9.3.1. . 9.3.2. -
. 9.3.3. -
-
. 9.3.4.
. 9.3.5.
.
9.4. . . . . . . . . . . . . . . . . . . . . . . . . . .. 513
9.4.1.
. 9.4.2. :
.

9
9.4.3. : -

. 9.4.4. .
9.5. . . . . . . . . . . . . . . . . . . . . . . . . . . . . . . . . . . . . . . .. 523
9.5.1. .
9.5.2. . 9.5.3. -
. 9.5.4. .
9.5.5. . 9.5.6. -
.
9.6. . . . . . . . . . . . . . . . . . . . .. 534
9.6.1. . 9.6.2. , -
. 9.6.3. , .
9.6.4. ,
. 9.6.5. -
, .
9.7. . .. 542
9.7.1. . 9.7.2. .
9.7.3. . 9.7.4.
-
.
9.8. . . . . . . . . . . . . . . . . . . . . . . . . . .. 548
9.8.1. . 9.8.2. -
. 9.8.3.
, . 9.8.4.
: . 9.8.5.
,
. 9.8.6. -
.
. . . . . . . . . . . . . . . .. 561
. . . . . . . . . . . . . . . . . . . . . . . . . . . . . . . . . . . .. 565
. . . . . . . . . . . . . . . . . . . . . .. 565
.1. . .2. -
.
. . . . . . . . . . . . . . . . . . . .. 573
.1. . .2. , , -
. .3. -
. .4. .
. . . . . . . . .. 583

10

, ,
.
, , -
,
.
,
, , ,
, ,
,
,
.
.
1. . 2.
. 3. : . 4.
: . 5. . 6. .
7. (). 8.
. 9. . -
, -
, ,
- , -
. -
100 .
.
.
,
, , (
), , -
.

* * *
,
, -
: ,
, ,
, ,
, . -
-
.

11
1


1.1.

-
. -
, -
-
, -
. , -

. -
.
() . -
, -
.
, ,
; , , -
. :
, ,
.
,
,
, -
. -
,
, ,
.
.
,
, .

? : ,
, -

12
. ,
-
,
12 .
. -
,

. -
,
; -
, ,
.

( ). -
, -
. -
.
-
. ,


.
, ,
,
.

, ,
,
. :
.
. -
,
.
,
.
, . -
-
, .
.
,

. , , -
. -
, .
, -
.

13

.
, -
, -
. , , .
, ,
, .
, -
, . . , ,
. -
.
(, -
), , -
.
,
. ,
, .
(, , , -
) .
, -
, . ( kinema )
, -
. -
. -
( dynamics ). -
( statos ),
, , .

. -
,
.
,
,
.
.
, -
.
. ,
, -
. , -
, .
, ( ,
), . -
,
. -
,

14
, . -
, ,
-
. - . 1.1
, -
(. 1.1).
,
. -
( , ).

1.2.

.
, -
, ( -
) . ,
.
, , , ,
? ,
,
(-
speed) (--
velocity).
-
x, y, z (. 1.2, ). A
t x(t), y(t), z(t),
- r (t). ,

- r (t) ,

)
)
. 1.2

15
. , -
t t + t, -
1.2, . B t +
+ t ( - r (t + t)). s
, . . , -
t t + t. -
-

s
v = ------
-. (1.1)
t
, v ;
.
1.2,

r = r (t + t) r (t) (1.2)
t t + t.
-



v = r
------ . (1.3)
t
,
v . v

r .
, -

.
-
, (1.1) -
v t. , , t 2 ,
s 2 .

.
,
? t
v
?

( , ,
).
, -
t ,
.

16
t -

r ( -

, v ).
, -
-
-

| r | . 1.3

( , | v |). -

r ( ,
v )
t. 1.3,

AB, BC, CD. -

r 1, r 2, r 3 , -
,
.


, 
. ,
.

1.3.

1.3.1.

- , -
() t.
, -
.
, , -
, .
, -
, ,
, ,
.
t
( v (t)), -

17
ti = ti t
(i = 1, 2, 3, ):

r 1 r 2 r 3 r i
--------- , --------- , --------- , , -------- , , (1.4)
t 1 t 2 t 3 t i


r i = r (t + ti) r (t). (1.5)
ti -
, i
, . -
, , t + ti t,

r
--------1- -
t 1

v (t). , t -

ti. -
, (1.4) ti 0.

r
: lim --------i . -
t i 0 t i

, t:

r
v (t) = lim --------i . (1.6)
t i 0 t i

1.4 -
. t A,
t + ti (i = 1, 2, 3, 4) A1, A2, A3, A4,
. , ti -

. , -
ti 0 v (t)
A. ,

-
-
-
, .
(1.5) (1.6)
:


r ( t + t ) r ( t )
i
v (t) = lim ----------------------------------------
-.
t i 0 t i
. 1.4

18

:

r ( t + t ) r ( t )
v(t) = lim --------------------------------------
-. (1.7)
t 0 t

1.3.2. : v(t) x(t)

, -
. -
Ox. t x(t), -
t + t x(t + t). (1.7),
t
t:
x ( t + t ) x ( t )
v(t) = lim ---------------------------------------- . (1.8)
t 0 t
(-
, , ) -
, -
.
-
x(t), , -
(1.8), -
v(t). , x(t)
, 1.5.
-
? t0?
, -
-
t0 t0 + t . 1.5

x ( t 0 + t ) x ( t0 ) BC
---------------------------------------------- = --------- = tg . (1.9)
t AC
t. B x(t)
, A.
, t 0 (1.9) -

v(t0) = tg0, (1.10)
0 x(t),
A, t0. ,
v(t0) -
x(t) t0.

19
1.5 x(t),

. , .
t = 0.
t0 , ,
-
, tg 0.
t = t1 ( D -
x(t) ).
, 1.5
. -
, x1.
x(t)
. x0
( t = 0), x(t)
t. t x(t) x0,

x ( t ) x0
----------------------- . ,
t
t ( t, , 2 , 2
x(t) x0). ,
x ( t ) x0
----------------------- , t, -
t
v :
x(t) x
v = ----------------------0- . (1.11)
t
,
x(t) = x0 + vt. (1.12)
, x(t) (-
) -
.

1.4.

1.4.1.

, -
. -
, .
, -

20
( , ).
, .

-

. a t t + t -


v ( t + t ) v (t)
a = ----------------------------------------
-. (1.13)
t

a (t) -
t 0:

v ( t + t ) v (t)
a (t) = lim ----------------------------------------- . (1.14)
t 0 t


,
. 
. 
, 
. , 
, , , .

(1.14)
:
v ( t + t ) v ( t )
a(t) = lim ----------------------------------------
-. (1.15)
t 0 t
v(t) ( v(t + t) > v(t)),
: a(t) > 0 ( ). -
v(t) ( v(t + t) < v(t)), -
: a(t) < 0 ( ).
v(t + t) = v(t), -
, . . .
, -
. -
- ,
.

, .
.

1.4.2.
-
R. v. -

a . -

21
) )
. 1.6

1.6, . A -
t, B
t + t:
v 1 v 2 -
. -
OA OB. t -
, AB -
, . . vt. 1.6,

v 1, v 2, v = v 2 v 1. -
t
v
v 1 v
a 2 - = ------- .
= ----------------- (1.13, )
t t
, , ,

AB v
--------- = -------
OA v
AB vt
, , v = v --------- = v ---------- . ,
OA R
2
v v
------- = ------ . (1*)
t R
(1.13, ) (1*)
2
a = lim v v
------- = ------ .
t 0 t R
, -
R v -
:
2
a = v
------ . (1.16)
R

?
?
1.6, ,
.

22

v 1 v ( v 2 v ), -

+ 2 = .
,

= --- ------- . (2*)
2 2

,
a t

, v. , a
, (2*), ,

/2. t 0 a -
, . ,
-
( -
).
.

1.4.3. ()

, -
: ,
. a (t)
t -
:

a (t) = a (t) + a (t). (1.17)

a (t) ; -
, -
, (1.16):
2
(t)
a(t) = v
------------- (1.18)
R
( (1.16) ).
a (t) , -
( )
( ). -
, .
-
, -
. () -

23
, -
. :

v ( t + t ) v (t)
a(t) = lim ------------------------------------------- ; (1.19, )
t 0 t
v ( t + t ) v ( t )
a(t) = lim ----------------------------------------
-. (1.19, )
t 0 t
, . . -
. R -
. , (1.18),

(1.19) :
a (t) = a (t). , -
, .

1.4.4. ,

v
R. t t + t
MN
A B (. 1.7).
=
------- (1.20)
t
, .
(1.20) R; R = R
------- . -
t
R AB; vt. ,
v = R. (1.21)
,
v(t) = R(t), (1.21, )
(t) .
(1.21, ) (1.18) :
a(t) = 2(t)R. (1.22)
,
. -
(1.21) ( (1.21, ), -
). -

MN (. . -
) ,
:
. 1.7 ,

24

,
(. 1.8).

. 1.8 . 1.9

, (t) v (t) .
-

r (t) (. 1.9). , -

1) (t) r (t) v (t)
, (t)r(t) sin. ,

v (t) = (t) r (t). (1.23)
() -
(: ),
(: ). -
(1.1), :

= .

,
(: /).
(1.13) :

= .

,
(: /2).
(1.22) :

= .

(, )
(: ); 1 = 180/ 571445.
(-
: /).

1) .

25
1.5.
( )

1.5.1.


, -
. -
, .

v 0

( t = 0), v (t)
t.
0 t , (1.13),

v (t) v
a = ----------------------0- . (1.13, )
t

a (1.13, ) -
t. a -

v(t) v
----------------------0- t. -
t
t
t 0,
a :

v (t) v
a = ----------------------0- .
t
,

v (t) = v 0 + a t. (1.24)
-

. v 0
, -

, t = 0. v 0 -
, -

. , v 0
.
-
, . -

, v 0 a .
-
, .

26
1.5.2.
(1.24)
:
v(t) = v0 + at. (1.25)
v(t), v0, a -
x-,

v 0. v(t) 1.10 :

a > 0 ( , v 0 a -

), a < 0 ( , v 0 a ).

. 1.10

1. -
a = 5 /2. t1 = 3 -
aR = 3 /2.

.
(1.25) v(t1) = at1. -
T at0 + a(T t1) = 0. ,
a
T = t1(1 ----
- ) = 8 c.
a
v(t) 1.11.
v(t),
, . . x-.
, t = 0
:
x(0) = 0. -
,
.

, (1.12), x(t) = v0t. -
, -
-
. 1.11 v(t)

27
( 1.12, ). -
, -

v(t) 0 .

. 1.12

, -
. O N -
T
t = ---- t1, t2, t3, , tN
N
(, tN = T). -
t ,
;
T
, , , i- , xi = v(ti) ---- .
N
0 t , xi (i = 1, 2, 3, ...
N
, N). , x i
i=1
v(t) 0 (. . 1.12, ). N,
x(T), ,
v(t). N = 6.

28

, v(t) = v0 + at (. .
1.12, ). x(T) -
, . -
,
x(t) = v0t + 1--- at2. (1.26)
2
t = 0 x0,
(1.26)

x(t) = x0 + v0t + 1--- at2. (1.26, a)


2
(1.25)(1.26, )
, -
.
(1.25), -
()
(1.26) ( (1.26, )). x(t) ( -
). a > 0, , -
a < 0, .
t, (1.25) (1.26) -
:
2
v(t) = v 0 + 2ax ( t ) . (1.27)
, -
.
2.
a = 4 /2. x = 8 , -
.
T = 5 ?
, t = 0. -
t0 , -
. (1.27), v(t0) = 2ax . (1.25),
v(t ) 0 2x
- =
t0 = ------------ ------ .
a a

t0 v(t0)(T t0). -
,
2x
x + v(t0) (T t0) = x + 2ax T ------ =
a

= T 2ax x = 32 .

29
1.5.3.

1.131.18. -
v(t) x(t)
( , x0 = 0). x(t) 1.17
(. . 1.15). 1.3 , -
t -
x(t) .
,
. ,
x(t) , -
. v -
x(t) , v
. , ,
x(t) ( x(t)
).

. 1.13 . 1.14 . 1.15

. 1.16 . 1.17 . 1.18

30
1.19 -
. t1 -
v(t) : -
v1
.
x(t) t1 .
, -

, , x(t)
.
,
-
. - . 1.19
-
1.151.18.

1.5.4.
( )

,

, , v 0 a
. -

, ; v (t)
. -
xy-.
(1.24)

x- y- v (t), v 0,
a:

vx(t) = v0x + axt;


vy(t) = v0y + ayt. (1.28)

-
, (1.26):

1
x(t) = v0xt + --- axt2;
2 (1.29)
1
y(t) = v0yt + --- ayt2.
2

xy--
,
( )
a .

31
y-. ax = 0 (1.28) (1.29)
:
vx(t) = v0x, (1.30, )
vy(t) = v0y + ayt; (1.30, )

x(t) = v0xt, (1.31, )


2
ay t
y(t) = v0yt + ----------
-. (1.31, )
2


-
x- v0x -
y- ay v0y.
, -
, . , -
, -
-
,
. , -
,

, .

1.6.

1.6.1.

, -
, . -

; -
, . -
,

( ). -
9,8 /2 -
. -
g ( gravitas, -

); g.

32

:
) ;
) ;
) ;
) .
) ) -
( ), ) )
,

v 0 g ( xy-).
-
(1.30) (1.31), , y- -
( , ay = g), x-

v 0 . -
y0, (1.31, )
y0.
(1.30) (1.31) :
vx(t) = v0x, (1.32, )
vy(t) = v0y gt; (1.32, )

x(t) = v0xt, (1.33, )


2
y(t) = y0 + v0yt gt
-------- .
2 (1.33, )

,
.

1.6.2.

). . -
t = 0 , H, , -
. T
(1.33, ), t = T, y(T) = 0, y0 = H, v0y = 0:
2H
T = -------- . (1.34)
g
(1.32, ), t = T, v0y = 0, vy(T) = v1 (1.34), -

v1 = gT = 2gH . (1.35)

33
).
. t = 0 -

v 0, . -
:
T1 H -
T2. T1 , , -
(. . t = T1)
. (1.32, ) t = T1, v0y = 0, vy(T1) = 0,

v
T1 = -----0- . (1.36)
g

, (1.33, ), t = T1, y0 = 0,
v0y = v0, y(T1) = H; H = v0T1 1--- gT12. (1.36)
2
,
2
v
0
H = -------- . (1.37)
2g
(1.37) (1.34) (1.35),
2H v
T2 = -------- = -----0- (1.38)
g g

v1 = 2gH = v0. (1.39)
, T1 = T2, . . ,
, , -
, v1 = v0, . . -
. T -

2v
T = T1 + T2 = ---------0 . (1.40)
g

, t = 0 h. -
v 0.
T1, H, T2, v1, T? , T1 -
v
, - -----0 . -
g
2
v0
h (1.37), . . H = h + ------
-.
2g

34
T2 v1
(1.38) (1.39), ,
H:

2H 1
---------- = --- 2gh + v 0 ;
2
T2 = (1*)
g g
2
v1 = 2gH = 2gh + v 0 . (2*)

, T2 > T1 v1 > v0. T -



1
(
T = T1 + T2 = --- v0 +
g
2gh + v 0
2
). (3*)

, v0 = 0 h = H (2*)
(1.35), (3*) (1.34).

, -


v 0.
, v0, -
g, H. -
, (1.25), v1 = v0 + gT, v1 -
, , (1.27), -
2
v1 = v 0 + 2gH . ,

1
(v1 v0) = --1- ( 2gH + v 0 v0).
2
T = ---
g g

v0 = 0 (1.34).

1.6.3.

).
. H

t = 0 c v 0, -
x-. -


x- v0 -
y- -
. ,

35
T y- v1y; (1.34)
(1.35),
2H
T = -------- v1y = 2gH .
g
,
:
2H
L = v0T = v0 -------- . (1.41)
g
x- , x-
: v1x = v0. ,

2 2 2
v1 = v 1x + v 1y = v 0 + 2gH . (1.42)

v 1
x-.

1y v 2gH
tg = -------- = ----------------- . (1.43)
v 1x v0

-
xy-
y-
x-. , (1.33),
y0 = H, v0y = 0, v0x = v0:
x = v0t;
1 (1.44)
y = H --- gt2.
2

t , -
:
g
y = H ------------ x2. (1.45)
2
2v 0

(1.45) -
; A -
xA = 0 yA = H (. 1.20).
,

v0. ,
H
2H
: T = -------- . v0 ,
. 1.20 g

36

.
). . -
t = 0

v 0, . -
,
. -
, T -
. -
(1.37) (1.40), v0
v0y = v0 sin. :
2 2
v 0 sin
H = ------------------------ ; (1.46)
2g
2v 0 sin
T = ----------------------
-. (1.47)
g
-
v0x = v0 cos. (1.47)
:
2
v 0 sin 2
L = v0xT = -------------------------
-. (1.48)
g

2
0 v
, 45. L = -------- .
g
, (
)
, . -

v 1
(1.42) (1.43), v0 cos v0 , -
2 2
0 v sin
(1.46), H ------------------------ .
2g
(1.42) (1.43) :
2
2 2 v0 2 2 2
v1 = v 0 cos + 2g -------- sin = v 0 cos + sin = v0;
2g
2
v0 2
v 1y 2g -------- sin
2g sin
tg = -------- = ------------------------------------- = ------------- = tg .
v 1x v 0 cos cos
, -
. , -

37
x- . -
,

| v 1 v 0 | = 2v0sin. (1.49)

(1.49) (1.24)
(1.47). (1.24) :

v 1 = v 0 + g T. , ( v 1 v 0) = gT. (1.47), 
(1.49).

xy--
. , (1.33), y0 = 0, v0x =
= v0 cos, v0y = v0 sin:
x = v0t cos;
1 (1.50)
y = v0t sin --- gt2.
2
t, y(x):
2
gx
y = x tg ---------------------------- . (1.51)
2 2
2v 0 cos
(-
) -
(. 1.21). -
:
2 2
v0 v0
xA = -------- sin2; yA = -------- sin2 .
2g 2g
(1.52)
(1.52) (1.46) (1.48),
. 1.21 , xA = L
---- , yA = H.
2

, (1.52). (1.51) -

2
g
y = --------------------------- 2 v 0 xsin2).
2 2
(x ------- (1*)
2v 0 cos g
:
2 2 4
v0 v0 v0
x2 ------- xsin2 = (x ------- sin2)2 ---------
- sin2 2,
g 2g 2
4g
(1*) :
2 2
g v0 v0
y = ---------------------------
2 2
(x ------- sin2)2 + ------- sin2 .
2v 0 cos 2g 2g

38
, (1.51) y =
g
= ---------------------------
2 2
x2, ,
2v 0 cos
2 2 2
v 0 sin 2 v 0 sin
x- xA = ------------------------
- y- yA = ----------------------
- .
2g 2g

1. h -

v 0 .
.
T1 , T2 ,

0 v sin
. (1.47), , T1 = ------------------- ,
g
(1.34) (1.46), ,
2
v0 2
2 h + -------- sin
2g
T2 = -------------------------------------------- .
g

,
2
v0 2
2 h + -------- sin
v 0 sin 2g
T = T1 + T2 = ------------------- + -------------------------------------------- . (2*)
g g

v
(
L = Tv0 cos = -----0- cos v0 sin +
g
2gh + v 0 sin .
2 2
) (3*)

, h = 0 (2*)
(1.47), (3*) (1.48).

2. -
O
(. 1.22).

v 0
-
. -
-
-
. . 1.22

39
B (1.51)
= x tg (. ). xB -
,
g
x tg ---------------------------- x2 = x tg .
2 2
2v 0 cos
:
2 2
2v 0 cos
xB = ---------------------------- (tg tg ). (4*)
g
, -

2 2
xB 2v 0 cos
- = ---------------------------- (tg tg ).
l = ------------ (5*)
cos g cos
T , xB.

v0cos -
B x
, xB. , T = -------------------
- . (4*),
v 0 cos

2v 0 cos
T = ----------------------- (tg tg ). (6*)
g
, = 0 (5*)
(1.48), (6*) (1.47).

1.1.
- ?
. - 1.4.
- -
v0.
- -
?
1.2. , ,
5/6 -
. , -
1.3. ?
- 1.5. -
- , -

40
1.12. -
. -
1.6. - x = 16
n = 3 .
R , - , -
- , -
. v0 = 6 /?
R/2 - 1.13. -
, -
v0. - t = 0 v0.
, -
, ,
? t = t1
1.7. 1.17 v = 5 /, , -
t1 = 3 . , t = 0?
x1. 1.14.
1.8. x1 , ,
, t = 0

1.17 1.18, , x = x0 v 0
v - x-.
, t1 x-
1,2 , . t = 2t1.
1.9. - 1.15. ,
- 1.14, , -
0,5 . -
v 0 -
-
2 /, x-.
1.16. ,

6 /2.

-
-
?
?
1.10.
1.17. -
-
-
.
v0.
, 4 ,
vy(t) y(t).
12 /, 1.18. -
12 /. ?
1.19. , -
? , -
1.11. - -
- v0.
. h .
- -
? .

41
1.20. t = 0 - .
H -
?
v 0.
? -
-
?
H
----- ? 1.24. -
2
1.21. - v 0 -
H. . -
-
, -
H? .
1.22. - 1.25. -
-
6 , -
v 0. ,
(
-
1,62 /2). , -


45.
,
?
1.23. - ( -

v 0 - ).

42
2


2.1.

2.1.1. ( )

,
. -
. , -
, -
( , )
,
. , -
, .
, (
). :


,
-
.

, , -
.
-
- ; ,
( in-
ertia, ).
.
, ,
- .
, , . .
, , -
.

43
1638 :
, -
, -
, , -
.
(1644 .): -
, , -
-
, -
. -
.

, 1687 .
-
-
( , ).

2.1.2.

( ) -
, -
:
,
, -

, -
. ,
: -,
, -, , -
. ,
:

, -
, -
, -
.

, , -
.
, -
, ,

, . ,
-
, .

44
, -
, , , -
. ,
, -
. , ,
, , , , -
.

, , 
(, ), 
.
.
,
, , , 
. 
.

.
, .

, -
,
. (, -
) 2.5.

2.1.3.

,
, -
,
, , -
. ,
, , , ,
. -
.
,

. ,
-

v 0 , -

v 0
. , -
( !), ,
( -

45
!), . -
-
, , .
-
. -

, -
. -
:


. , -

.

, , -
, , -
; -

.
1905
. -
,
, -
, , . .
-
:

2.2.

2.2.1. , ,

.
, , .
, . -
, -
. -
- : ,

46
, , . -
: , , -
, . ,
,

, .
.
:
(: ).
, 1 , 1

1 /2 (1 = 1 --------------
2
).

, , -
-
. :

. , , -
;
, .
, , -
,
. :

, , -
, ( -
) .

, .
( ), .
, , -
. , ,
.
, , -
, . ,
,
.
,
( !) ,
, ,
. :
;
; ,
, -
. , -
. ,
.

47
,
-
, (
). (-
, ) ,
. -
, , -
. ( ,
, ,
3).
,
( ), ,
.
, ,
, . -
2.3, , -
, . ,
, -
, -
.
.
. ,
, , ,
.

2.2.2.
( )

,
-
. , -
-

( ), . . , .
. -
:

-
,
, -
, .

,
. ,
.

48


F(t)
(t) = ----------- . (2.1)
m

a (t) t, m

, F (t) ,
t.

(2.1) , F (t) = 0, a (t) = 0

. , F = 0 -
t t + t. -
, . .
, -
( t). , -

. ,
. -
,
, -
.

2.2.3.

, -

. p (t)1) t
m -

v (t):

p (t) = m v (t). (2.2)
, t ,

a (t)

v ( t + t ) v (t )
a ( t ) ----------------------------------------- . (2.3)
t
,
(. 1.4, (1.14)), t 0.
(2.3), (2.1)

v ( t + t ) v (t) F(t)
----------------------------------------- ----------- . (2.4)
t m

1) , -

49
(2.2),

m v (t + t) m v (t) = p (t + t) p (t),
:

p (t + t) p (t) h F (t)t. (2.5)
:

-

, .



(2.3), , 

F (t).

, -

:

-

, .

-
, . , -
, -
( ).
-

.
,
- . (2.5)

p ( t + t ) p (t)
----------------------------------------- F ( t )
t
t 0.
:

p ( t + t ) p( t )
lim ----------------------------------------- = F ( t ) . (2.6)
t 0 t

v ( t + t ) v (t)
, lim ----------------------------------------- a (t),
t 0 t
. . .

50

p ( t + t ) p (t)
lim ----------------------------------------
- (2.6)
t 0 t
. (2.6)
:

( ) -

, .


, -

.
(2.6)
,
, .
m ,

p ( t + t ) p ( t ) = m lim v ( t + t ) v (t)
lim ----------------------------------------
- ----------------------------------------- = ma (t) .
t 0 t t 0 t

(2.6) m a (t) = F (t),
(2.1).

, F , , a ,
, (2.5)
:

p = F t. (2.7)

p = p (t + t) p (t) t t.

2.2.4.

, -
( , ,
, , -
-
). , ,
, , . -
,
. -

,
.
,
.

51
-
? ?
,
( -
) . -
:
, , -
.
.
?
?


( g),
, ,
. m
F -
g:
F
m = ------ .
g
-
, .

F
(2.1)
g = ------ .
m
, -
( aditivus, -
): .
, , m1 ()
m2, m -
: m = m1 + m2.

.
, .

m V,
, :
m
= ----- . (2.8)
V
m, V ,
, , -
. , ,

52
.
.
. -

. ,
.

2.2.5.

(,
).
,
(-
r (t))

( v (t)).
, t = 0 -
- ,

v 0. , , -

t = 0, F ,
.
, t = 0, -

F
a = ----
- , m . -
m
, ,
. , -
. :
,
, -

. t v (t) =

= a t, t

v (t) = v 0 + v (t) = v 0 + a t. (2.9)
( . (1.23).)
- , , ,
,
. , ; -
. -

v f F
( -
, ).
, ,
, ,

53
.
,
, , -
-
. (2.9), -


F
v (t) =
v 0 + ----- t. (2.10)
m
,
, -
.
. , -
, , . .
, .
-
, ;
, . -
.

. -
. .
. -
,
, . -
.
, ,
.
, ,
. , -
, -
,


.
- , -
, -
. , , -
, -
,
, . -
, :
-
.
, , -
, ,

54
. ,
. ,
,
.

2.2.6.



F 1, F 2, F 3, , Fn .
n

F 1 + F 2 + F 3 + + F n, : Fi .
i=1

a

n
1

= -----
m Fi , (2.11)
i=1

m .
:
n

ai ,

= (2.12)
i=1

F

i = -----i , F i .
m
, , -
, . -
.
.
. m -

H F
.
. -
.

: F

F . , -


F+F
= ---------------- .
m
-

F
a = ----
- (
m

55

), g (
).

a = a + g. x- , y- -
. x-
vx(t) = Ft
------ ,
m
y- -
vy(t) = gt. x- y-
2
1 Ft
x = --- --------- ; (1*)
2 m
1 2
y = H --- gt . (2*)
2
T (2*), y = 0:
2H
T = -------- .
g
L (1*), t = T:
2
1 Ft FH
L = --- --------- = --------- .
2 m mg

2.2.7. ,
. 2.2.1 , -
* , (, ,
, , , ).
, ,
, , -
.
, , .
, . -
,
1
( ) I = ------------- : I
R+r
1 .
,
. : , -
( -
), ,
( ), ,
, ( ,
. . ).
, , XIX -
. ,

56
, -
. -
.

2.2.8.

-


F (. 2.1). -
-
.
; -
l
:
F = kl. (2.13)
. 2.1
-
k .
, -
.
(2.13)
. , , -
, ,
. ,
,
. -
-
.
, 2.1.
,

F

a = -----------
- , m .
m

F , (. 2.2).

, F F ;
,

F = F .

. 2.2

57

! : F ,
,

. F

F , F -

, , , F

F , , ,
(2.13).

, F , -

. F
, -

F . , -
, -
l
(. 2.3). -

F , ,

F .

. 2.3

2.4 , -
. A
, -

. F

F . , F = F .

. 2.4


, ,
. , 

. F

58
. ? ,
, 
, , 
( , , ).

2.5 , -

F . -
. ,

F = F .

, -
.
.

,
- (. 2.6, ).

, , F + F + F = 0
( ,
, ). . 2.5
, , -
0 .
F, F,
F , -

F , F , F .
,
( , . . 2.6, ).
, -
.

) )

. 2.6

59
-
.

.
, -
. -
. ,
, ,
.
-
() -
( ), ( ),
( ). -
1
, , .
, -
, , .


= .
, -

( )
: --------------
2
.

(-

: H); 1 = 1 --------------
2
.

, 1
1 /2.

(2.2)

= .
, -
(: /).
(2.8)

= .

,
(: /3).
,

= ,

: -
(: H/); 1 / = 1 /2.

60
2.3.


, , :
, ;
;
.
. -
.

2.3.1.

, -
-
. ,
. ( -
6).
,
( ). -

F

F = mg , (2.14)

m , g 1);

g ; 9,8 /2.
, 1.5 -
.
.
-
? , -
?
,
. (
) A (. 2.7, ). -

: F T ( -

). , , , F + T = 0.

, F = T -
,

1) -

61
, A
( AK 2.7, ).

- .

; B
(. 2.7, ). -
, ,
-
) ) BM. ,
AK
. 2.7 BM ( C)
-
. . :
, -
.
, -
.

. , -
, , , -
.
. -

.
, ,

(, , -
). -
() .
( 3.)
, ,
.
, ,
, -
.

2.3.2.

2.5

, F -

F ( F + F = 0). -
. -
, ,

62
, -

F ; T

(. 2.8). , F + T = 0. T
.
? ,
?
T ?
, . .
-
. , . 2.8
, -
, .
-
.

, T ,
, -
, . ,

T , . -
. 2.3.1
.

T .
: , -
, , -
( ).
( . 2.4.)
, -
,

.
, , . 2.9

, : F

N . , F + N = 0.

.
, -
.
(
)

N ( -
),


F . . 2.9

63
:
,
-
.
: -
! , . . -
. -
: , , , , . ,
- (,
, -
), -
, -
. -
,
. , , -
,
, . -
, -
, .

2.3.3.

-
,
-

. 2.10 F ,
(. 2.10).

F (
, , -
2.4). , -
, ,

F , F -
:

F = F .
. -
,
.
,
, ,
, , -
. -
, -

64
, ,
.
.
, -
FR, -
.
, .
,
F = N, (2.15)
N ,
; ,
, -
, .
, -
, , ,
.
N ( N = F);
, , .
, -
F, . F = F0
. F j FR

a , , -
,
F F
a = ---------------- , (2.16)
m
m . , -
, -
. ;
, (2.15):
F = N. (2.17)
. -
R , ,
F.



(. 2.11). F < F
F
(
F = F), F l F
F
( F = N -
F ). . 2.11

65
, F > F
, , -

, .
F , -
. -
.
,
(,
0,5,
0,04).

F N . ,
, -
.

2.3.4.

?
?
.
, -
, .
, .
-

, -
, ; .
, :
-
-
. ,
, -
, .
( ),
-
. : ,
, ,

. -
, .
.
,
, , , -
, .

66
- , , -
.
, -
. ,
:
.
.
, -
: ,
. , , -
, .
,
. ,
-
. -
, -
. ( 2.4.)
, ,
. - (-
, ) , -
, -
.

.
, ,
.

;
. -

.
.

2.3.5.

, -
.
. -
. -
-
() , -
. ,
. -
.

67
,
(
).
-
, , , . -
-
, , ,
.
:
;
;
;
( -
).
, , , -
. , -
, , .
.
. (-
), (,
, ).
,
(), . -

----------
- . ( .

7.)

( ) .
, ()
, -
. -

, Fc ,

R v ,
, -
:

Fc = 6R v . (2.18)
.

, Fc -

v .
-
, -
, -

68

v2. Fc -

F = CSv2, (2.19)
, S ,
C , (-

, C = 0,2, C = 1,11,2). Fc -

v , , -
(2.19) :

Fc = CSv v . (2.20)

2.3.6.

(2.20) , ,
* ( ), ,
. , m -
S.
t , V = Svt. -
n ; n = /m, -
. t N :
Svt
N = nV = ------------------ . (2.21)
m

f , , (2.7), -

( mv )
t :
f = ------------------ . ,
t
( ,

) , , ( mv ) = 0 mv = mv
. ,

mv

f = --------- . (2.22)
t
( 2.4),
, -
. ,

t, Fc ,

( ): Fc =
f N. (2.21)
(2.22),

Fc = Sv
v. (1*)
(2.20) C = 1 . ,
(2.20) , -
, (1*)
v
v (2.20).

69
2.3.7.

1.6 , . . -
. -
, -
, () . -
- , ,
.

.
,
-
. -
(2.20), .
m -

m g F . , -

v , a ,
,

m a = m g CSv v . (2.23)
-

g;
ma = mg CSv2. (2.24)
mg ,
CSv2 v.
, ,
, , -

v (

). , ,
, a = 0 (2.24).
mg
v = ------------ . (2.25)
CS
(2.25), vy , -
R. C = 0,2,
S = R2, m = 4--- R3, , (2.25) :
3

4Rg 6, 7 Rg
v = ----------------- ----- = ------------------------ .
3 0, 2

, g = 9,8 /2, = 1000 /3, = 1,3 /3.


, /,

70
v = 50, 5 R /, R .
2 , v = 10 / . ,
2 10 /.
, , -
, .
(, , -
).
, H = 2
. , (1.35), -

v 1 = 2gH = 2 9, 8 1000 / d 200 /.
.
. 70 .
, (2.25) CS = 0,2 /. , -
60 /. -
, , , 500 .
, 7 -
500 , . , -
, 6 ,
60 /. 100 , . .
. ,
, 60 /, , , . -
, 56 /, -
.

2.4.

2.4.1.

-
. . 2.2.1, -
, (
A), , (-
B). . -
, B A, -
A B.
, A B (
), , -
B A (
). ,
, .

71
, A B
, ( B A A B). ,
.
-
2.3. ,
.
,
.
, .

( ).
-
. -
, .

. .
, .
, -
.

2.4.2.

, -
. ,
, -
.

.
?
?
, -
.
, -
, ,
. ,
. , ,
, ,
.
, -
:
, , -
-
.

72
:

-
,
-
.

( m1) F 1 -
( m2). -

, F 2,

F 1,
:

F 1 = F 2. (2.26)
,


m1 a 1 = m2 a 2. (2.27)

a 1,

a 2. (2.27) ,
a1 m2
------ = ------- . (2.28)
a2 m1
, -
.
.
m

F , -

g, 9,8 /2. F =

= m g ,
m. , -
m .
M = 6 1024 . (2.28) , a -
,
m, a = mg
--------- . ,
M
, m = 1 , a = 1,6 1024 /2. -
.
, a = 0.
m . :
-
!

73
2.4.3.


(:
reaction).
, -
-
.
1. (. 2.12)
m.

.


F = m g N , -
-
.
,


, N F -
, -


. 2.12
. F -
(-

F 1) (

F 2); ,
F1 = mg sin F2 = mg cos .

a -
. , ma = F1 , -
,
a = g sin . (2.29)
, , -
; ,
N = F2 = mg cos . (2.30)


N R. , N = N , ,
N = mg cos .
2. -
m1 m2, bc (. 2.13).

74
. 2.13

m1 -

F .
.

1 : F 1 = m1 g, -

N 1, F , T 1. 2

: F 2 = m2 g, N 2,

T 2. T 1 T 2.
. -
,

T 1 = T 1; T 2 = T 2. (1*)

, N 1 + F 1 = 0 N 2 + F 2 = 0
-
.

a -
:
1: m1a = F T1;
(2*)
2: m2a = T2.

m, (2*)
: ma = T1 T2. -
, m = 0, T1 = T2. (1*) -
,
T1 = T1 = T2 = T2. (3*)

75
T, -
(2*)
m1a = F T;
(2.31)
m2a = T.
:
F Fm 2
a = --------------------- ; T = --------------------- .
m1 + m 2 m1 + m2

3. m,
L,
R (. 2.14).

.

F = m g -

T . T -

: T T ,
. ,
. 2.14 2 2
T R T L R
------ = ---- ; ------ = ------------------------ . (4*)
T L T L

T F , T -
2
v
, -----
- ,
R
v . , (4*),
:
2 2
T L R
---------------------------- = mg ;
L
2
TR mv
-------- = ----------- .
L R

, :
mgL g
T = ------------------------ ; v = R ------------------------ .
2 2 2 2
L R L R
2 2
L R
, R
---- = sin ; ------------------------ = cos ,
L L
, :
mg gL
T = ------------- ; v = sin ------------- .
cos cos

76
4. (. 2.15), -
m1 m2, m2 > m1.
. -
, ,
, .

. 2.15


1 F 1 = m1 g

T 1. 2 F 2 = m2 g

T 2. -

T 1 T 1, T 2 T 2.

F 1 F 2 N .
,

T 1 = T 1 ( 1 ),

T 2 = T 2 ( 2 ),
(5*)
T 1 = F 1 ( ),

T 2 = F 2 ( ).

F 2 F 1
. , . 4.2.4, -
,
, F1 = F2. (5*):
T1 = T1 = T1 = F1 = F2 = T2 = T2 = T2. (6*)
T. -
,

77
, -
. , :

1: m1 a 1 = m1 g + T 1;

(7*)

2: m2 a 2 = m2 g + T 2.


a 1 a 2 1 2, ( a 1 = a 2;

a). m2 > m1, a 1

, a 2 . (7*)

g; :
1: m1a = m1g T;
2: m2a = m2g T. (2.32)

:
m2 m1
a = g --------------------- ; (2.33)
m1 + m2
m1 m2
T = 2g --------------------- . (2.34)
m1 + m2

,
4gm 1 m 2
N = 2T = ----------------------- . (8*)
m 1 + m2
(m1 = m2 = m),
( -
). (2.34) (8*)
T = mg, N = 2mg.
5. m1 -

, m2 (. 2.16). -
. -
.

: F 1 = m1 g,

N T 1.

F 2 = m2 g T 2,

T 1 T 2. -

N R. a 1, -

78
. 2.16

a 2 (a1 = a2, -
a). ,

T 1 = T 1; T 2 = T 2; N = N .
, ,
T1 = T1 = T2 = T2.
T.
,
, :

: m1 a 1 = T 1 + N + m1 g;

: m2 a 2 = m2 g + T 2.

a 1 ,
a1 = a T1 = T.

a 2 , a2 = a T2 = T.
:
: m1a = T m1g sin ; (2.35, )
: m2a = m2g T.
(2.35, )
:
m 2 m 1 sin
a = g ----------------------------------- ; (9*)
m1 + m2
m 1 m2
T = g --------------------- ( 1 + sin ) . (10*)
m1 + m 2
(9*) , -
,
m2 = m1 sin . (11*)
m2 > m1 sin -
(9*), m2 < m1 sin -
1 g ( m sin m ) 2
a = -------------------------------------------- .
m 1 + m2

79
2.4.4.

-
. -
; .

, -
-
.

.
1. .

F = m g ( ) N ,
. , -
,

F + N = 0. (1*)


N . ,

N = N . (2*)
(2*) (1*), ,

N = F . (3*)

, N
, . -

P (3*)

P = m g. (2.36)
2. .

F = m g ( ) T , -
. , ,

F + T = 0. (4*)

T . -
,

T = T . (5*)
(5*) (4*), ,

T = F . (6*)

, T , -

, . . P . (2.36).

80
, (. . 2.5),
.
.
-
, .
, -
.
.
,
-
.

. ,

. , , .
() , !
, ,
, , 

. , N !

2.14 2.18. 2.18 T 1 !

, T 2 . !

T 1 T 2 2.17 ,

T 1 T 2 2.15 .

2.4.5. ?
: .
(2.36) ,
( ) ,
, ,
.
( -

) a , -
, P
:
P = mg + ma, (2.37)
m . -
,
P = mg ma. (2.38)
(2.38) , (

g), P = 0.

81
) )

. 2.17

, -
( ).
(2.37) (2.38). -
, m ,
, .

m g N (. 2.17, ). -
,

ma = mg + N. (1*)

(1*) g;
mg N = ma, ,
N = mg + ma. (2*)
P = N, (2*) (2.37).

a , -
(. 2.17, ), (1*)

g, mg N = ma , -
,
N = mg ma. (3*)
, P = N, (2.38).
(2.37) (2.38) ,
.
. ,
, -

g
.
1. , ,
1 2 (. 2.18). 1 m1 = 1 ,
P = 12 . 2.
.

82
P > m1g; -
, 1 ,
2 . m2
2. (2.37) 1
:
P = m1(g + a), (4*)
a .
2 P 1,
-
,
, -
. (2.38) 2 . 2.18
:
P = m2(g a). (5*)
(4*) (5*), :
P
------- = g + a ;
m1
P
------- = g a .
m2

, a; -

P P
------- + ------- = 2g .
m 1 m2

,
m1 P
m 2 = -------------------------
- = 1,6 .
2m 1 g P

2. M
(. 2.19).

m, -

.
.


a , -


a = g sin (. (2.29)). . 2.19

83

a ( a ) ( a ) -
. ,
a = a cos = g sin cos ; a = a sin = g sin2 . (6*)

: m g, N ,

F . ( )
, -
.
,
ma = F;
ma = mg N.

(6*), :
F = mg sin cos;
mg sin2 = mg N.
:
N = mg cos2 .

P N ( -
). , N = N. ,
P = mg cos2 , F = 1--- mg sin 2.
2

3. , , -
M (. 2.20). -
m. .
.
a , ,
. , (2.33) m1 M,
m2 M + m:
gm
a = -------------------- . (7*)
2M + m

m g

N . -
,
ma = mg N.
(7*) :
2Mmg
N = -------------------- .
. 2.20 2M + m

84

P N ,
N ( ). ,
2Mmg
P = -------------------- .
2M + m

2.4.6.

. 2.3.4 ,
. , -
,
.
-
? ?
, .
2.21 -
. -
, -

. -
, . 2.21
; -

F , -
. -
,
, .

F ,
-
. - ,
. , -
, -
, .
, , -
.
.
, -
. , ,
, . -
. ,
.
: ,
?
, ?

85
, , -
, ,
. ,
. -

2.22. F 1 F 3 , -

, . F 2 F 6
, , . -

F 4 F 5 , -
, . ,

F 1 = F 2; F 3 = F 4; F 5 = F 6.

. 2.22

,
, ( ).
, ,
, ,
. .

, F3 > F6. F 3 ,
F 6 .
.

. , -
, . . ,
.

2.5.

2.5.1.
( )
,
, ,
. , -
. -

86
, -
. , -
(
). -
, . ,
, , , -
. ,
( , -
) .
-
, .
,
, -
, .
, - ,
,
,
, - . -
-
,
. -
, ,
, , -
- .
,
, ,
, ? -
, , -
. .

a m
N

, F j
j=1

F 1, F 2, F 3, , F N (,
N ). ,
N

F .

ma = j (2.39)
j=1

a -

. , m a
N

F , a a . F

j j -
j=1

F ( ),

87
, . .

N

F

m a = j + F . (2.40)
j=1
,

F = m ( a a ). (2.41)
, (2.41) (2.40), (2.39).
,
-
.
: , -
!
.
. -
. ,

( ).

2.5.2. ,


()

a . a

a = a a , (2.42)

a .
(2.42) (2.41), :

F = m a , (2.43)
m .
. m
(. 2.23), -

a . -

.

,
.

a -
. 2.23 . , -

88
, , , -

, : m g,

T ( ), F
(. ). -
:

m g + T + F = 0. (1*)
(2.43) (1*) x- y-;
:
x-: Tsin ma = 0;
y-: mg + Tcos = 0.
:
Tsin = ma;
Tcos = mg
. ,
a
tg = ----- .
g

, a ,
m. ; , -
,
:

m g + N m a = 0. (2*)

a . (2*)

g, mg N + ma = 0,
,
N = mg + ma. (3*)

a . (2*)

g, mg N ma = 0,
,
N = mg ma. (4*)


(3*) (4*), ,
. 2.4.5 
(2*) (3*). 
, .
,
, , .

89
2.5.3.

, -
R, -
. m, -

. a
;
(2.41) :

F = m a. (2.44)

a -
. ,
(. (1.22)):
a = 2R. (2.45)

a -

R , , -
. ,
(2.45) :

a = 2R . (2.46)
(2.46) (2.44),

F = m2R . (2.47)
(2.47) -

( F ).
-
m . -

: F , N F , -
( , ,
,
). -

a , -
:

ma = F + N + F . (2.48)


F + N = 0 (1*)
( ), (2.48)

: m a = F . (2.46)


m2R = F . (2.49)

90

F , m -

2R , .
, -
.
, -

( F + N +

+ F ) F . -
,
:

F + N + F + F = 0 (2.50)
, (1*),

F + F = 0. (2*)
(2.47), (2*) :

F + m2R = 0. (2.51)
: -
! (2.49) (2.51) ,
. (2.49) , -

F (
) m

2R . (2.51) ,

F m2R ,
. -

2R , .

(2*) , F -

F -
. , -

. F
( -

). F ,
, ,
.

2.5.4.

, -
, - . -

91


, .
-
.

F -
( , -

- R ) v

. . 2.5.3 F

, v = 0 ( )

. (2.47). F v , (2.47)

, v 0. ,
, -
,

F = m2R . (2.52)

, ( F ), -
, , -

v .
:

F = 2m( v ), (2.53)

( v ) -

v . ; -
-
(. . 1.8).
, -
, -
, (2.53) , -

, .

v , v -
2.24, S

v , -

. v
S. -
, ,
.

v

. 2.24 | v | = v sin . (2.54)

92
v B , = --- ;

2

| v | = v. (2.55)

v , = 0

= ; | v | = 0, . . -
.
.
- , -
, -
, .
. , -
, -
, . -
-
, .
,
, .
. , -
, .

2.5.5.

(2.53) ,
* m, , -
R, -
. -
R , -

v = v + R, (1*)
v .
2
v
, a a = ------ . (1*),
R

2
( v )
a = ------------ + 2v + 2R. (2*)
R

F , -
R ( , , -
, ). ma = F ,
(2*),
2
( v )
m ------------ + 2mv + m2R = F. (3*)
R

93
2
( v )
m ------------ = ma, a
R
, (3*) :
ma = F m2R 2mv. (4*)
(4*) , -

F . -
(4*) ; ,

F (
). ( m2R) -

F . 2mv. -
v B
, , -
(2.55), ,

v
2mv = 2m| |.
, -

F .
. 2.25 2.25.

2.5.6.

,
? ( ) -
, ?
1857 -
. m,
A v , -
, (2.53),
;
m (. 2.26). -

m, B
.
m -
, .
-
, -
.
, -
. 2.26 , , , , , .

94
2.6.

2.6.1.

F ,
(2.7) , :

p p = F t. (2.56)

p p

t, F

m; p = m v , p = m v

( v , v -
). (2.56) ,

, -
.
, t
. ,
, (2.56). -
(2.56).
:

p 1 p 1 = ( F 1 + F 12 + F 13) t;

p 2 p 2 = ( F 2 + F 21 + F 23) t; (2.57)

p 3 p 3 = ( F 3 + F 31 + F 32) t.

F i , i- ; F ij
, i- j- ; i, j = 1, 2, 3.

F i , F ij .
.
, (2.57); -


p p =

= [F 1 + F 2 + F 3 + ( F 12 + F 21) + ( F 13 + F 31) + ( F 23 + F 32)] t. (2.58)

p = p 1 + p 2 + p 3 , p = p 1 +

+ p 2 + p 3 .
( ),

F 12 = F 21, F 13 = F 31, F 23 = F 32.

95
(2.58) :

p p = ( F 1 + F 2 + F 3)t.
, -
, N :
N

F t.

p p = i (2.59)

i=1
:

.
,

. i Fi
, (2.59) -
t.

2.6.2.

, (,
, ), -
, .
(2.59) :

p p = 0. (1*)
(1*) ,
. ,
, . . ,
:
N

mi v i(t) = const. (2.60)
i=1

: v i(t) -
!
, -
,
(). (2.60)
,
:

, ,
, .

96
, , -
, (-
).


, i Fi 0.

,
,
. -
.

p .

p ,
, . ,
, -
(,
), -
- -
.
, (2.60) -
. , ,
,
.
,
, -
.

2.6.3.

, -

. -
,
.

. m1, v 1, -

m2, v 2

(. 2.27). v 1 v 2 -

.
.



m1 v 1 + m2 v 2, - . 2.27

97

(m1 + m2) v , v .

, m1 v 1 + m2 v 2 = (m1 + m2) v , -
,

1 1m v +m v 2 2
v = ----------------------------------
-. (2.61)
m1 + m2

(2.61) , v v 1

v 2. , x-,

, v 1 ( . ).

v 1, v 2,
v x- y-,
(2.61) :

m 1 v 1 + m 2 v 2 cos m 2 v 2 sin
v cos = ------------------------------------------------ ; v sin = --------------------------
-. (1*)
m1 + m2 m1 + m2


v 1 v , -

v1 v 2. ; -

v2(m1 + m2)2 = m12v12 + m22v22 + 2m1m2v1v2cos.

,
1
v = --------------------- m 12 v 12 + m 22 v 22 + 2m 1 m 2 v 1 v 2 cos . (2*)
m1 + m2

(1*) ,

m 2 v 2 sin
tg = ------------------------------------------------ . (3*)
m 1 v 1 + m 2 v 2 cos


( = ), (2*) (3*)

m v m v
- ; = 0 = 2.
1 1 2 2
v = ----------------------------------
m 1 + m2


m1v1 > m2v2 v -

v 1, m1v1 < m2v2 -

v v 2. m1v1 = m2v2 -
.

98

v 1 v 2
v1 > v2, (2*) (3*) ,
m v +m v
- ; = 0.
1 1 2 2
v = ----------------------------------
m1 + m2

2.6.4.

, -
, -
. : -
. , ,
, . -
,
.
(-
), .
.
. -
, -
. +
. -
,
,
,
,
, -
. .

. - , -
m1 m2,
( ).
,
. , -
m1 -

v 1, m2 v 2.

( p = 0); -

p = m1 v 1 + m2 v 2.

, p = p , . . m1 v 1 + m2 v 2 = 0. :
m
v2 = -------1 v 1. (2.62)
m2

99

, v 1 v 2 -

. , , v1 ,

m1, v 2
m2, -

. m1 n m2, (2.62) , | v 2| n | v 1|.
, M m -

v
() .
(
) ; m

u M,


v + u . M -

v + v 1. ,
, -


(M + m) v = M( v + v 1) + m( v + u ).
-


M v 1 = u m. (2.63)

(2.63) , v1 u .

, u -

v . M

v ,
-
v + um
-------- .
M

2.6.5.

-
.
.
-
. -
,
.

100

v -
t. t + t -

v + v .
, M

u . u -
. t
M. t
, , -

-
M ( t 0
-
). -
t M, t + t M M.
, t ,

M v . t + t ,
-, , M M

v + v , , -, ,

M v + u .
(
), -
:

M v = (M M) ( v + v ) + M( v + u ). (2.64)
,

M v = M v + M v M v M v + M v + M u . (2.65)

M v , M v .

M v , -
; , -
.
(2.65) ( (2.63)):

M v = u M. (2.66)
t;

v M
M ------- =
u --------- . (2.67)
t t

, v = v (t + t) v (t),
(2.67) t 0:

v ( t + t ) v(t) M
M ( t ) lim ----------------------------------------- =
u ( t ) lim --------- . (2.68)
t 0 t t 0 t

101
( t + t )
v v(t)
lim ----------------------------------------- a (t) -
t 0 t
M
( (1.14)). ---------
t
t . -

M
( t ) = lim --------- (2.69)
t 0 t

t.
(2.68) :

M(t) a (t) = u (t)(t). (2.70)

a (t)

F p(t) = u (t)(t), (2.71)
. -
-
.
,

Fp(t), F (t), (2.70)
:

M(t) a (t) = F p(t) + F (t). (2.72)

. -
( -
, -
).

2.6.6.

, N ( , -
) m1, m2, , mN. -

r 1(t), r 2(t), , r N(t) - -
t. ( , )
, - r C(t) -
:
N

m

r C (t)m = i r i (t), (2.73)
i=1

102
N
m = m . ( )
i
i=1
,
. ,
,
(. . 3.4.4).
-
r C(t + t) :
N

m

r C (t + t)m = i r i (t + t).
i=1
(2.73) -
t:
rc ( t + t ) rc ( t ) ri ( t + t ) ri ( t )
N
m -------------------------------------------- =
t mi -----------------------------------------
t
-.
i=1

t 0,
N

m

m v C (t) = i v i (t). (2.74)
i=1

v C (t) .
, , -
, , ,

m1 v 1(t) + m2 v 2(t) + + mN v N(t) = p = const
, , :

p
vC = ----
-. (2.75)
m
(2.75) , , -
, ,
.
, , -
-
. -
.
, -
* . , -
, , .
( , ) , -
- .

103
, .

+ . -
.
. , -
, , ,
, . -

, .
l + -
. (2.73) N = 2 x-,
(. 2.28).

l (m1 + m2) = 0m1 + Lm2. (1*)

. 2.28

m1 m2 , , L
. , 81 ,
(1*) :
Lm 2 1 L
l = --------------------- = L --------------- ------ .
m1 + m2 81 + 1 82
L = 384 000 , , l d 4680 . -
6370 . ,
+ ,
.

2.1. - , -
, . -
. -
,
. -
, -
? ?
2.2. 2.3. -
. -

104
- 2.11. ,
, ? .
2.4. , ,
- .
. -
2.5. ?
? 2.12. ,
2.6. . -
-
. , ,
- ,
? ,
2.7. ?
. 2.13. -
- .
, -

?
?
2.14. 5 . 2.4.3,
2.8. -
,
) ),
, -
2.29.
.
2.15. -
?
2.16. , -
, -
M.
: )
? )
-
. 2.29 ,
m? ) -
m ,
. ?
2.9. 1 . 2.4.3, 2.17. , -
, , -
, - 1 2.
.
a = 2 / 2
2.10. ,
P = 15 H, . -
, -
.
: ,
2.18. , -
, . -
, -
-
1 2. 1

m1 = 2 a , -
? 4 /2 -

105
. 2.22.
2. R
2.19. - l.
1 . -
-
0,5 /2. . -
- ?
2.23. ,
. ,

? , -
- ?
, - 2.24. , -
?
2.20. ,
. m

g, v0.
. - -
,
m, ?
2.25. . 2.6.3,
, -
(M + m) g.
?
, -
- , .
? 2.26. -
2.21. - H .
, - -
. - , -
- m v0.
. ,

- ? -
-
. .

106
3

:

, , -
,
.
(,
, ,
).
-
, . . , -
- .
.

. -
. ,
, , ,
. -
, -
,
, . , -
.
,
( ?).
, , -
,
. ,
. -
(
),
,
.

107
3.1.

3.1.1.

, , , -
, -, , -, (
),
.

F B ( -

)

=
r B F , (3.1)

rB , B

F ( . 3.1, ). -

r B F .

S , F B ( ,


r B). .
, , -
: rB

F , -
.
3.1, rB

. S -
, -
, .

) )

. 3.1

108


= rBFsin = lBF. (3.2)


r B F ( 2 ).
lB = rB sin (3.3)
.

B F . B
, lB = 0. -
.


F , 3.1, ,

S F . F B =

= r B F . , 

; .

(3.2) =
= rB F sin = l BF. , ,
3.1, .

3.1.2.

N ( F 1, F 2, F 3, , F i, , F N),
B ( -

) , -
:
N N

= i = r
Bi
Fi . (3.4)
i=1 i=1


r Bi , B

Fi ; i F i B.
, B -
; . -
, , -
z- ,
( ).

109
3.2 -
: -
a

( F 1, F 2, F 3), , B,
. -
B:

1 =
r B1 F 1; 2 = r B2 F 2; 3 =

=
r B3 F 3.
. 3.2
, 1 2
B ; -
-
z- , , z- , ,

a
--- F1 aF2. 3 B
2
, z-, z--
a
- F3. , z- z -
+ ------
2

( = 1 + 2 + 3)

z = a a
--- F1 aF2 + ------- F3 =
2 2
= lB1F1 lB2F2 + lB3F3 , (1*)

lBi i B. (1*)
3

l

z = i BiFi. (2*)
i=1

i = 1, i z-, i = 1,
. (2*)
N :
N N
z = i i = l i BiFi. (3.5)
i=1 i=1

ii z- i , lBi i ;

i = 1, i z-; i = 1, -
.

110

3.1, ( F F ).
(3.5) z-
B
z = lBF lB F. (3.6)

3.1.3.


(3.1), (3.2) F , -

B. -

S, B F . -

B z-, (. 3.3, ).

z-:
z = cos = lBFcos. (3.7, )

F z-.

) )

. 3.3

, -
. -
, B,
, -
B.
(3.7, ) : z = = 0.

, -
.

111

(3.7, ), F -

-
- , -

. F -
(

). F . -
A , -

, f , -

, F . -
3.3, z-;
, A,
Sz; O z-

Sz; F f ; l0 -

, A f . F
z-
z = l0f = l0Fcos , (3.7, )

= 1, , f 0, -
z-; = 1, . , -
3.3, , = 1.
(3.7, ) (3.7, )
. .

(3.7, ) , F
() z-, 90 180 , -
, z = 0. , z = 0 ,

z- F (
l0 = 0).
, , (3.5) (3.6) -
-
z-.

3.1.4.

,
, , -
. , -
,
( ).

112
3.4
,

F 1 F 2; F 1 + F 2 = 0; F1 = F2 F.
S ,

F 1 F 2
( ).
-


B. F 1 -

1 =
r B1 F 1, . 3.4

F 2 2 =

=
r B2 F 2. (3.4)
:

= 1 + 2 =
r B1 F 1 + r B2 F 2. (1*)

, F 1 + F 2 = 0, :

=
r B1 F 1 r B2 F 1 = ( r B1 r B2) F 1 = r F 1.
,

= r F 1. (3.8)

r = r B1 r B2 , -

F 2 ( A2) F 1 ( A1).
(3.8) ,
, (1*)

F 1 F 2. : F 1 F 2 -
B, -
, , B
! (3.8) , -
.
(. )
= rF sin = lF. (3.9)
l ;
, . , 3.4,

( ) , S.


F 1

F 2 F 2 -

F 1.

113
, -
A2.

F 2 F 1. -

A1, F 1 -

F 2.
, , -
, -
. -
- ,
( , -
). - -
. - . -
,
, .

(3.2) ,

= .
, -
(: ); 1 = 1 2/2.
- , -
1 , -
1 .

3.2.

3.2.1.


m, -
( , . 2.3.1).
A (. 3.5, ).

m g ( C)

T 1 ( A). -
, ,

. , , T1 = mg. -
T1 CA;
A (. 3.5, ). ,
T1 -
.

114
) ) )

. 3.5

. ,
, m, -
M, , m (. 3.5, ). -
2 T mg
T 2 m g a = ---------------------
- , -
m

. T 2
CA (, A A), ,
, .
: -

. -
, . - -
, .

3.2.2.


.
, -
? ?
3.6

F , A. -

(, )
B, -
. B


F F ( F = F ; F + F = 0). . 3.6

115

F F , -

A B. F F ;

F

B. , F ,
A,
, -

F , B, ,

F B, . .

=
r B F .

, N : F 1, F 2,

F 3, , F N. : A1, A2, A3, , AN, -
. , -

F 1. ,
:


F 1, F 2, F 3, , F N -

N

F , B,
i
i=1
, -
N N

r

B, . . i = Bi F i.
i=1 i=1

3.2.3.

-
, -
N , .
, -
, B, -
. -
: -

,
.

116
:

, -
:
N

F

i = 0. (3.10)
i=1

( -
):

,
,
, :
N

r

Bi F i = 0. (3.11)
i=1


r Bi , B

Fi ; B , -
(,
, ).
, , -
( S), (3.11)
,
(3.5):
N

l i BiFi = 0, (3.12)
i=1

lBi F i B, -
S; lBiFi . (3.12)
z-,
B S ( , -
z-).
, S c -
z- (
). i = 1 , -
, i = 1 , -
. , ,
B ( z-, B) -
, ,
, B

117
.
(3.12): , -

,
,
.
1. l m -

A B; AB = 3--- l (. 3.7). -
4
. 3.7 .

m g T A

T B (. ). (3.10) -

: m g + T A + T B = 0. -

g, :
mg TA TB = 0. (1*)
TA TB, . -
(3.12), , -
, A. (3.12)
:
1 3
--- lmg --- lTB = 0. (2*)
2 4
(1*) (2*),
TA = 1--- mg, TB = 2--- mg.
3 3
, , , -
(3.10), -
A, -
B.
2. l m -
-
, (. 3.8). -
M. -
.

: m g M g, -

T F .
( ). (3.10):

F + T + m g + M g = 0.

118

, -

x- y- :
Fx Tcos = 0; (3*)
Fy + Tsin (m + M)g = 0. (4*)
-
(Fx, Fy, T).
,
(3.12) 0
(. ). -
(3.12) : . 3.8
1
--- lmg + lmg Tlsin = 0. (5*)
2
(5*) :
1 --- m + M g
2
T = ------------------------------ . (6*)
sin
. (6*)

(3*) x- F :
Fx = Tcos = ( 1--- m + M gctg ,
2
)
(4*) y- :
Fy = (M + m)g Tsin = 1--- mg.
2

F .
:
F m
tg = ------y = -------------------- tg .
Fx m + 2M

F
M 2
F x + F y = 1--- mg 1 + 2----- ctg + 1 .
2 2 2
F =
2 m

3.2.4.
?
-
* h l (. 3.9, ). -
? , ,

N .

119

m
g , N -

F . . 2.3.1 ,
, m
g ,
;
C.

, N F
.
-
. 3.9, - BD.

) ) ) )

. 3.9


, N F A
(. 3.9, ).
, -

F BD. A.

N A .
, 3.9, . -

N A C; -

, N . C

: F F (F = F ; F + F = 0).
, 3.9, ,
, 3.9, . -

, m
g + N + F = 0, F F ,
, (-
).
1 1
= --- hF = --- hmg sin. (1*)
2 2
, -
,
. 3.9, ; -
. , 3.9,

. A N .

120

, N K -
m g (. 3.9, ).

N ,
C ,
h
= --- tg N. (2*)
2
N = mg cos , (1*) (2*) , = . -
(3.10), :

, , ,
. , ,
( -
C).

N ( -
) , , -
. ,
l
AK --- , . , -
2
, .

3.2.5.

,
. ,
-
, -
. -
, -
, , ,
. , , -
,
.

, (.
3.10, ). . 3.2.1;
, , -
A CA
(C ). : 3.4,
3.4, ( A)
C.
.
, .
, , , -

121
) )

. 3.10


F = T + m g (. 3.10, ). , ,
T T. T = mg, T = mg cos .
, A (
) ,
C (. 3.11, ).
,
. ( !)

. 3.11, ,

lmg = h sin mg,
h A C.
, -
A, ,
3.11, .

) ) )

. 3.11

122
( 3.12, )
.

F = N + m g, -
. (
3.12, ) -
.

F = N + m g, . -
(. 3.12, )
. -
.

) ) )
. 3.12

,
. , , -
, , .
,
, . -

.

3.3. ,

3.3.1. ,


F A F B, -
; A B (. 3.13, ).
,

R = F A + F B. (3.13)

123

F A F B
,
R = FA + FB. (3.14)
)
,

R .
, -

R

,
)
F A F B. -
. 3.13 -

R ; C. -

C R ; , -

F A F B

( F A F B -
). : AB = l, AC = lA, CB = lB.

F A C A = lAFA;

. F B -
0 B = lBFB;
. -
,
lAFA = lBFB, (3.15)
:
lA FB
----- = ------- . (3.16)
lB FA

, -
, -
, -
, -
(. 3.13, ).
-
A (. . lA) (3.15), ,
lB = l lA:
lAFA = (l lA)FB; lA(FA + FB) = lFB;
F B lF B
lA = l --------------------
- = ---------- . (3.17)
FA + F B R

124
3.3.2. ,

, - )


F A F B , -
(. 3.14, ).
FB > FA. (3.13) -
-

R :
R = FB FA. (3.18) )

(

F B, . . -
).
. 3.14

( C)

, F A F B -
. : AC = lA
BC = lB,
:
A = lAFA B = lBFB.

A

, B . -
, lAFA = lBFB, ,
l F
, ----A- = ------B- .
lB FA
. 3.3.1. , -
( C) A
B, , C
AB
( B). , ,
, -
,
, ; -
-
,
(. 3.14, ).

125


F A , , lAFA = lBFB
lB = lA l:
lAFA = (lA l)FB; lA(FA FB) = lFB;
lF B lF B
- = ---------- .
lA = -------------------- (3.19)
FB FA R

F B,

F A, , (3.18) (3.19),
, ,
lA . ,

FB FA R , , -
. FB = FA -

. F A F B -

.

3.3.3.

-
, , .
-
,
, .
x-.
x- -
? ?


.

R , , F 1

F 2; R R F 3 

R F 4 . . , 
, 

R . , x

R , 
. , , 
,
. .

126

N F 1, F 2, F 3, , F N,
( S).
x-
S, (. 3.15).
x1, x2, x3, , xN.
y-.
(z-)
S , -
(
. , . .4, . .10, ). S
xy- , z- -
, .

. 3.15

Fi i- ,
iFi y- . y- -
, , ,
i = 1, , , i = 1. -

R y-
N
R = F. i i (3.20)
i=1
-
O. z- (-
). z- -

, (3.5). z- :
N
z = xF.i i i (3.21)
i=1

127

F i ,
O .
z-, z- (3.21)

(i = 1). F i , -
O .
z-, z- (3.21)
(i = 1).
x0 x-

R . , , z- x0R -
O z--

, (3.21):
N
x0R = xF. i i i
i=1
(3.20) :
N

i xi Fi
i=1
x0 = -------------------------
N
- . (3.22)

i Fi
i=1

(3.22)
. :
N = 2, 1 = 2 = 1, x1 = 0, x2 = l, F1 = FA, F2 = FB.
B lF
(3.22), x0 = --------------------
- , . . -
FA + FB
(3.17).
, ,
: N = 2, 1 = 1, 2 = 1, x1 = 0, x2 = l, F1 = FA, F2 = FB. -
lF
B B lF
- = --------------------- , . . -
(3.22), x0 = --------------------
FA F B FB FA
(3.19).

, , ,
, ( xy-).

, y-
y- . ,
. ,
. ( -
!) x- ,

128
y- ,
.
, -
( ),
. .

F A, F B, F C (. 3.16, ).

) )

. 3.16

. D, AB,

F C F C , (

F C) F C (. 3.16, ). -
3.16, , -
3.16, ,

, F A, F B, F C, F C , F C .

F A, F C , F B , y-,

F C F C , x-.

3.4.

3.4.1. ,

. 2.3.1 , -
(, , , , -
. .)

129
. ,
-
,
.
, -
. ,
,
x-
(. 3.17).

. 3.17

N m1, m2, m3, , mN.


, , x1, x2, x3, , xN.
xC . -
, xC , x0

m1 g, m2 g, m3 g, , mN g,
. , xC -
(3.22), Fi = mig i
i = 1:
N

xi mi g
i=1
xC = -------------------------
N
- . (3.23)

mi g
i=1

g (3.23),
N

xi mi
i=1
xC = ---------------------
N
- . (3.24)

mi
i=1

130
1.
(. 3.18, ). : r1, r2,
r3, r4, h.
. -
.

. 3.18


m1 g, m2 g, m3 g, m4 g.
,
3.18, , x- (
). mi i- -
; mi = ri2h, , -
. xC -
, (3.24):
4 4

mi xi ri xi
2

=1 i=1
xC = i---------------------
4
- = ---------------------
4
-. (1*)

mi
2
ri
i=1 i=1

3h 5h 7h
3.18, , x1 = h
--- , x2 = ------- , x3 = ------- , x4 = ------- .
2 2 2 2
, (1*) :
2 2 2 2
h ( r + 3r + 5r + 7r )
1 2 3 4
xC = -----------------------------------------------------------
2 2 2 2
.
2 ( r1 + r2 + r3 + r4 )

131
2. r, -
--r- --r-
4 2
(. 3.19, ). .

. 3.19

MN; , -
. -
, . -
, -

FO , O,
, -
A B -
. -

F A ( r/4) F B ( r/2).
, -
3.19, , x- MN. xC
( ) -
-
. (3.22),
1 O x F x F x F 2 A 3 B
xC = ------------------------------------------------------ . (2*)
F O FA FB
:
1 1
FO = r2dg, FA = -----
- r2dg, FB = --- r2dg, (3*)
16 4
d , .

132
xi :

x1 = r, x2 = --r- , x3 = 3r
------ . (4*)
4 2

(3*) (4*) (2*), xC = 39r


--------- .
44

3.4.2.

xC yC -
, -
3.20, .

-
A ( mA, xA yA)
D ( mD, xD yD)
B )
( mB, xB yB).
)


( A,
D, B),
.
. 3.20
-
, x- -
. , , -
3.20, .
(3.24), xC
:

A A x m +x m +x m B B D D
xC = ------------------------------------------------------------
-. (1*)
mA + mB + mD

90 O
, y- -
(. 3.21, ). ,
, 3.21, . -
, yC
, (1*):

A A y m +y m +y m B B D D
yC = ------------------------------------------------------------
-. (2*)
mA + mB + mD

133
,
mA = a2d,
mB = (a + b)cd, mD = f2d, (3*)
d ,
.
) 3.20 3.21 ,
a+b
) xA = a f
--- , xB = ------------- , xD = a + --- ; (4*)
2 2 2
d+f
yA = d + a d
--- , yB = --- , yD = ------------ . (5*)
2 2 2
(3*) (4*) (1*),
. 3.21 xC. (3*) (5*) (2*), -
yC. ( , , -
.)
, N
, -
, -
xy-. , -
mi xi yi (i = 1, 2, 3, , N)
. xC yC
, -
(1*) (2*):
N N

xi mi yi mi
i=1 i=1
xC = ---------------------
N
- ; yC = ---------------------
N
- . (3.25)

mi mi
i=1 i=1

. -
, ,
(. 3.22). c,

. 3.22

134
a b. -
.
, -
, xC yC
. (3.25)
:
+ ( a b )m + 2am + 2cm
xC = 0---------------------------------------------------
-, yC = 0--------------------
-.
4m 4m

, xC = 1--- (3a b), yC = --c- .


4 2

3.4.3.

3.20, - A, B, D:


r A, r B, r D. r C - .
. 3.4.2
:

r m + r m +
A A r m B B D D
rC = ----------------------------------------------------------
-,
mA + mB + mD
N :
N


ri mi
=1
= i--------------------
rC N
-. (3.26)

mi
i=1

, (3.26) -
, ,
xy-. -
, - (3.26) -
x- y-.
xy-, -
z-; (3.26) -
,
(3.25) :
N

zi m i
i=1
zC = ---------------------
N
. (3.27)

mi
i=1

135
m.
(
), -
. N
(3.26) , -
-
:
1
rc = -----
r imi, (3.28)
m
i


r i mi - i- .
(3.28) (2.73), ,
( )
. , , .

3.4.4.

. 2.6.6,
* ( ) ,
. , -
. (3.23)
(3.24). (3.23) x- -
( !). (3.24) x--
; (2.73),
x-.
(3.23) (3.24), : -
g
(3.23). ,
()
g . -
, ( -
,
).
.
,

, .
- (3.24),

x m g i i i
xC = ------------------------
i
-.
i
mi gi

136
3.5.

: -
? (, )
,
?
. ,
, -
, , , -
.
(
). , , , -
, . , , -
, , -
.

3.5.1.

3.23, .

: R , f , F . R ;
O, -

. f A.

, O

; = Lf, L f . F

B. , -
O ; = lF, l

F . (3.12)
:
Lf + lF = 0. (3.29)

) )

. 3.23

137
,
:
f l
---- = ---- . (3.30)
F L
3.23, . -

, f F (
A B) O,
, O

f F , ( F > f). -

A f , -

O , F ,

B, , O
; = Lf = lF.
(3.12) :
Lf lF = 0. (3.31)
(3.30).

? ? ? -
?
, , -
(. 3.24).
: M.

M g N ; -

N + M g = 0 , , N = Mg. -

: R , f (

, ) P ;
.

P F , -
3.23, .
-

, P = N , -
, P = N.
-

f , -
,

N , -
; -
. 3.24 .

138
, ,
P. , -

f , P ,
L l.
P L
---- , , ---- .
f l
. -
, , , ,
, , -
. , -
, , , -
, .

3.5.2.
, .

, , .
.
(
) (. 3.25,
). , .
: ( 1), ( 2),
( 0, 1, 2) M, -

. A T 0;

. .
3.25,
, -

, S , -
2 -
. T-
, ;
. 0

T 0 T 0 , 1 T 1

T 1 , 2 T 2 T 2 .
F- ;
.

F 0 F 1 1,
)
F 1 F 2 2.
F-, 1 - )

N , 2 -

M g. . 3.25

139
, , , -
, . , ,
0. ,

T 0 + T 0 = m
, (3.32)
m ,
. .

, , T 0 + T 0 = 0 , , T0 = T0.
, . -
,
T0 = T0; T1 = T1; T2 = T2. (1*)
,
F0 = F1; F1 = F2. (2*)
, (2*) , -
( , . . 4.2.4).
,

T 0 = F 0; F 1 = T 1; T 1 = F 1; F 2 = T 2; T 2 = S .
,
T0 = F0; F1 = T1; T1 = F1; F2 = T2; T2 = S. (3*)
(1*)(3*), ,
T0 = T0 = F0 = F1 = F1 = T1 = F2 = T2 = T2 = S. (4*)

, T- F-,

S T 0,
A.

1 , F 0 + F 1 + N = 0,
2 :

F 1 + F 2 + M g = 0. (5*)

(5*) g, :
Mg F1 F2 = 0. (6*)
(4*) :
Mg 2T0 = 0, T0 = 1--- Mg. (7*)
2
, , -

, M g

T 0. ,
Mg
--------- = 2.
T0

140
, , ,
, , , -
.
, 2 .
: ,
(7*) .
, , (1*) (2*) -
- ,
(3*) . -
, (4*) -
. , (5*).
, -
. 2 -

a ,

(5*) F 1 + F 2 + M g = M a .
,
* 2 , .
, .
( ),
.
. -

-

. 3.26 , -

.
,
.

, T 0.
-
M. M g , -
F-,
, . 3.26
, . .
. 2N (N
N ), :
2N

M
g + F i = 0. (8*)
i=1

(8*)
g ,
2N
Mg F i = 0. (9*)
i=1

141
F1 = F2 = F3 = = F2N = T0, (9*) ,

Mg
Mg 2NT0 = 0, T0 = --------- . (10*)
2N
(10*) , , 2N -
, 2N .

,
(, -
-
). -

3.27. 1 : F 1, F 1, N 1; 2 -

: F 2, F 2, N 2. , F1 = Mg, M
. :

F 1 + F 1 + N 1 = 0 F 2 + F 2 + N 2 = 0. -
x-, :
N1 F1 F1 = 0;
(11*)
N2 F2 F2 = 0.
; , F1 = F1 F2 = F2.
F2 = N1 F1 = Mg, (11*)
:
N1 2Mg = 0;
N2 2N1 = 0.
, N2 , -
2, N2 = 4Mg.
2 .
4 -
.

. 3.27

142
3.5.3.

, -
, 3.28.
r
L,
,

A F . F , -
. A
(. -

); F
-
- . 3.28
.
, M.
. ,
, ,
.
(3.12) , -
, : rMg + LF = 0.
, F = Mgr
------------- . -
L
, , . -

A F ,

, L
---- F .
r

3.5.4.

-
, , -
.
, -
M
,
h -
. -
, ,
-
. -

F (. 3.29).

F ,

M g - . 3.29

143

N . , ,
( ). -
,

M g + N + F = 0. (3.33)
(3.33) x-, : Mg sin +
+ F = 0. ,
F = Mg sin . (3.34)
(3.34) , , -
1
F , ------------
- .
sin
( k) , -
,
1
k = ------------
-. (3.35)
sin
= 30, k = 2. = 15, k d 3,9. = 10,
k d 5,8. , .
0 ,
(,
).
, (3.33) -
,

F :

M g + N + F + F = 0. (3.36)
(3.36) x- y-, :
Mg sin F + F = 0;
Mg cos + N = 0.
, F = N (
),
F = Mg(sin + cos ). (3.37)
,

1
k = ------------------------------------
-. (3.38)
sin + cos
= 0,4 ( -
0,3 0,5),
k d 1,2 = 30, k d 1,5 = 15, k d 1,8 = 10.
. -
, --1- .

144
, -
,
-
M
r. ,

. -
.
-
( ) -
B, , -
A
-
. 3.30
T 1 (. 3.30). -

1 , T 1, T 1, -

2 T 2 T 2.

: M g, N , F 1, F 2.
:

M g + N + F 1 + F 2 = 0; (1*)
rF1 rF2 = 0. (2*)
, (2*) , -
C.
(1*) x-;
Mg sin + F1 + F2 = 0. (3*)
, , (2*), F1 = F2, ,
T1 = T1 , ,
T1 = F1, (3*)
Mg sin + 2T1 = 0,
:
T1 = 1--- Mg sin . (3.39)
2
,
2
k = ------------
-. (3.40)
sin
(3.40) (3.35), , -

2 , ,
-
.

145
,
3.31 ( ),
-
. . -
k1 = 2.
-
1
, k2 = ------------- (. (3.35)).
sin
. 3.31 , 3.30,
2
k = ------------- . ,
sin
k = k1k2.
, 3.30
.

3.5.5.

-
, , -
. , -
r
, ( BC,
) 2r (. 3.32). -
BC r,
( AB)

h = 2r tg, (1*)
. h -
( ).

. 3.32

146
,
( -
). -
.
. ,
, h . ,
, , -
h, ,
(1*).
, , ,

. m

,
F = mg sin ( (3.34), -
). -
1
------------
-.
sin
, -
. -

(, ). -
, ,
. M -
, L, , -

F = (Mg sin ) ---r- , (3.41)


L
, (1*).
, (3.41), -

.

.
(1*) ; 13. -
, sin = tg (3.41) -
:

F = (Mg tg ) ---r- = Mg ----------


h
-. (3.42)
L 2L
h = 0,5 , L = 40 , , F d 0,002 Mg.
( ) , -
10 , -
5000 .

147
-
, ,
; , -
. , ,
(),
( , , )
. ,
, .
, (3.41) (3.42) -
.
, -
()
( ). ,
,
:
.

3.1. (. 3.33).
? -
3.2. : ) C, ) -
. A, ) D?

-
?
O
3.3.
A
S.

-
A - . 3.33
: ) -
S; ) - 3.5. ,
S; ) 3.4.
- -
S? : ) CD,
3.4. , - ) AC, ) ,
- C
a = 8 , - , ) ,
C
B F 1 (F1 = 3 )
60
O F 2 (F2 = 6 ) ?

148

3.6. F 1 F 2, -
l. -
3.33, -


,
F 1 + F 2 . - ,
, -

F 1 + F 2
-
: ) O,
?
) B, ) D?
3.12.
3.7. m
. -
a -
-
.
,
ABCD.
,
:
) A, ) .
ABCD, ) ,
A B? - , -
?
- 3.13. 1 . 3.2.3,
. -
3.8. , B
3.7. M.
3.14. , -
: -
) AB, ) - , -
AC, ) , m1,
A ,
, -
, )
? m2.
3.9. - ?
- 3.15.

, - m. -
. -
. -
?
3.10. , , -
(-) ?
- 3.16. -
, -
. ,
3.11. -
h, , -

149
3.34? -
,
.
-
. A C
.

. 3.36


,
. 3.34 .
3.19. -
3.17. -, N
- , ,

, a.
3.35. -
? ?
3.20. -

100 ,
200 , 300 , 400 , 500 . -
-
15 .


. 3.35 ?
3.21. -
3.18. , , r = 10 -
, a = 8 .
, ,
(. 3.36). - l = 4
r = .
= 1 , R = 8 .
-
= 30. ?
3.22.
.
. - 3.37 -
- ,
, ; r = 11 .

150
.
?
3.25. -
-
,
3.39? -
M = 30 .

. 3.37

3.23. -
3.38. -
-
(a = 5 ).
. 3.39

3.26. ,

,
. -


,
?
. 3.38 3.27.
, -
3.24. 3.30,
-
, - , ?
, 3.28. ,
. - -
,
, 100 .

151
4

:

4.1.

4.1.1.


. -
;
1.1. ,
-
. ,
,
.
-
, -
:

-
, -

.

-
,
. -
-
, , , , ,
( , ). ,
, 1.1,
.

,

152
(, ). -
-
.
, , -

.
, ,
, -
, -
. -
,
.

4.1.2.

, -
, . -
, , .
. 1.4.2 1.4.3

( a ) , ( a ) .

a a , v
, , . .
. -
,
. , -
( , !) -
.

.
, . . -
. -



,

v , a , a .
. 1.4.4 -
, R
(. (1.20)). , -
R ,
(1.20) -
, . (1.20) -

t.

153
-
, -
, ,
-

z-.
S,
( -
4.1 ). -
S -
,
. 4.1 xz--
. t
( , ) (t). -
t t + t S ( , )
(t + t) (t). t
,
t t + t:
( t + t ) ( t )
= ----------------------------------------
-. (4.1)
t
, (t + t) (t)
t; .
t , , -
:

=
------- (4.2)
t
( (1.20)).
-
(t):
( t + t ) ( t )
(t) = lim ----------------------------------------
-. (4.3)
t 0 t
(t)
t 0.


, .
;
-
:
(t) = 0 + t, (4.4)
0 t = 0. -
, t, (4.4) -

154
0, (t) t (t), (t + t) t, . -
(4.4) :
(t + t) = (t) + t, (4.5)
,
( t + t ) ( t )
= -----------------------------------------
-. (4.6)
t

, (4.6)
t t + t.
t 0 -
(t):
( t + t ) ( t )
(t) = lim -----------------------------------------
-. (4.7)
t 0 t
, -
-
, .
R . t t + t
(t + t) (t);

S = [(t + t) (t)]R.
v(t) , ,
S ; ,
lim -------
t
t 0

( t + t ) ( t ) R.
lim ----------------------------------------
v(t) = - (4.8)
t 0 t
(4.3) (4.8) :
v(t) = (t)R (4.9)
( . (1.21, )).
(1.19, ) (4.9) -
() a(t) , -
R :
v ( t + t ) v ( t ) ( t + t ) ( t )
a(t) = lim ----------------------------------------
- = lim ------------------------------------------ R. (4.10)
t 0 t t 0 t
(4.7) (4.10) :
a(t) = (t)R. (4.11)

2
v (t)
- = 2(t)R.
a(t) = ------------ (4.12)
R

155
, -

.
1.9. -
. 2.5.4 -
. . -

,
(t) .

.
(4.6) ,

.
.
=

,
(: /2).
-
, -
1
1 /.
(. 4.2). 1 R ,
n T.
2 r. 2 3
R. 3 4
r. . -
4, -
E --r-
2
4.

. 4.2

n T, 2n
( ). , 1 1 1 = 2n
---------- .
T
(4.9), vA A vA = 1R.
vB B 2,
. , 2
v v R
2 = ------B = ------A = -----------
1
.
r r r

156
3 2:
R
3 = 2 = ----------
1
-.
r
2
R
vC 3: vC = 3R = -------------
1
-.
r
2
1 R
vD D 4: vD = -------------
-.
r
4:
2
v 1 R 2nR
2
4 = ------D- = -------------
2
- = -----------------
2
-.
r r Tr
vE E:
2
1 R nR
2
vE = 1--- 4r = -------------
- = -------------- .
2 2r Tr
E:
2 4
1 R 2 2 4
a = 1--- 24r = -------------
3
-
2 n R
= ----------------------- .
2 3
2 2r T r

4.1.3. ()

,
,
, ,
, , .

. -
() -
; -
. ,
, .
,
.
R, -
. ,

v C -
. -
.
.

157
) ) )
. 4.3

, A, C, B,
4.3, , -

t. C v C -
, . A t -
. , -
A . :
t , - -
, -
:

v A(t) = 0. (1*)
t -
A. , A
( -
,
). -
t A
. t + t -

.
()
, ,
, : -
. -
A t -
, .
A ,
C. , C
t A. -
vC, , (4.9), -
v
A A = ------C . B
R
t A; -

158
v
, vB = 2AR = 2 ------C R = 2vC. v B
R
AB. ,

v B(t) = 2 vC. (2*)
B C 4.3, .
-
, ,
. -

v C.
:

v (t) = v (t) v C , (4.13)

v (t) v (t) -
, .
,
(1*) (2*), A, C, B ,
:

v B(t) = v B(t) v C = 2 v C v C = v C ; (3*)

v A(t) = v A(t) vC = 0 vC = vC ; (4*)

v C = v C v C = 0.
-
. -
, (. .

, v C), -
(3*) (4*), , (4.9),
v
= ------C . ,
R
= A. (4.14)
,
, , ,
-
.
,

v , . 4.3,

v A, v B, v F, v D ( A,
B, F, D); R = vC -
.

(4.13), (

159
) . , F D -
:

vF = v F + v C (vF = 2 vC);

vD = v D + v C (vD = 2 vC).
4.3, , -

v B, v F v D.

1 (. 4.4).
R -
. -

V

. . -
,
. 4.4 ,
.
, A
, B , -

: vA = 0, v B = V . A -
v
V
. A A = -------
B
- = -------- .
2R 2R

: vC = AR = V
---- .
2
1
vC V ; , vC = --- V . -
2
, ,
V
, (4.14): = A = -------
-.
2R

2 (. 4.5).
-

V 1 V 2.
-
R, -
.
, -
. 4.5 ,
. , V2 > V1.
A , B

: v A = V 1, v B = V 2. (-

160
D) , , A
B D -
:
vA vB
--------- = ------------------------ . (5*)
AD 2R AD
, D AB

, v A v B (. ).
D
v
D = --------
A
- . AD ,
AD
(5*):
A v 1 V
AD = 2R -------------------
- = 2R -------------------- . (6*)
v A + vB V1 + V2
,
v v +v v +v V +V
D = ------A -------------------
A B A B
- = ------------------- 1 2
- = -------------------- . (7*)
R 2v A 2R 2R
D

C; v C
(. ), -
:
vC = D OD = D(R AD). (8*)
(6*) (7*) (8*),
V +V 2V V2 V1
1 2
vC = --------------------
2R
R(1 --------------------
1
V1 + V 2
)= -------------------- .
2
(9*)

,
, -
(4.13). (9*) vBR B
:
V V V +V
vB = vB v = V2 --------------------
2 1 1 2
= -------------------- .
2 2
,
v V +V
= --------
B 1 2
= -------------------- .
R 2R
, ,
(4.14), , -
D:
V +V
= D = --------------------
1 2
.
2R

161
4.1.4.

m -
, -
( . 3.4.3

). p (t) -

p (t) = m v (t),

p (t) = i i i (4.15)
i i

mi, v i (t), p i (t) ,
i- ; m i
i = m. (2.74) -

m v (t).
1
v C(t) = ----
- i i (4.16)
m
i
(4.15) (4.16) ,

p (t) = m v C (t). (4.17)
,
,
.

(4.17) , 
,

.

4.2.

4.2.1.

:

, -
m, R. -
R ,

162

. F ,

. ,
ma = F, a , -

F . , (4.11),
: mR = F, .
R,
mR2 = . (4.18)

= RF F .
(4.18) , -
( )
. -
i- -
. mi Ri
, ;

Oi . F i -
, i- ,

F i , .

i i Oi -

F i F i, .
z- ,
-
, ; -
z- -
. (4.18) i- , -

miR2i = iz + iz, (4.19)


iz iz , . . z--

i i, Oi (-
. 3.1.3).
, -
(4.19) , :

mR
i
i i
2 =
i
iz + .
i
iz (4.20)

i
iz = 0. (4.21)

163

(4.21) ,

, , 
, . , 
,
, , 
.

i
iz -

, - -
.
, - -
- .

i
iz ( -

) , -
. N, -

F j (j = 1, 2, 3, , N). ,
, . . z-,

f j. f j

fj = Fjcosj, j F j , -
z-. 4.6 , ,
j- ;
Sj Oj -

Aj. F j.
-

3.3, . f j Oj

j; z-
jz = ljfj = ljFjcosj, (4.22)
lj ,

Oj f j.
-
(3.7, ) (4.22) :
jz = jljfj = jljFjcosj, (4.23)

, , j = 1, f j
. 4.6 Oj z-, j = 1

164
. , 4.6,
j = 1.
N

i
iz ( , ) -
jz
j=1
z , (4.22), :
N N
z = jz = l F cos .
j j j j (4.24)
j=1 j=1
(4.21) (4.24), (4.20) :

mR
i
i i
2 = z. (4.25)

J = m Ri
i i
2 (4.26)

.
-
.
, z
, , (t), -
(4.25), (4.26), :
(t)
(t) = -------------
z
-. (4.27)
J
(4.27) -
:


, ,

,
,
.

: (4.27)
! (2.1) , -
, -
.
-
. -

a (t) (4.27) (t);

165

F (t) -
z(t) ,
; m
J .

4.2.2.
-
-
, , .
, ,
, -
. ,
.
,
-
m,

l (
, -
).
(4.26),
-
, -
4.7:
. 4.7
2 2
J1 = m --- + m --- = 1--- ml2;
l l
1: (1*)
2 2 2
2 3 2
J2 = m --- + m --- = 5--- ml2;
l
2: (2*)
4 4 8
3: J3 = ml2; (3*)
4: J4 = m(l d)2 sin2 + md2 sin2 =
= m sin2 (l2 + 2d2 + 2ld). (4*)
J1, J2, J3, J4 (4.26), -
. (4.26)
, , , -
. -
:
m R -
,
:
J = 1--- mR2; (4.28)
2

166
m R
,
;
m, R, l -
,
:
J = m --- R2 + -----
1 2
1
-l ; (4.29)
4 12
m R -
:

J = 1--- mR2. (4.30)


4
(4.30) (4.29) l 0 ( -
l
);
m R
:

J = 2--- mR2; (4.31)


5
()
m a, b, c -
a b, :
1
- m(a2 + b2);
J = ----- (4.32)
12
m l -
, ,

1
- ml2.
J = ----- (4.33)
12

4.2.3.

( XIX -
) :

J
J -
, , -
m
a :
J = ma2 + J. (4.34)

167
* .
m MN (. 4.8).
C MN, MN,
i- mi,
-
ri ,
. i- , MN
MN; Si. Si MN -
Ai, MN Ai.
r i -

R i, R i
h i (. ).

. 4.8


, R i =
+ R i.
mi ;

mi (Ri)2 = mia2 + 2mi
R i + miRi2. (1*)

m (R ) m + 2 m R + m R .
2
i i = a2 i i i i i
2 (2*)
i i i i

(4.28) ,
m i
i = m,

m R
J = ma2 + 2 i i + J. (3*)
i

168
JR J , , MN
MN, MN, a -

C. R i =
r i h i ,

m R
i i :
i

m R = m ( r ( m r ) m h .

i i i

i h i) =
i i i i (4*)
i i i i

m r


B h i,
h i = 0. i

i m
i
- , . -

m R
, . , i i = 0.
i
, (3*) (4.34), -
.

m l -
,
1
- ml2.
(. . ), , (4.33), J = -----
12
-
-
,
:

- ml2 + m( --- )2 = --- ml2.


1 l 1
J = ----- (4.35)
12 2 3
4.9 -
( 1, 2, 3, 4, 5), -
- . 4.9
m R.


? ?
J1 1 J2
2 : J1 = 1--- mR2 (. (4.28))
2
1 2
J2 = --- mR (. (4.3)). J1 J2
4
, :
3:
J3 = J1 + ma2 = m( 1--- R2 + a2); (4.36)
2

169
4:

J4 = J1 + mR2 = 3--- mR2; (4.37)


2
5:
J5 = J2 + mR2 = 5--- mR2. (4.38)
4
(4.26) ,

= .

,
- (: 2).
-
1 , -
1 .
-


.

4.2.4.

, -
,
.
,
,
.
(. 4.10). m -
R
m1 m2 (m2 > m1). ,
,
.
-
.

m1 m1 g

N 1, m2 -

m2 g N 2.

1 T 1 T 1,

2 T 2 T 2.

. 4.10 : m g,

170

N F 1 F 2.
.
, T1 = T1 T2 = T2 ( -

) N 1 = T 1 F 1 = T 1 ( -
), :
N1 = F1 = T1 = T1. (1*)
, T1 = T1 T2 = T2 ,

, N 2 = T 2 F 2 = T 2, :
N2 = F2 = T2 = T2. (2*)

a1 m1 (

), m2 ( ). -
a2
,
a1 = a2 a. (3*)
,

m1 a 1 = m1 g + N 1 m2 a 2 = m2 g + N 2.

g -
(1*)(3*), :
m1a = T1 m1g; (4*)
m2a = m2g T2. (5*)



. m g N

. F 1 F 2. -
= RF2 RF1. -
c (4.27) :
J = RF2 RF1, (6*)
, J , -
, (4.28), 1--- mR2. ,
2
a;
a
, = ---
- . (6*) -
R
:
1
--- ma = F2 F1 = T2 T1. (7*)
2

171
, (m = 0),
F1 = F2 , ,
.
(4*), (5*) (7*), :
m m
2 1
a = ------------------------------------
-g . (4.39)
1
m 1 + m 2 + --- m
2
a, 1 2 :
1
2m 2 + --- m
2
T1 = m1g ------------------------------------
-;
1
m 1 + m 2 + --- m
2
1
2m 1 + --- m
2
T2 = m2g ------------------------------------
-. (4.40)
1
m 1 + m 2 + --- m
2

4.3.

4.3.1.

m v ,


p = m v. (4.41)

L , -
O,
:

L =
r 0 p, (4.42)

-, O -
r0
; -


r 0 p . -
-
,

=
r 0 F , (4.43)

. 3.1.1. , (4.43) r 0

-, F

172
O, -
.
O z-. -

L z- ( Lz) -
z-.

z F z- (-
. 3.1.3).
(z-) -
m,
R. R.
Lz z-, -
,

v .
4.11 -
,
, , -
. -
, -


v

. -
-
(-
. 4.11

). ( z-) -
O --

r 0. L O, (4.41)
(4.42),

L = m(
r 0 v ). (4.44)
,

, L
(
r 0 z-) -


r 0 ( . ). r 0 B v ,

L
L = mr0v. (1*)
, v = R, (1*) :
L = mr0R. (2*)

173

, L z-. -
, R = r0cos . ,
1
L = mR2 ------------- . (3*)
cos

L , z-:
Lz = Lcos = mR2. (4.45)

4.3.2.

(4.45) -
, -
-
( , ) .
,
-
-
z-, -
. -

( 1, 2, 3),
, ,
,

(. 4.12).
. 4.12 1 2
z-,
(
). 3
z-, (
;
, ). -
z- O
, (4.44):

L i = mi (
r 0i v i), i = 1, 2, 3. (4.46)

mi i- , -
vi
,
r 0i - ,

O. L 1, L 2, L 3 (L 1 B
r 01,

L2 B
r 02, L 3 B r 03). , Lz1, Lz2, Lz3
z- . ,

174
,
Lzi ,
z-. , (
) ,
(4.45):
Lzi = miR2i. (4.47)
(4.46), -
O , :

L m (r

L = i = i 0i v i). (4.48)
i i
(4.47), z- , . .
z- ( ):
Lz =
i
L=
i
miR2i .
zi

(4.49)

(4.26) (4.49)
Lz = J. (4.50)


.
, -

L ) -

O ( L = i
i

, . . z-.
4.12 . ,
, , -

, L J ( (4.50)).
,

. -

-
-

L i + L i


, . .

(. 4.13). -
-
(4.50),

L = J. (4.51) . 4.13

175
4.3.3.

, (4.50) -
t, (4.50) :
Lz(t) = J(t). (4.52)
Lz(t)
z-, (t) .
-
: J(t) = z(t). (4.7) -
:
( t + t ) ( t )
- = z(t).
J lim ----------------------------------------- (4.53)
t 0 t
(4.52),
L z ( t + t ) L z ( t )
- = z(t).
lim ---------------------------------------------- (4.54)
t 0 t
(4.54),
:


-
, -
.

(
. -
, -
,
.)
z , (4.54)
:
Lz(t + t) Lz(t) = zt (4.55)
t t.

(4.54) z
(2.6) ( 

). (2.6) (4.54), p (t)

z Lz(t) , F (t)
z(t) ,
, . (4.27), (4.54), (4.55)

176
. (2.1), (2.6), (2.7) 
.

, (4.54)

(4.27), , -
. , -
(4.54) (4.27).
(4.42) ,

= .

,
- (:
2/). -
,
1 / 1
,
.

4.4.

4.4.1.

-
. (4.27) (4.50) -
, -
, .
, ,
,
(, -
) , -
. ,
.
-
, .
, -
-
.
,
. -

177
, -
, ,
.
( ),
.
.
-
( J1, J2, J3).
. , -

( Jmax) ( Jmin),
.
:
Jmax = J1 l J2 l J3 = Jmin. (4.56)
, . .
(, , , ,
, , . .), -
,
, ,
, -
J1 = Jmax.
, -
, -
. :
J2 = J3 = Jmin.

, -
(. 4.14, ).
2- ( n- ,
2
------
n
). -

) ) )

. 4.14

178
m a, b, c ,
(4.34), :
1 1
- m(a2 + b2);
J1 = ----- - m(a2 + c2);
J2 = -----
12 12
1
- m(b2 + c2).
J3 = ----- (4.57)
12

, -
, ,
(. 4.14, ). (4.28)
(4.30),
m, R l :
J1 = 1--- mR2; J2 = J3 = m --- R2 + -----
1 2
1
-l . (4.58)
2 4 12
-
, , -
,
, -
(. 4.14, ). (4.28) (4.30), -
m R:
J1 = 1--- mR2; J2 = J3 = 1--- mR2. (4.59)
2 4
. -
(4.31), m R

J1 = J2 = J3 = 2--- mR2. (4.60)


5
-
, .

4.4.2.

,

LC = J , (4.61)

L C ,

, J -
.
, , -
.
, ,
2- , -

179

(4.61) (4.51). L (4.51) -
,
, . -
,
.

4.4.3.

-
. : -
.
-
, .
-
? ?

, -
m. .
. (-
, ) -
(. 4.15, ). : KM
, K M, AB
, C .
,

, F F -
C, -
-

) )

. 4.15

180

. , F ,

N .
C ,
( Fl sin ,
l ). -

(. ). , -
,
.

F F R .
, -


f = F K f = F M.

f f
KM , ,
.
. -
KM, s (.
4.15, ). , ,
, -
O.

F, F, N O, -
O. O

F;
.
2mgs. ,
, -

F F .
, K

M f f ( f = F , f = F ).
, , .
.
KM. ,
. -
-
. , -

.
, -
. -
, (4.61) -
. -

181
,
. -
(4.61) (, , -
(4.50)).
-
(, ,
)
. -
(
), -
, .

4.5.

, -
(4.27) (4.54),
. : -
-
.
, . .
, -
.
.
,
.
. -
m -
O. ,
(4.44):

L (t) =
r (t) p (t),

p (t) ,
r (t) -,

O. F (t) (
).

p = p (t + t) p (t) d F (t)t,
t . t 0 -

( (2.6)):
p
lim ------- = F (t). (1*)
t 0 t

182

L

t t + t; L = L (t + t) L (t).
,

L
lim -------- =
r (t) F (t). (2*)
t 0 t


(2*). , L (t + t) =
r (t + t)
p (t + t)

r =
r (t + t)
r (t), p =
p (t + t)
p (t), -
:

L =
r (t + t)
p (t + t)
r (t)
p (t) =

= [
r (t) + r ][
p (t) + p ]
r (t)
p (t) =

=
r (t)
p (t) + r p (t) +
r (t) p + r p
r (t)
p (t) =

= r
p (t) +
r (t) p + r p .

r p , -
. -

L = r
p (t) +
r (t) p t,

L r p
-------- = ------ p (t) + r (t) ------- .
t t t
t 0:

L r
lim -------- = lim ------ r (t) lim p
p (t) + - .
t 0 t t 0 t t 0------
t
(1*) (1.6), :

L
lim -------- = v (t)
p (t) +
r (t) F (t).
t 0 t
,

p (t) = m
v (t), .
(2*).


(4.43), r (t) F (t)

(t) 0, F (t). ,

L ( t + t ) L ( t )
- = (t).
lim ----------------------------------------- (4.62)
t 0 t
-
.

183
: -
-
-
-
.

m1 m2, -
-

F 12(t) F 21(t). ,

; m1
. 4.16
F1(t), m2

F2(t) (. 4.16). L 1 L 2

O, L

: L (t) = = L 1(t) + L 2(t). -
(4.62), :

L
lim ----------1 = r1(t) F1(t) + r1(t) F12(t);
t 0 t


L
lim ----------2 = r2 F2 (t) + r2 (t) F21(t).
t 0 t

, -

F 12 = F 21,

L
lim -------- = r1(t) F1(t) + r2(t) F 2(t) + ( r 1 r 2) F 12(t). (3*)
t 0 t

F 12(t)
r 1(t) r 2(t)
(. ),
(
r 1 r 2) F12(t) = 0. (4*)
, -
:

1(t) =
r 1(t) F 1(t) 2(t) = r 2(t) F 2(t). (5*)
(4*) (5*), (3*) :

L ( t + t ) L ( t )
lim ------------------------------------------ = 1(t) + 2(t). (4.63)
t 0 t
-
.

184
(4.63) , -
.
, .

L i = r i p i i- , -

i(t) , i- (-
, , ).

(4.63), L (t)

1(t) + 2(t) ,

. . i L i (t) i i (t).

:

L ( t + t ) L ( t )
lim ------------------------------------------ = (t). (4.64)
t 0 t
:

L (t) = i r i (t) pi (t); (4.64, )


(t) = i i (t) = i r i (t) Fi (t) (4.64, )

(
r i (t) - i- , -

, F i (t) , i- ).

,

,
, -
.

(4.64) (4.54), -

, . (4.54),
(4.64) .
. , -
-
O, C. -
r i (t) i- , -
O, - r iC (t) ,
C,

r i (t) =
r iC (t) + r C (t), (4.65)

185
r C (t) -
(. 4.17).
O - -

L (t), -

L C (t).
,

L (t) = i r i (t) p i (t); (1*)


. 4.17 L C(t) = i r iC (t) p i (t). (2*)

,

L (t) = L C(t) +
r C(t) p (t); (4.66)

L = L C + r C p (t) + r C(t) p; (4.67)

L L
lim -------- = lim ----------C- +
t 0 t t 0 t i r C(t) F i (t). (3*)


p (t) = i p i (, p (t) =

= m v C (t), m , v C(t) ),

F i (t) , i- .

, (4.66),
(4.67), (3*). (4.65), (1*) :

L (t) = i [ r iC (t) +
r C(t)]
p i(t) =
i r iC(t)
p i(t) +
r C(t)
p (t).


(2*) (4.66). L (t + t),
t . (4.66) :

L (t + t) = L C(t + t) + [
r C(t) + r
C ] [ (t) + p ] =
p

= L C(t + t) +
r C(t)
p (t) + r
C (t) +
p
r C(t) p + r C p . (4*)

r C p ,

, L (t) (4*) (4.66),
(4.67). (4.67) t t 0:

L C r C
lim L p
-------- = lim ----------- + lim ---------- p (t) + r C(t) ( lim ------- ). (5*)

t 0 t t 0 t t 0 t t 0 t

186

r
lim ---------C- = v C (t)
p (t) = m v C (t),
t 0 t

rC
lim ---------
t 0 t-
p (t) = 0. (6*)
,

r

rC(t) lim p
------- =

C(t) F i(t). (7*)
t 0 t
i
(6*) (7*) (5*), (3*).

(4.65), (3*) :

L
L
r (t) F (t) r

lim -------- = lim ----------C- + iC (t) F i (t). (8*)
t 0 t t 0 t
i i
i i
(8*) -

(t) O, -

C(t)
. (8*) :

L L
lim -------- = lim ----------C- + (t) C(t). (9*)
t 0 t t 0 t

(4.64), lim L
-------- = (t). , (9*)
t 0 t


L C ( t + t ) L C t
lim -------------------------------------------
- = C(t). (4.68)
t 0 t
:

r

L C (t) = iC (t) p i (t); (4.68, )
i

r

C (t) = iC (t) F i (t) (4.68, )
i
(
r iC (t) - i- ,

, F i (t) , i- ).
(4.68) -

.
,
,
,
.

187
4.6.

4.6.1. ,

t1
(z-) Lz (t1), t2
Lz (t2). (4.55), Lz (t2) Lz (t1)
Lz (t2) Lz (t1) = z (t2 t1), (4.69)
z t1 t2 -
z-. -
,
.
, , -
:
z = 0. (4.70)
(4.70), (4.69) ,
t2 t1 -
, . . :
Lz = J = const. (4.71)
, -
.
:


, (-
) .


. -
, -
. -
, -
.
-
, -
-
. , -
,
.

188
: -
,
. -
. -
-
, .
, -
, Lz (4.71) -

L
j
zj = J
j
j j = const, (4.72)

Jj j z- j- -
, . , -
, , -
.
, (4.72) :
Lz1 Lz2 = J11 J22 = const. (4.73)
,
J1 J2 . -
-
, ,
J1 -
1. -
, J2
J
2 = ------------
1 1
.
J2
, -
.
. , -
,
,
. , -
. ,

( )
, .
-
, (.
4.18, ). ,
, -
. -
,

189
(. 4.18, ).

( -
).

) )
. 4.18

.
,
, , -
, ,
. :
,
. -
,
,
.
,
.

, F ,


, -
. 4.19
(. 4.19). -

. ,

F , -
. -
. , -
.

4.6.2.

,
, ,

190
. (4.68) -


L C(t + t) L C(t) = Ct (4.74)
t t.
C = 0, (4.75)

(4.74) , t t: L C(t + t) = L C(t).
, -


L C = const . (4.76)


, -
.


, ,
( . 4.6.1) :

, , (4.74) ( . 4.6.1 !
(4.69)).
. !
(4.68). ,

c = 0, (
) . !
!
, . . .

-
,
,
(4.64):

L = const , = 0 (4.77)

( L -
, -
). -
:

-
,
.

191
, - -
() .
, -
, . .

(4.75). (4.75) , L C = const , ,
, ,

L C -

L C = J , J
( (4.61)). ,
,

L
= ------C- = const . (4.78)
J

,
, , -
-
.

-
, . .
, ,
.
-
, . . -
. (,

J = 2--- mR2, m , R .)
5
-
; 66,5 -
. -
, -

.

, -
( , )
. , -
-
, , -
.

192
4.6.3.

,
(
),
, -

-
.
4.20. 1 -

2 AA1. 1 -
3, -
BB1
. 4.20
AA1. 3 -
DD1 4; DD1 BB1. -
1 3, -
DD1 .

-
, .
:

L C = const . DD1
,
, -
2.
;

L
= ------C- = const , J
J
.
, -
* , , , ,
. . -
, (
) .
-
. , , -
.

4.6.4.

,
, (
) .

193

: m g, N

F ,
. , -
(4.64).

O N F , . . -

. N F
O.
,
. ,

L O

(L = const )


= L
---- = const , (1*)
J

J ,
.
; ( , -
) .

. ,
; -
,
, -
(. 4.21, ).
-
. , n
,
. , -
.

) )
. 4.21

194
,
, ,

. L const const ,



= L
---- (2*)
J
( ).
-
? , O?
-

O, m g:

0(t) =
r C (t) m g, (3*)

r C (t) - ,

O. 0 (t) -


r C (t) m g .

4.21,
r C(t) m g

S(t) . 0 (t)
;
0 = rCmg sin , (4*)
rC O C.
t t + t.

0(t) -

L , , (4.64),


L = 0(t)t. (5*)

L , 0(t), -

S(t). L ,

t L (t)
(. -
)
L
= ----------------
-. (6*)
L sin
(4*) (5*) (6*):
0 r C mg
------- = ----------------- = --------------- . (7*)
t L sin L

195

L (t) -
S(t); L .
(2*), L = J. (7*), -

r C mg
------- = --------------- .
t J

,
------- = . ,
t
-
:
r mg
= ---------------
C
. (4.78, a)
J
, .
:
! , ,

L ( ,
) ,
,
O.
,
, -
.
( ) -
, . , -
, z-, -
x-, -
y-.

4.7.

4.7.1.


, -

,


p (t) L C (t)
.

196
:

p ( t + t ) p (t)
lim ----------------------------------------- = F (t). (4.79)
t 0 t
:

p (t),

p (t) = i (4.79, )
i

F (t).

F (t) = i (4.79, )
i



(); p i (t) F i (t) i-
, i- .

, p (t) = m v C (t):
, m ,

v C (t) . ,
(4.79) ,
, -
.

p (t) = m v C (t), -
(4.79) :

Cv ( t + t )
v (t) C
m lim ----------------------------------------------- = F (t), m a C(t) = F (t), (4.80)
t 0 t

a C (t) .
(4.68):

L C ( t + t ) L C ( t )
- = C(t).
lim ------------------------------------------------
t 0 t
(. (4.68, ) (4.68, )):

r

L C(t) = iC(t) p i(t),
i

r

C(t) = iC(t) F i(t),
i

r iC(t)
- i- ,
. -
-
.

197
(4.79) (4.80) -
, (4.68) -
.

4.7.2.

-
(4.80) ( (4.79)) (4.68), . . -
, ,
(x-, y-,
z-).
()
, -
.
xy- ,
.
:
maCx(t) = Fx(t), (4.81)
maCy(t) = Fy(t), (4.82)

aCx aCy a C.

xy-. , LCx LCy

LCz(t):
L Cz ( t + t ) L Cz ( t )
- = Cz(t),
lim ----------------------------------------------------- (4.83)
t 0 t
z-
:
( t + t ) ( t )
- = Cz(t), J(t) = Cz(t).
J lim ----------------------------------------- (4.84)
t 0 t
, (4.83) (4.84)
(4.54) (4.27), . z- -
, .
,
(4.80) (4.68),
,

xy- ( (4.81) (4.82))
z-,
( (4.84) (4.83)).

198
4.7.3. ()



() m R -
. -
, -
. :

m g, N

F (. 4.22).

, F . 4.22
.

v C (t) -
t
vC (t) = RA (t), (1*)
wA(t) A, -
t .
(t) -
, ( ,
). (4.14),
(t) = A(t). (4.85)
(4.85) (1*) :
vC (t) = R(t). (2*)
-
,
.

a C

C v ( t + t ) v ( t ) C
aC = lim ----------------------------------------------- .
t 0 t
(2*), :
( t + t ) ( t )
aC = R lim -----------------------------------------
-
t 0 t
, (4.6),
aC = R, (4.86)
,
(aC , -
).

199
, ,
-
, -
. ,
:
maC = mg sin F; (4.87)
J = RF, (4.88)
J -
.
( (4.86)(4.88))
(F, aC, ). , :
J sin
F = mg ---------------------
-;
2
(4.89, )
J + mR
2
mR sin
aC = g -------------------------
2
-; (4.89, )
J + mR
2
sin
= gmR
------------------------------
2
. (4.89, )
J + mR

J = 1--- mR2, :
2

F = 1--- mg sin; (4.90, )


3

aC = 2--- g sin; (4.90, )


3

sin
= 2--- g ------------
-. (4.90, )
3 R

4.7.4.

, , -
-
. m, R. ,
, : , -
, . aC -
, -
, . -
aC ,

200
(4.89, a), -
J = 2--- mR2. :
5

F = 2--- mgsin; (4.91, )


7

aC = 5--- gsin; (4.91, )


7
sin
= 5--- g ------------
-. (4.91, )
7 R

.
. ,
h, .
?
T, -
h
, aC, l = ------------
- , -
sin
1
. (1.26), l = --- aCT2. -
2
2l 2h
, T = ------ = -------------------- . (4.91, ), :
aC a C sin

14h
----------
5g
T = -------------- . , .
sin

4.7.5.

, ,

(
). -
, , -
, .
(4.86) (4.88), F -
R
Ja C
F = J
------ = ---------
2
-, (4.92)
R R
J aC
().

201
, (4.92)
, -
N, -
, N .
, -
:
Ja C
2
- < N.
--------- (4.93)
R
m, (4.93)
J = 1--- mR2. :
2
1
--- maC < N. (4.93, )
2
m, (4.93)
J = 2--- mR2. :
5
2
--- maC < N. (4.93, )
5

ac (4.90, ), -
N mg cos . , -
(4.93, ):
1
--- tg < . (4.94, )
3
aC
(4.91, ). N = mg cos , (4.93, )
2
--- tg < . (4.94, )
7
, m R


F ;
. -
.
aC = R;
maC = F F; (4.95)
1
--- mR2 = RF,
2
aC, , F. ,
aC = 2--- ----
F
-. (4.96)
3 m

202
(4.93, ) -
:
1
--- F < mg. (4.97)
3

(. 4.23). m1
-

m2,
, .

-
, - . 4.23
?
,
.
, (4.97), -
:
1
--- T < m1g. (1*)
3
T , :
m1a = T F;
m2a = m2g T; (2*)
1
F = --- m1a,
2
a (-
(2*) (4.86) (4.88)).
:
3m m g
1 2
T = ----------------------------
-. (3*)
3m 1 + 2m 2
(3*) (1*), :
m
> ----------------------------
2
-.
3m 1 + 2m 2

4.7.6. ,
?

, , . ; -
* , -
, . ?

203
: m
g (m ),

N F (. 4.24, ).
( A ), -

, , N F

F
.
c = --------
- .
m
. -
. ,
.
,
C ,

. N m
g

C; F . ,
(. )
= RF, (1*)

R . C
RF
= -------------
- , J .
J

. , F , -
, -
, .
?
, -
, . . .

.
, . 4.24, -
4.24, , , -

) ) )

. 4.24

204
.

N
l, ,
, , -

. N C,

N = lN = lmg. (2*)

N . lmg > RF,
C
,

N lmg RF
= ----------------------- = ----------------------------------
-. (3*)
J J
(3*) ,
( l)
.
, (3*). -
, , . -
,
, , (
4.24, , 4.24, ). -
, , -
. -
.
, -
.
. -
.
, F
:

N
F = ----------
-, (4.98)
R
. -
. -
. -
, .
,
, (4.98).
-
,
, -
.

205

4.1. - 4.7.
ABCD a = 20
- -
n = 4 c1 (4 - v. -
) , -
R.
-
D. . -
A B, -
? ?
4.2. ?
. 4.8. OC,
,
- ,
: ) R, -
, )
? R (. 4.25).
4.3. -
, 20,5 - B ?
.
2 ? -
. ?
4.4. -
-
= 40 /2.

T = 5
?
5 ?
4.5. -
-
40 -
2000 /. . 4.25

40 ? 4.9. -
4.6. -
?
( t + t ) ( t )
= ----------------------------------------- 4.10.
- t
t
t. - a;
? -

206
-
m. - ? m, -
, - r,
: ) R.
- 4.16. -
, ) l, -
, ) - -
m1
- m2. ,
, ) - -
,
? -
4.11. - .
- 4.17. , -
, ,
? m, m1 m2. -
a.
4.12. - ,
- -
? m, - .
a. .
4.13. m1 a1 + m 2 a
2
, ----------------------------------
-.
m1 + m2
-
m r. 4.18. -
, m.
l. 4.19. -
, l
-
m


?
.
z-,
C, -

.
4.14. -
m -
r ,
-
?
4.15. -
.

-
. 4.26

207
4.26; AC = BC. ,
- ?
- 4.24. ,
B? 4.10, -
? - ,
- : )
z-. -
4.20. - ,
- ) -
4.26, .
-
B? ?
? 4.25.
- M, -
C. R,
4.21. m -
m = 2 - a. -
r = 10 -
- .
= 4000 /
T = 4 . -
- ?
, 4.26. -
. M R -
4.22. m - .
r -
, - m. -
h (h < r). -
- , -
,
, - r
?
?
.
-
.
4.27. m -
.
4.23. - M -
R, -
, J = 0,03 2.
, n = 250 - .
. -
= 0,5 ,
. -
r v
? ?

208
. , ,
-

.
(v r), -
. ?
4.28. - -
M R -
l (l n R) -
,
m. -
?
.
4.31. ,
-
4.23,
, m
? : ) , ) -
4.29. 150 -
, ) ,
160 / . -
-
0,8 /. ,
,
.
5 4.32.
? -
4.30. ?
= 30 ?

209
5

5.1. ?

.
. -
.

5.1.1.

. -
. , ,
, .
. -
, ,
, -
. -
.

-
, , -
.
-
, . .

. -
, , . .
.
-
-
. , -
. , , -

210
, .
-
, .
, -
. ,
, , -
, -
,
.
.

5.1.2.

, .
( ).
, , -
, , .
, (
). , -
, ,
. ,
, , .
: -
? ?
, ,
, ?
, -
, .
-
, -
.

. ,
, , (
). , -
( -
),
( ),
( ), ( -
, , ).
. .
, - -
. , .

211

. - -
( ), - -
( ).

5.1.3.

, , ?
,
. -
,
.
, -
. ,
,
. -
- . ,
, . ,
, ,
, .
,
. -
, , .
, , -
, .

. -
, , . .
, . ,
.
( , -
, - ) -
.
- ,
, , -
. : , -
.
. -
,
.
; ,
, , .
; -
, , -
.

212
-
. , -
,
.

5.1.4.

, ( -
) ,
,
, -
,
. ? ? -
, ?
,
.
,
, ( -
momentum, , ). -

mv, . . ( ) . -
-
mv2, .
2
mv
----------
- ,
2
. -
,
( ).
: ,
, ,
. , -
, .
, -
: -
.
. -
: -
-
. -
,
,
, , ,
. , ,

213
,
. -
, -

, , -
.

,
, -
.

5.2.

-
. . -
, , -
, ,
, ,
. . -
.
.
, .

5.2.1.


, F , -
( ) -

M N. . F
,
(
M N).
, .

, F ,
.

F
s
A = Fs cos , (5.1)
,
.

214
( = 0), (5.1)
:
A = Fs. (5.2)

0 m < --- , cos > 0 , , A > 0.


2
, s.
= --- , A = 0, . . s.
2

--- < m , A < 0 s -
2
.
,
. , - -
.
-

. r
, M
N.

F r
A = Frcos, (5.3)

F r .

,

F r -

F r :


A = F r . (5.4)

, (5.3) ( , (5.4)) -
(5.1),
s = r. -
,
-
, ,
. -
, ,
, . s -
( ) -
r (. 5.1). , -
, ,
( . 5.2.4). . 5.1

215
(. 5.2).
m ;

r -
N. N

; r -
. N
h, r = r = N.
m g.
-
. 5.2
r :


A = m
g r = mgh cos = mgh.


m
g r -
:

A = m
g r = mgh cos 0 = mgh.

(5.2) ,




=
.

, -
- -
(1 = 1 /).
-,
(: )
;
1 = 1 .
, 1 -
1
.
.

5.2.2.

N ( F 1, F 2, , F N).

Ai F i r
. (5.3) :

Ai = Fir cosi = Fi r, (5.5)

216

Fi = Fi cos i Fi -

r , i Fi r . (-

) r
N N
A = A i = r F . i (5.6)
i=1 i=1

Fi -

() R r :
N

F i = R. (5.7)
i=1
,
N
A = A i = Rr. (5.8)
i=1
(5.8) , (5.4),
N

F r = ( F

A = i 1 + F 2 + + F N) r = R r . (5.9)
i=1
, ,
( ) (-
) .
(. 5.3). m
-
. ,
h?

m g, N

(N = mg cos ), F (F = N = mg cos ).
-
, -
. , -

r . -
,
(5.8), R

m g, N F

r :
R = mg sin F = mg(sin cos ). . 5.3

217
h
(5.8) , r = ------------
- , -
sin
A:
A = mgh(1 ctg ). (5.10)
, 1 l ctg , . . tg l .
tg = ; -
-
. tg <
. -

, N B r .

5.2.3.


: F
,
(-
) .

M
N , , -
-
. 5.4 , -

. F i r i -
i- (. 5.4).
(5.4)
, i- ,

Ai = F i r i.

, F M
N, , , Ai
:

A = F r .

A = i i i (5.11)
i i
, (5.11) -
, .

5.2.4.

, MN

F

218
. 5.5

, .
(5.11)

r ).

A = F (
i (5.12)
i

r

i
i
(. . 5.5):

r

i = r . (5.13)
i
(5.13) (5.12), (5.4) -

r . , , -

,
.
1 (. 5.6).
-
m

MN ,
h. -

M N?
- . 5.6

( m g), A -

r (. ):

A = m g r = mgr cos ,
. = + --- ,

m g r
2

A = mgr cos ( + --- ) = mgr sin = mgh.


2

219
2 (. 5.7). m
h, MNN, -
; N h (h > h).
-
MN MN?

. 5.7


: m g

N .

N
; , -

N , , M N
. , -
,

, m g

: r MN r
MN. :
MN

A = m g r = mgrcos = mgh;
MN

A = m g r = mgrcos = mg(h h).
: A < A

.

5.2.5.


F
M N x-.

Fx F x-. M
N , ,

220
( -
). A,

F M N,
Ai, -
:

A = F x = F x cos .

A = i i i i i i (5.14)
i i i

Fi

xi i- , i

Fi xi

(. 5.8). Fi
x- Fxi = Fi cos i. -
. 5.8
, (5.14) :

A = A = F
i
i
i
xi xi. (5.15)

5.9,
Fx (x). -
Fxi , Fx(x)
. Ai = Fxixi
, . A
M N
.
M N
,
Fx (x) A -
, Fx (x) x- -
M N. ,
5.9, .

) )
. 5.9

221
5.2.6.

, -
( , ) k,
m.
. -

m g

( , ) F (x).

5.10, m g F (x), x- -
; , . . x = 0, -
. -
, x,

F (x) x- , ,
Fx(x) = kx (5.16)
( (2.13), F -
x = l).

. 5.10 . 5.11


m g F (x)
x- O N ( x = 0 x = l). A1

m g
A1 = mgl;


5.11, . A2 F (x) .
(5.16), -
(. 5.11, ).
,
2
( l )
A2 = k----------------
-. (5.17)
2

222
, , -
x = 0 x = l,
2
A = A1 + A2 = mgl k (------------
l )
-.
2

5.2.7.

(5.4) F

r . -
t t t + t.

, r =
r (t + t) r (t), r (t) r (t + t) -

- F t
t + t, . , (5.4)
:

A = F r = F (
r (t + t) r (t)). (5.18)

? : F
A?

, . N,
A t,
A
N = -----
-. (5.19)
t
, t -
.

, F ,
, -
.
(5.19)

r
N = F ------ = F v . (5.20)
t

v = r

------ -
t
; t (. -
(1.3)). t

F -

( v ).
t 0

v (t) ( . 1.3.1).

223
(5.20), ,
:

r ( t + t ) r(t)
N(t) = lim N = F ( lim --------------------------------------
- ) = F v (t). (5.21)
t 0 t 0 t

(5.21) F .

F (t) , t,
(5.21) :

N(t) = F (t) v (t). (5.22)

,
,
. , ,
, .
.
, , -
. (5.22), .
! , -
, , -
. :
, .
( ) -
. -
.

(5.19) ,

= .

- ,
(-
: )
; 1 = 1 /. ,
1 1 .

5.3.

5.3.1.

, -
. -
,

224
(
), . , ,

.
?
?
, ,
, .

1 2 F , -

r

( , 2). , . . A = F r , -
, 1,
, 2. , ,
, , -

.
, ,

.

M, B -
, f
A (. 5.12, ).

3.24 . 3.5.1. )
, -
:
N L
---- = ---- , (5.23) )
f l
. 5.12
l L
, ; N = Mg.
h -

N r 2 (r2 = h),

f r 1 (. 5.12, ).
r
--------1- = L
---- , (5.23) :
r 2 l

N r 1
---- = --------- . (5.24)
f r 2
(5.24) : , -
, ,
, r1,

225
, r2,
. ,
. -
.
: , , .
, ,
(,  . .).
(5.24) :
fr1 = Nr2. , :
A1 = A2, (5.25)

A1 = fr1 f r 1, A2 = Nr2

N r 2. (5.25) -
.

, -
, , -
, XVIIIXIX -
. 1826
,
( ) ( ).

(5.25) , , -
( A1), , -
( A2). -
,
. -
: , ,
. , ,
, ,
.
(-
, , . .), -
, -
, , -
.

5.3.2.
?

, -
,
. ,

226
.
-
.
, ,
.
: ,
! ,
. , -
, ,
.
, -
.
, -
,
. , -
. -
- , -
.
.
( ) -
,
.

5.3.3.

,
, . -
, . ,
-
, , . . ( ).
, , -
, -

F , , r .
.
-
( , , -
).
, , -
. -
. -
(

227
, , -
).
, ,
. -
.
, -
, , -
.
, -
:

, ,

, , -
, , -
, . -
, ,
. -
, -
(
). ,
-
( ).
,
.
, -
, -
.
(
-
). -
. ,
. ,
.
-
.
.

228
5.4. 1)

5.4.1.

m t v (t), -
, -

K(t) = 1--- mv2(t). (5.26)


2

:


, .

, , m,

t = 0 v 0, -

F . -
. 5.4.2.
,
(1.23)

v (t) = v 0 + a t, (5.27)

a . t -

1
r = --- ( v (t) + v 0)t. (5.28)
2
, (5.27)
1
r = v 0t + --- a t2, (1.26).
2

F
, a = ----
- ( ), -
m

v (t ) v
(5.27): F = m ----------------------0- .
t

v (t) v
r : F r = m ----------------------0- r
t

1) . 5.4.15.4.5 .

. 5.4.6 5.4.8 ,
.

229

r (5.28); -
:

F r = 1--- m( v (t) v 0) ( v (t) + v 0),
2
:

F r = 1--- mv2(t) 1--- mv20. (5.29)
2 2

t: K(t) K0. -
, t. ,
A = K(t) K0. (5.30)
.
A , (K(t) K0) > 0, . . -
. A -
, .
,
, .
,
, , , -
, . ,
, ,
.
. m = 600
h = 1,5 = 45,
= 0,4. -
v0 = 2 /.
?
,
K K = A + K0,
h 1
A r = ------------
- , K0 = --- mv20
sin 2
. A -
(5.10), :
K = mgh(1 ctg) + 1--- mv20 = 6,5 .
2

5.4.2.

*
F . , -

(5.30) , F ,

230
, , -
.
0 t N -

( = t/N) , F

. , r

r i (i = 1, 2, 3, , N) ( 5.5).

r i -
(5.30), i- -
:
Ai = K(i) K((i 1)). (1*)
(1*) i 1 N:
N N

Ai =
[K(i) K((i 1))]. (2*)
i=1 i=1

A i A 0 t (

r ). (2*)
K() K0 + K(2) K() + K(3) K(2) + + K(N) K((N 1)).
,
: K(N) = K() K0. , (2*)

: A = K(t) K0 , , F
.

5.4.3.

, -
, -
, , ,
.
, ,
, ,
, -
. , , -
,
, -

. , -
. -
-
-
(. . 5.6.5).

231
, , -
. -
, , ,
, , ,
(. . 2.6.2), . -
.
, m1 m2.

v 1 v 2, -

, u 1 u 2.

m1 v 1 + m2 v 2
2 2
1 1m v m v2 2
- + -------------- .
-------------
2 2
2 2
1 1 m u 2 2 m u
- + -------------- . -
m1 u 1 + m2 u 2 -------------
2 2
-
, :

m1 v 1 + m2 v 2 = m1 u 1 + m2 u 2; (5.31)
m1v12 + m2v22 = m1u12 + m2u22. (5.32)


, (
).
, , -
( , -
, -
, -
).
-
x- -

m1 m2.
. 5.13 5.13 ,
. -
, . -
, -
: x-
. , -
x-, ,
,
, .

232

x- v 1, v 2, u 1, u 2,
(5.31) (5.32) :
m1v1x + m2v2x = m1u1x + m2u2x;
(5.33)
m1v21x + m2v22x = m1u21x + m2u22x.

u1x u2x.
:
m1(u1x v1x) = m2(v2x u2x);
(5.34)
m1(u21x v21x) = m2(v22x u22x).

,
u1x + v1x = v2x + u2x. (5.35)
(5.35) m2 -
(5.34);
1 ( m m )v + 2m v
2 1x 2 2x
u1x = --------------------------------------------------------------
-. (5.36)
m1 + m 2
(5.35) m1
(5.34),
2 ( m m )v + 2m v
1 2x 1 1x
u2x = --------------------------------------------------------------
-. (5.37)
m1 + m 2
(5.36) (5.37) .
: (v2x = 0).
:
1m m 2 1 2m
u1x = --------------------- v1x ; u2x = --------------------- v1x. (5.38)
m1 + m2 m1 + m2
m1 > m2 -
( ), m1 < m2 -
. ,
.
: . -
:
u1x = v2x ; u2x = v1x. (5.39)
.
, ,
.
: , -
. :
u1x = 0; u2x = v1x. (5.40)

233
,
, .
, , -
* . ,
5.14.
1. , , -
2, 1 -
, 2 .
, -
. 5.14 , -
5.15. -
1. 2
, 10.
, 1 -
2,
3, -
. 5.15 4 .
10.


-
, .
, (5.38).
:
m1
------- 1
m2
u1x = ----------------
- v1x. (1*)
m1
------- + 1
m2
, (, -
) , ,
,
m
. -------1 0, (1*)
m2

u1x = v1x. (5.41)
. , -
, , -
, -
.
-
5.16,
. m
. 5.16
, v ,

234

u 1 u 2 . -
, :
2 2 2
mv mu 1 mu 2
----------- = ----------- + ----------- ;
2 2 2 (2*)

m v = m u 1 + m u 2.

, (2*)
:
2 2
v2 = u 1 + u 2 ;
(3*)
v = u 1 + u 2.
(3*);

v2 = u21 + 2 u 1 u 2 + u22.

(3*), , u 1 u 2 = 0 ,

, u 1 B u 2, .

5.4.4.

.
, , ,


p K.
:

p = F t;
(5.42)
K = F r .

F , (-
); t ,

r ,

F .

(5.42), p

F t t. ( -
;
!). ,
(. (2.7)). (5.42) ,
K -

F r , r .

235
(5.30) ,
.


v -
m :
2
mv
p = m v ; K = ----------
-. (5.43)
2
(5.43) , p K :
2
p
K = -------- . (5.44)
2m
, ,
. -
, -
,
.
, ,
,
.
(5.42), -

r
t:

K = F r = F v t = v p.
,

K = v ( mv ) . (5.45)

5.4.5. , ,

v, , , -
, m -
(5.43)(5.45) . -

XX
() .
(
relativus, ).
,
:
v n c, (5.46)
( = 3 108 /).

236
(5.46) , ,
, -
v (. .
, -
). -
:
m0
m = ------------------
-, (5.47)
2
1

= v
--- . m
c
, m0
. -
m0 ,
.
m(v) 5.17. -
(5.47), , -
, , v = 0,01 (-
v 3000 /, -
100
),
m = 1,00005 m0, . . -
. 5.17
-
0,005%. , ,
m = m0,
.
,
(5.43)(5.45)
:

p = m0 v ; K = 1--- m0v2; (5.43, )
2
2
p
K = ----------
-; (5.44, )
2m 0


K = v ( m 0 v ) . (5.45, a)

p = m0 v
:

0 m
v
p = m v = ------------------
- (5.48)
2
1

237

, , ( m 0 v ) (5.45, )
:

K = v ( m 0 v ) . (5.49)
, , -
( , ,
):
2
0m c
E = mc2 = ------------------
-. (5.50)
2
1
, -
:

, -
(. (5.50)):
E = mc2. (5.51)

, ; -
, . ,
, .
, , n 1,
(1 ) d 1 (5.52)
. = 2
= 1--- :
2
1
1 --- 1
------------------- = (1 2) 2 d 1 + --- 2
2 2
1
(5.50) , n 1 (v n c):
2
0 m v
E = mc2 d m0c2 + -------------
-. (5.53)
2
(5.46) , (5.53)
:
E = m0c2 + K, (5.54)
K .
(5.54) :

:
K = mc2 m0c2. (5.55)

238
m0c2 , (5.55) ,

K = (mc2) = c2m. (5.56)



F r , ,
. -

F r ,
( , -
2).
, (5.55),
2
v
(5.49). (5.47) m 1 -----
2
- = m0 -
c
;
(mc)2 (mv)2 = m20c2. (1*)
,
f
( (3.52)):
(f2) = (f + f)2 f2 d 2ff. (5.57)
(5.57), (1*) :
2mc(mc) 2mv(mv) = 0. (2*)
2m,

(mc2) = v(mv) = v ( mv ) . (3*)

(5.56) (3*), , K = v ( mv ) , . . -
(5.49).
E -

p . (5.48) (5.50) ,
2 2
p v
------2 = -----4- . (4*)
E c

2
v
(5.50) E 1 -----
2
- = m0c2 -
c
;
2 2 4
v
2 E m0 c
-----2- = --------------------------
2
. (5*)
c E

239
2 2 4
p
2 E m0 c
(5*) (4*), ------ = -------------------------- . ,
2 2 2
E E c
2 2 2 4
E = p c + m0 c . (5.58)
(5.46) (5.58), (5.52):
1
---
p 2
2
p
2
E = m0c2 1+ ------------- d m0c2 + ----------- . (5.59)
2 2
m0 c 2m 0

(5.59), , (5.54).

5.4.6.


.
m
. ,
. mi Ri -
i- .
vi = Ri,

Ki = 1--- miv2i = 1--- mi2R2i. (5.60)


2 2
K -
, . ,

K = 1--- 2 m R
2
2 i i . (5.61)
i
(4.26), (5.61)
J . ,
2
K = J
---------- . (5.62)
2
, -
K(t) t :
2
(t)
K(t) = J
----------------- , (5.63)
2
(t) .
(5.63) ,
J . (5.63)
J J(t).

240
5.4.7.

, -
. t = 0
0. (4.6)
0 t
(t) 0 = t. (5.64)
t

= 1--- ((t) + 0)t.


2
(5.63), -
t

K(t) K0 = 1--- J2(t) 1--- J20 = ((t) 0)((t) + 0) --J- . (5.65)


2 2 2
(5.64) (5.65) : K(t) K0 = J. -
-
(4.27),
z = K(t) K0. (5.66)
z
, . z -
, -
. (5.66)
(5.30).
. m ,
m. t = 0
m. -
t?
t = 0
. ,
, -
. (5.64)(5.66),
K(t) :

K(t) = 1--- zt2. (1*)


2

= ---a- , R , a -
R
; z = (T2 T1)R, T1 T2

241
. -
, (1*)

K(t) = 1--- (T2 T1)at2. (2*)


2

a (4.39), T1
T2 (4.40), m1 = m, m2 = 2m. -
: a = 2g 9mg 10mg
------- , T1 = ------------- , T2 = ---------------- , -
7 7 7
2 2
mg t
(2*) : K(t) = ----------------
-.
49

5.4.8. ,


t m R, -
.

K(t), v C(t), C(t), , -
,
,
, ( -
. 5.18 -
,
). -
t AA ,

(. 5.18). A(t) -
AA, JA -
AA, K(t) ,
(5.62), :

K(t) = 1--- JA2A(t). (1*)


2
wA(t) = w(t) ( (4.85)), (1*) -
:

K(t) = 1--- JA2C(t). (2*)


2

(. . 4.2.3),

JA = JC + mR2, (3*)

242
JC CC, -
. (3*) (2*),
K(t) :
K(t) = 1--- JC2C(t) + 1--- mR22C(t). (4*)
2 2
RA(t) = vC(t) A(t) = C(t), , ,
vC(t) = RC(t). (5*)
(5*) (4*),
2 2
Cmv ( t ) C C J (t)
- + ---------------------- .
K(t) = ------------------- (5.67)
2 2
, , -
, -
, ,
, -
. -
( ), , -
, .
(5.67)
* () . -
:

m
2 2
mv C ( t ) J C C ( t )
- + ---------------------- ,
K(t) = ------------------- (5.68)
2 2
vC(t) , C(t)
,
, JC(t) -
.

, , JC
(5.68) (5.67).

1. 2
. 4.1.3, m.
, -
V1 + V2
2 . 4.1.3, : vC = 1--- (V2 V1), C = -------------------- . -
2R
2
1 2
(5.67) , JC = --- mR , -
2
1
: K = --- m[(V2 V1)2 + 1--- (V1 + V2)2].
4 4

243
2. m -
. -
t
?
vC ( t ) 1
, C(t) = -------------
- JC = --- mR2, -
R 2
(5.67) :
2
vC ( t )
K(t) = 1--- mv2C(t) + JC ------------- 3 2
2
- = --- m v C (t). (6*)
2 2R 4


v (t)
C
aC = -------------
- . (6*) :
t

K(t) = 3--- ma C t .
2 2
(7*)
4
(4.90, ) (7*) ,
: K(t) = 1--- mg2t2sin2 .
3

5.5.

5.5.1.

.
.

, .
, . 5.4.5 (
,
).
, , -
,
, (, ,
).
, -
. -
, -

(, , , , ).

244
,
, -
.
,

.
, -

,

. AI(1 2),
AII(1 2), AIII(1 2)
1 2, . 5.19
-
I, II, III (. 5.19). ,
AI(1 2) = AII(1 2) = AIII(1 2).
1 2
. ( -
conservatio, ). -

, , ,

r ( r
, 1 -
2).



, -
.

. -

(. 5.20, ). A
. -
1 2. - )
AI(1 2) 1 2 I,
AII(2 1) 2 1 II
(. 5.20, ). ,
A = AI(1 2) + AII(2 1). (1*)
AII(1 2) 1 2 )
II (. ). . 5.20

245
, AI(1 2) = AII(1 2) (1*)
:
A = AII(1 2) + AII(2 1). (2*)
, AII(2 1) = AII(1 2), (2*): A = 0, -
.

, -

-
.
, -
: 1) , ,
,
; 2) ,
.

5.5.2.

-
, .

, -
, . ,
-
( -
), , -
, -
, .
, -
( ,
, ). -
, , , -
. , -

, -
.
,
A(1 2) 1 2
U ,
U2 U1 , , U(1 2).
,
U(1 2) = A(1 2). (5.69)

246
(A(1 2) > 0), -
(U(1 2) < 0). -
(A(1 2) < 0), -
(U(1 2) > 0).
, -
? -
?

, .
, - , -
, , , .

, .
, -
.

(5.69) :
A(1 2) = U(12) (5.70)
-
:

, , -
, -
.

, , , -
(). 1
( U1) 2 (
U2).
A(1 2).
, -
1 2 ,
-
O, -

(. 5.21):
A(1 2) = A(1 0) + A(0 2).
(1*)
(1*) , , -

A(1 2) = A(1 0) A(2 0).
(2*) . 5.21

247
, A(1 0)
, , A(1 0) = U1.
, A(2 0) = U2. (2*)
:
A(1 2) = U1 U2 = (U2 U1) = U(1 2).
, (5.70), -
.

5.5.3.
,

, m, -
1 h1 -
, 2 h2
(. 5.22). , -
,
.
, -

m g r 1 2.


. 5.22 A(1 2) = m g r =
= mgrcos = mgrsin = mg(h2 h1).
A(1 2) (5.69),
U(1 2) , -
h1 h2:
U(1 2) = A(1 2) = mg(h2 h1). (5.71)

1 (. 5.23). l, -
m,
.
,
?
, A(1 2)

r (
1 2)

A(1 2) = m g r = mgrcos =
. 5.23 = mgh = mgl(1 cos).

248
(5.69),
U(1 2) = A(1 2) =
= mgl(1 cos). (5.72)
2 (. 5.24). , -
,
m M (M > m).
. -

t ,
?
. 5.24
t -
s = 1--- at2, a . (2.33),
2
Mm
a = g ----------------- ;
M+m
m
s = 1--- gt2 M
----------------- . (1*)
2 M+m
Um(1 2) UM(3 4) -
m M, ,

Um(1 2) = Am(1 2) = m g r 1 = mgs; (2*)

UM(3 4) = AM(3 4) = M g r 2 = Mgs. (3*)
U
U = Um(1 2) + UM(3 4) = (M m)gs. (4*)
(1*) (2*)(4*),
Um(1 2) = 1--- mg2t2, UM(3 4) = 1--- Mg2t2,
2 2

U = 1--- (M m)g2t2,
2
m
= M
----------------- .
M+m

5.5.4.

,
. ,
, .

249
, -
-


F , -

r
(. 5.25).
-

r , -
(5.17),

A = F r = 1--- k(r)2, (5.73)
2

. 5.25 k .
-
: 1) x1 = r1, 2) -
x2 = r2.
x : F = kx.
U(1 2) -
1 2. -
1
r1 = x1, , -
(5.73), A1 = kx2. r2 = x2
2
kx
A2 = ---------2 .
2
1 2
2 2
kx kx
A(1 2) = A2 A1 = ( ---------2 ) ( ---------1 ) = k--- (x22 x21).
2 2 2
(5.69), :
U(1 2) = A(1 2) = k--- ( x 2 x 1 ).
2 2
(5.74)
2

5.5.5.

, -
,
.
, ,
, . .
. -
m h1 -
U1 = mgh1, h2 U2 = mgh2. h2 > h1,

250
U2 > U1 U(1 2)
h1 h2
U(1 2) = U2 U1 = mgh2 mgh1 = mg(h2 h1). (1*)
, 5.22, , -
1. -
, U1 = 0 U2 = mg(h2 h1). -
U(1 2) :
U(1 2) = U2 U1 = mg(h2 h1) 0 = mg(h2 h1). (2*)
,
2. U1 = mg(h2 h1) (-
1 !)
U2 = 0. U(1 2) -
:
U(1 2) = U2 U1 =
= 0 [mg(h2 h1)] = mg(h2 h1). (3*)
, (1*)(3*)
(5.71), -
.
1 . 5.5.3
? -
, -
. ,
- ; H.
1 H lcos , ,
U1 = mg(H lcos). 2 H l ,
, U2 = mg(H l). U(1 2) -

U(1 2) = U2 U1 = mg(H l) mg(H lcos) =
= mgl(1 cos). (4*)
, -
.
U1 = mglcos U2 = mgl. U(1 2)

U(1 2) = U2 U1 = mgl (mglcos) =
= mgl(1 cos). (5*)
, (4*) (5*) (5.72) -
, .

, , ,

251
. ,
U(1 2) . 5.5.4:

U(1 2) = U2 U1 = k--- x 2 k--- x 1 .


2 2
l l
-
1 ( x1),
, U1 = 0, U2 -
5.25 ( -
):
kx + kx
(x2 x1) = k--- ( x2 x 1 ).
1 2 2 2
U2 = -------------------------
2 2

U(1 2) = U2 U1 = k--- ( x2 x 1 ) 0 = k--- ( x2 x 1 ),


2 2 2 2
2 2
(5.74). , -
, -
, -
.
, -
, ,

.
. ,
, -
.

5.5.6.

-
. , -
, -
( ).
, -
.
, ,
, , .
( -
) -
, ,

, , ,

252
. , -
, , (-
) .
:
? ,
. 5.5.3 ,
, . 5.5.4 -
?
(5.71), (5.72), (5.74) -
, .
? , , , -
(5.71), ,
, -
, 5.22, .
:

U = (m g r 1 + F r 2), (5.75)

m g r 1 , -

, F r 2 ,
(,

F = m g). ,
r2 n r1, (5.75) -
.


, .

.

5.6.

5.6.1.

,
, ( , ).
W -
K U:
W = K + U. (5.76)

253
(5.76) .
, -
.
, N , -
:
N
K = K . i (5.77)
i=1
Ki i- . -
N ,
, -
(
):
N N
U = 1---
2 U ij . (5.78)
i j
Uij i- j-
. (5.78) Uii
Ujj, .
1--- , Uij Uji .
2
, N = 3. :
K = K1 + K2 + K3; (5.77, )
3
U = 1---
2 (U i1 + Ui2 + Ui3) U11 U22 U33 =
i=1
= U12 + U13 + U23. (5.78, )
U12 1- 2- ,
U13 1- 3- , U23 2- 3- .
N = 2,
K = K1 + K2; (5.77, )
U = U12. (5.78, )
, m,
h

v ,
.
1--- mv2, .
2

254
, , -
, . 5.5.6,
. ,
, ,
. -
(5.77, ) (5.78, ) : K = 1--- mv2
2
U = mgh. , W
:
2
mv
W = ----------
- + mgh. (5.79)
2

5.6.2.

XIX -
, (-
) -
( ),
( ). , -
-
, . , -
-
.
. -
:

-
, ,
.

, -
, -
.

-

, . F 21 F 12 -
, , , 1 2

. r 1 r 2
t. K -
t , (5.77, ),

255
K = K1 + K2, , (5.30) (5.4),
:

K = F 21 r 1 + F 12 r 2. (1*)
U t ,
(5.78, ) (5.69),

U = U12 = ( F 21 r 1 + F 12 r 2). (2*)
( (5.75)). W -
, (5.76), W = K + U. -
(1*) (2*), :
W = 0 , , W = const. (5.80)
(5.79),
:
2
mv
----------- + mgh = const. (5.81)
2

* , , -
. -
N .
5.26

. F ij
, i- j-

(i, j = 1, 2, 3). r 1, r 2, r 3

t. (5.77, ), K = K1 +
+ K2 + K3. -
:

K1 = (F 21 + F 31) r 1; K2 =
. 5.26
= (F 12 + F 32) r 2; K3 = (F 13 + F 23) r 3.
,

K = (F 21 + F 31) r 1 + (F 12 + F 32) r 2 + (F 13 + F 23) r 3. (3*)
(5.78, ), U = U12 + U13 + U23. :

U12 = (F 21 r 1 + F 12 r 2); U13 = (F 13 r 3 + F 31 r 1);

U23 = (F 32 r 2 + F 23 r 3).
,

U = (F 21 r 1 + F 12 r 2) (F 13 r 3 + F 31 r 1)

(F 32 r 2 + F 23 r 3). (4*)

256
(3*) (4*),

W = K + U = (F 21 r 1 + F 31 r 1) + (F 12 r 2 + F 32 r 2) +

+ (F 13 r 3 + F 32 r 2) (F 21 r 1 + F 12 r 2)

(F 13 r 3 + F 31 r 1) (F 32 r 2 + F 23 r 3). (5*)

, F r , (5*),
, : W = 0.

5.6.3. ,


m.

1 (. 5.27). m
H -

v 0. -
, -
.
t
: . 5.27
W = K(t) + U(t) = 1--- mv2(t) + mgh(t), (1*)
2
v(t) h(t) , -
t.

A. , v (t) = v 0 + v 1(t) (. )
, ,
2 2
v2(t) = v 0 + v 1 (t). (2*)
, -

v1(t) = gt; h(t) = H 1--- gt2. (3*)


2
(2*) (3*) (1*),
2
W = 1--- m( v 0 + g2t2) + mg(H gt
--------- ) = --- m v 0 + mgH = const.
1
2 2
2 2 2

, W t, .

257
2 (. 5.28). m
-
-
H. , -

.

t
. 5.28
(1*).
gsin;
v(t) = gtsin

h(t) = H s(t)sin = H 1--- gt2sin.
2
v(t) h(t) (1*),
W = 1--- mg2t2sin2 + mg(H 1--- gt2sin2 ) = mgH = const.
2 2
, W t, .

3 (. 5.29). m

H (-

). . -
,
.

. 5.29 t
K(t) = 3--- m v C (t)
2
(4*)
4
(. (5.67) (6*) . 5.4.8). vC (t)
. , -

W = K(t) + U(t) = 3--- m v C (t) + mghC(t),
2
(5*)
4
hC (t) t. aC
, vC(t) = aCt. -
,
hC (t) = H s(t)sin = H 1--- aCt2sin.
2

258
(5*)

W = 3--- m a C t2 + mg(H 1--- aCt2sin).


2
(6*)
4 2

(4.90, ), aC = 2--- gsin. aC (6*),


3

W = 1--- mg2t2sin2 + mg(H 1--- gt2sin2 ) = mgH = const.


3 3
, W t, .
: -
! ,
.
( . 5.2.7),
.

5.6.4.

, m -

, F
(F > mg) (. 5.30). W -

r (r = s). -
K

r , . -

m g F , , -
,

K = (m g + F ) r = mgs + Fs.
U
. 5.30

r :

U = m g r = mgs.
,

W = K + U = F r = Fs. (1*)
, . -
M , N -
, mgs, , -
(F mg)s.
-

: F -

259
) )

. 5.31

.
.
2 . 5.5.3,
, , -
m M (M > m). m

m g T 1, M

M g T 2 (. 5.31, ). T 1 T 2

; T. T 1 -

m , F , -
5.30. m -
. Wm -
, (1*),

Wm = T 1 r 1 = Ts. (2*)

M. WM


WM = T 2 r 2 = Ts. (3*)
, (, -
m M), .
-
, . . W = 0, W
-
.

260
, W = 0 . -
* W -
:
W = K1 + K2 + K3 + U12 + U13 + U23. (4*)
1 m, 2 M, 3
. , , K3 = 0. -
,
U12 = 0, (5*)
(4*) : W = K1 + K2 + U13 + U23.
, K1 + U13 Wm (2*), K2 + U23 WM (3*).
,
W = Wm + WM. (6*)
(2*) (3*) (6*), W = 0, .

(5*). 5.31, :
, 5.31, .

, T 1 T 2,

U12 = (T 1 r 1 + T 2 r 2) = (Ts Ts) = 0.

5.6.5.

m , -
1--- mv20. -
2

. -
mgH, H (-
).
2 2
v v
(1.37), H = ------0- , , mgH = mg ------0- = 1--- mv20.
2g 2g 2
,

. ,
. -

.
-
1--- mv20 = mgH. -
2
-

261
.
,


-
.
-
v(t) = v0 gt.

v 2
K(t) = 1--- mv2(t) = 1--- mg2 t -----0- . (5.82)
2 2 g
,
(t = 0)
v
t -----0- 5.32. t
g
v
= -----0- , ,
g
. -
.
:
v 2
U(t) = W K(t) = 1--- mv20 1--- mg2 t -----0- . (5.83)
2 2 g
( W .)

. 5.32

1. t = 0

v 0. -
-
?

262
K(t) = U(t), , (5.82) (5.83),
:
v v 2
(t -----0- )2g2 = v20 t -----0- g2.
g g

v v
, t -----0- = ----------
0
- ; -
g 2g
v v
: t1 = 1 ------- -----0- ; t2 = 1 + ------- -----0- .
1 1
2 g 2 g

l
m
.

,
l, -
A B . 5.33
(. 5.33). -
( ).
K() U()

. A B
--- --- . :
2 2

A ( = --- )
2

K( --- ) = 0, U( --- ) = mgl;


2 2
C ( = 0)
K(O) = mgl, U(O) = 0;

B ( = --- )
2

K( --- ) = 0, U( --- ) = mgl.


2 2
A C -
,
C B .
B, -
: B C C A.
,
. ,

263
, (
),
. , -
-
. , -
( , A ) -
.
,
K() + U() = const = mgl

--- --- .
2 2
,
U() = mgl(1 cos), (5.84)
, ,
K() = mgl U() = mgl cos . (5.85)
5.34 K() (
) U() ( ). K() =
U(), 1 2, -
. cos = 1--- ;
2
, 1 = --- , 2 = --- .
3 3

2 (. 5.35). m, -
l, A. C -
, . -

C ?
A 2l.

v 0 (-
C).

. 5.34 . 5.35

264
, C. 1--- mv20 = mgl, -
2
. ,
v0 = 2gl . (1*)

v0 ; ,
l -
2l
v0. (1.41), s = v0 ----- . -
g
(1*), : s = 2l.
A mgl, D mgl + K, K
. -
, , mgl = mgl + K. -
: K = 2mgl.

-
. -

, -
. ( , -
) ,
( -
) . -
,
, -
, -
.
-
, -
. ,

, . 5.4.3.

3. M v 0 -
m (m < M). -
. -
. -
.
-
1--- (M + m)v2 + U, v -
2
, U -
.

265
1--- Mv20 .
2
,
1 1
--- Mv20 = --- (M + m)v2 + U. (2*)
2 2
, ,
Mv0 = (M + m)v. (3*)
2
1 Mmv
(2*) (3*), : U = --- -----------------0 .
2M + m

5.6.6.


,
.
1 (. 5.36). m


v 0 h
-
k .

?
x
-
-
MN (. ).
. 5.36 KA, UA, WA
, -
A, KB, UB, WB -
B. KA = 1--- mv20 UA = mgh,
2

WA = 1--- mv20 + mgh. (1*)


2

KB = 0 UB = mgx + 1--- kx2,


2

WB = 1--- kx2 mgx. (2*)


2

266
, WB = WA.
(1*) (2*), :
1 1 2
--- kx2 mgx ( --- m v 0 + mgh) = 0.
2 2

ax2 + bx + c = 0, a = 1--- k,
2
(
b = mg, c = 1--- mv20 + mgh . :
2
)
2
b b 4ac
x = ------------------------------------
- = --- (mg
1
m g + kmv 0 + 2kmgh ).
2 2 2
2a k
, . ,

x = --1- (mg + m g + kmv 0 + 2kmgh ).


2 2 2
k

2 (. 5.37). ,
l,
A. B 3--- l
4
.

-
( D )?
. 5.37

C. A
WA = mgl, D WD = 1--- mv2 + 1--- mgl, m
2 2
, v . -
, WA = WD, : mgl =
= 1--- mv2 + 1--- mgl. , v = gl .
2 2

3 (. 5.38). -
R -
.
?
A -
, R.
-
OA (. ), -
h , A.
,
h = R + Rcos = R(1 + cos). (3*) . 5.38

267
,
m - A; -
v .

m g N . -

m g + N -
A; : mg cos N.
,
2
mv
----------- = mg cos N. (4*)
R
2
v
------ -
R
A. A , N = 0 (4*)

2
mv
----------- = mg cos. (5*)
R
(3*), (5*)
v2 = g(h R). (6*)
, -
WC C (WC = 2mgR) -
WA A (WA = 1--- mv2 + mgh):
2
2mgR = 1--- mv2 + mgh. (7*)
2
5
(6*) (7*) h = --- R.
3
4 (. 5.39).

,

R?
v

( A). -
. 5.39 m

m g N . .
,
2
mv
----------- = mg + N, (8*)
R
2
v
-----
- A,
R
. , -

268
( ) ,
A , -
N l 0. , v A
:
2
mv
----------- = mg. (9*)
R
h , -
, WC = WA. -
, WC = mgh WA = 1--- mv2 + 2mgR, :
2

mgh = 1--- mv2 + 2mgR. (10*)


2
v2 (9*) (10*),
h = 5--- R.
2

5.7.
?

5.7.1.
( )

, -
, -
-
. (
) . 5.6 -
, , -
, .

m -

. ,
,

. -

m g, N (N =

= mg cos) F
(F = N = mg cos). -
5.40; - . 5.40

269

r B C ( -
). -

r

A = m g r + F r + N r = A + A,
A = mgr sin , A = mgr cos
;
.
, A < 0 A < 0.
! A (-
) -
( ).
, -

F r .
W -
B C W = K + U, K U
, .
(5.30), K = A = A + A. (5.69), U = A. -
,
W = (A + A) A = A = mgr cos . (1*)
, -,
, , -, -
.
:
! ,
,
-
. -
.
:
(-
) :
W = A, (5.86)
W , A
( ); A < 0. , -
, -
, . ,
, -
, . . -
.

270
, -

. , B v 0
(), C . r -
, . . ,
( , ,
mg sin > mg cos ). (5.86) ,
, (1*), :
KC KB + U = mgr cos. (2*)

KB = 1--- m v 0 , KC = 0, U = mgr sin, (2*)


2
2
1 2
--- m v 0 + mgr sin = mgr vcos, ,
2
2
v
r = -------------------------------------------------
0
-.
2g ( sin + cos )

(. 5.41). h1 -
-
1 -
,
-
2. -
-
.
. 5.41
?
B , -
, D , C
(. ). WB
WD B D, -
, h2 D, m .
(5.86), :
W = WD WB = mg(h2 h1); (3*)
h h
1
A = mg cos1 --------------- mg cos2 ---------------
2
. (4*)
sin 1 sin 2
(3*) (4*) (5.86),
mg (h2 h1) = mg (h1ctg1 + h2ctg2),
,
1 ctg
h2 = h1 -----------------------------1- .
1 + ctg 2

271
, , -
, (h1 = h2), . .
.

5.7.2.

, -
. , - -
? , ,
, ,
, . ,
.
.
-
. ,
-
( -
). ,
, .
.
,
, . . --
.
. ,
, -,
( ), -,
. , -
( K; -
)
( U). E
:
E = K + U. (5.87)
-
. , ,
, . . . -
, K,
, E.
XVII ( ,
! , ) ,
-
. , 1685 -

272
:
. -
, ,
, (
) ,
. . -


. , -
1798 -
. -
, -
, .
,
. -
.
-
. ,
, , . . -
K. -
U -
. , , , -
,
-
, .
, K -
, U
.
. : , -
, -
. ,
, -
, -
.

5.7.3.

h -
, ,
h, h. -
-

273
,
.
, -, -
, -, , ,
, , -
. , -

,
; h < h.
-
? ?
-
( ),

( ).
-
E W = mgh mgh. , mgh mgh = E.
:
W0 = W + E, (5.88)
W0 W -
, E ,
.
: 1) -
! ; (5.88) ,
, , ; 2) E -
-
.
, -
, -
,
. , -
,

(. . 5.4.3).

. -
, -
, (

).

274
1.
R m.
4--- R . ,
3
-
.

W0 = 2mgR. -
4--- mgR,
3
v2 = g(h R) (. 3 . 5.6.6),
h = 4--- R. ,
3

W = 4--- mgR + 1--- mg( 4--- R R) = 3--- mgR.


3 2 3 2
, (5.88),

E = W0 W = 1--- mgR.
2

2. 4 . 5.6.6 -
5--- R , -
2
.
h = 7--- R.
2
,
m
?

W0 = 7--- mgR. -
2
2mgR (R ),
2
mv 1 1
- = mg; --- mv2 = --- mgR.
----------
R 2 2
, -

W = 2mgR + 1--- mgR = 5--- mgR.


2 2

, (5.88), E = mgR.

275
3 (. 5.42).
M , -
.

l.
m
. -

. -

. 5.42 ,

.
-
( . 2.6.3). v
,
:
v m
0
v = ----------------
-, (1*)
M+m

v0
.
. -
1--- (m + M)v2.
2
(m + M)gh,
h = l(1 cos) ,
(
); ,
1
--- (m + M)v2 = (m + M)gh
2

, ,
v2 = 2gh = 2gl(1 cos). (2*)
(1*) (2*),
M+m M+m
v0 = ----------------- 2gl ( 1 cos ) = 2 ----------------- gl sin
--- . (3*)
m m 2

E, -
. -
-

276
W0 = 1--- mv20, W = 1--- (M + m)v2.
2 2
(5.88),

E = W0 W = 1--- mv20 1--- (M + m)v2,


2 2
, (1*),
2 mM
E = 1--- v 0 ----------------- . (4*)
2 M+m
(3*) (4*),
(M + m )
E = 2M
------------------------------- gl sin2 --- .
m 2
, m n M,
:
2
v0 = 2 ----- gl sin
M
--- ; E = 2 M
-------- gl sin2 --- .
m 2 m 2

5.7.4.

. 5.4.5 (5.50), -
, -
: E = mc2. ,
K E
m0c2 , (. (5.54)):
E = m0c2 + K.
, ,
(5.54) -
U:
E = m0c2 + K + U. (5.89)
, N ,
i- m0i. , ,
N
m0 = m 0i . . m0c2 -
i=1
K -
U:
N

m0c2 =
m 0i c2 + K + U.

(5.90)
i=1

277
, ,
.
N
m0 = m 0i
1
+ ----
- (K + U).
2
c
(5.91)
i=1
,

E =
m
i
0i c
2
+ K + U + K + U.

(5.92)

E = K + U.
,
. -
-
:


E = 2
m 0i c + K + U. (5.93)

i

5.7.5.

, -
, . . , -
. -
-
, .
. -
:
.
, ,

. -
: -
.
.
.
, -
, -
.
( , , , -
).

278

5.1. - 5.8. m = 3 -

- c h = 4 .
, m -
s. v = 5 /. -
: ) , )
, ) ? .
5.2. - 5.9.
- -
- ?
? 5.10. 5.43 -
5.3. -
: ) , - h.
, ) -
, ) 2 ,
, ) 1? -
.
, )
?
5.4. -
,
. -
, -
. 5.43
-
. 5.11. m = 500 -
5.5. ,

v 0 .
. ,
-
h? , v = 0,8 /.
5.6. , - -
?
v. 5.12. 1 12
2, -
v 2 . -
2v? : ) -
5.7. m 1 2, ) -
1
h . - 2.
5.13.
?

279
320 /, -
10 ? = 0,15. -
5.14. l - ,
m s = 600 ?
. ,
60 .
Fsin.
, - 5.19. -
- v0 -
, h,
-
30? - 1,5 h?
.
? 5.20.
5.15. : -
- .
, -

, -
.
. . ,
5.16. - , -
h = 2 - , -
= 45 ,
.
- 5.21.
. - m = 60
h = 4 . -
= 0,2. -
- ,
- ,
? -
5.17. , s = 1 ?
, - 5.22. -
F = 20 -
, = 30,
= 30 -
. -
, . , -
s = 600 ?
5.18. , -
m = 10 - . -
, - -
, , ?
= 30 . -
. -

280
- .
-
. 5.4.8.
? -
- 5.27. m,

. v 0, -
5.23. N = m. -
= 20 , -
m = 240 - . -
-
= 15 .
- -
-
= 0,2.
? ?
5.24. m = 30 - 5.28. m -
l = 3 . l.
, -
,
-
- ,
? - --l- . -
, 2
, T = 500 .
5.25. ?
m - .
, -
l/2 ,
-
, .
- 5.29.
.
v0 = 50 /, -

,
-

-
, .
-
-
?
. , -
,
- n = 49 ,
2mgl , -
(l ). = 0,25?
5.26. m 5.30. M
- . -
- l.
. - m, -
- ,
t ? ,

281
. - .

, .
- , -
?
5.31. m = 100 - ?
h = 10 . - 5.33.
h = 2 - l m -
v = 10,5 /. -
, . -
-
= 60. -

h? .
5.32. m, - , -
l, - -
,
?

282
6

6.1.

-

XVII ,
, -
. -
,
.

6.1.1.
( )

, -
.

,
. : -
, , .
-
,

, , -
. 6.1
F1 F2 (
2c),
AC ( 2a)
BD ( 2b). AC
BD - . 6.1

283
. P. -
, F1P + F2P ; -
, F1P + F2P = 2a.

= --c-
2
; 0 m < 1. ,

. = 0 (
).
, 6.1 -
F1. A, -
,
, C, , .
-
AF1 = a c = a(1 )
CF1 = a + c = a(1 + ), a -
(. .
) (-
: ..); 1 .. = 150
.
:

a, .. a(1 + ), .. a(1 ), ..
0,39 0,21 0,31 0,47
0,72 0,007 0,715 0,725
1 0,017 0,983 1,017
1,52 0,093 1,38 1,66
5,20 0,048 4,95 5,45
9,54 0,056 9,01 10,01
19,18 0,047 18,28 20,08
30,06 0,009 29,79 30,33
39,26 0,245 29,64 48,88

, -
< 0,1; ,
, a.
;
, . ,
= 0,97. , ,
(, = 0,055). -
( = 0,76).

284
6.1.2.
( )

-
.

6.2,
, -
-
M1 N1 M2 N2. )
-
, , -
,
.
-
: s1 < s2. ,
- )
.
, -
.
v
------ , v v )
v
-
. - . 6.2
.
t t
, . , -
- t t + t
t t + t 6.2, . t -
, 1--- vt a(1 ),
2
1
--- vt a(1 + ). ,
2
; , v(1 ) = = v(1 + ).
,
v 1+
------ = ------------ . (6.1)
v 1

v
= 0,017. , ------ = 1,035;
v
3,5% .

285
,
.
( ) -
* -
. ,
, z-,
.

t L -
(. (4.44)):

L = m(
r (t)
v (t)),

m , r (t)
v (t) - , -
. z-
Lz = L = mr(t)v(t) sin(t),

(t)
r (t)
v (t).
,
Lz = L = mr(t)v(t) sin(t) = const. (6.2)
t t + t -
M N (. 6.2, ). -
1
S = --- r(t)v(t)t sin (t), . . (6.2)
2
1
S = -------- Lt. (1*)
2m
t2 t1.
ti . Si i- -
(1*). S, - -

t2 t1, S =
S = --------
i
2m
i
L
2m
L
t = -------- (t
i
i 2 t1). L = const,

t2 t1 -
, .

6.1.3.
(
)

-
.

286
T1 a1 -
, T2 a2
,
:
2 3
T1 a1
-----2- = -----3- . (6.3)
T2 a2
:
, a -
r :
C
a = ----
-.
2
(6.4)
r
C , -
.
(6.3)
2 3
T r 2r
: -----12- = ----13- . , T1 = ------------1
T2 r2 v1

2r
T2 = -----------2- , v1 v2 , -
v2
:
2 2 3
r1 v2 r1
----2- -----2- = ----3- . (1*)
r2 v1 r2
2
a = v
------ ,
r
(1*) :
2
a 2 r1
---------- = ----2- . (2*)
a 1 r2
(2*) (6.4).

6.2.

6.2.1. ,


, -

287
. 6.3
; A, B, C, D -
-
-
.
, -
. -
,
. 6.3
, -
, , ,
.
-
: , ,
, ,

(, ,
).
:
-
.

, , .
: -
, , -
, , , , , -
.
, ,
, . -
-
. ,
, , -
;
. ,
,
, .
-
. ( )
.

, , -
.
.
, , ,

288
, . -
, -
. -
,
( ) , ( -
) , -
.
.
, -

(gravitas ).

6.2.2.

, -
? m1 m2,
r ?
-
.
, ,
, ,
, -
, ,
1
r , ----
-.
2
r

F 12 , m1 -

m2, F 21 , m2 -
m1. -
m m
, F12 f -------
2
2
F21 f -------
2
1
.
r r
m m
, F12 = F21. , F12 = F21 f ---------------
1 2
2
-.
r

:

,
, m1 m2 -
r :
m m
1 2
F = G ---------------
2
-. (6.5)
r

289
G -
. -
.

,
- -
-

r 1 r 2 (. 6.4):
m m
1 2
F 12 = G ---------------
3
- ( r 1 r 2), (6.6)
r
. 6.4
r = |
r 1 r 2|, F 12 , -
m1 m2.
,
(6.5) (6.6),
( ).
() ?
?




. -
,
. 6.5

,
, (. 6.5).
-
, , -
.
.
, -
(6.5)
( (6.6)). 1: (
,
). 2:
, ,
.
r (6.5) ,

() . , -

.

290
(6.5) ( -
(6.6)), -
, , . -
, -
, ,
,
.
, -
:

F = G Mm
----------
2
-, (6.7)
r
M , m , r
, . . . F
, c , , -
,

ma = G Mm
----------
2
-, (1*)
r
a
. (1*) ,

a = GM
----------
2
-. (6.8)
r
(6.8) (6.4), , C = GM.
F (6.7) , , ,
,
Mm
Ma = G ----------
2
-, (2*)
r
a
. (2*) ,

a = G m
----2- . (6.9)
r
(6.8) (6.9),
a m
---------- = ----- n 1. (6.10)
a M

, , -
! , ,
. ,
, -
.

291

,
,
, , -
.

6.2.3.

G .
F, m2 -
m1,
r. -
, (6.5), :
2
Fr
G = ---------------
-. (6.11)
m1 m2


1798 . -
, -
6.6.
1, -
2, -
3. 4,
. 6.6
,
5.
.
m1.
-
m2(m2 . m1). , -
-
. -
F

, . -
, -
m1 = 775 m2 = 50 , 2 .
1%
G, :
G = 6,67 1011 H 2/2.
,
-
, . -

292
-
. -
, , -
. , , -
2 , 10
, 2,7 1012 H, -
2 1020 .

.
, m, ,
mm
G ------------ , m R ,
2
R
2
m m gR
. , mg = G ------------ , , G = ----------- .
2 m
R

4 2
m = --- R , , -
3
G:
3g
G = ------------------- .
4R

-
: = 5 103 /3 ( -
, = 5,52 103 /3).
6 106 ( , 6,37 106 ). -
3

, G = 7,8 1011 ---------------- . , 1 = 1 /2, -
2

: G = 7,81011 H 2/2.
-
.

6.2.4.

,
, -
(. . 2.3.1). -
(2.14):

F = m g,

m , g .

293

F , , -
,
m, --
r (. 6.7). (-
). --

r , (6.6)
. 6.7
:

F = G mM
----------
3
- r (6.12)
r
( M ).
,

F F:

m g = G mM
----------
3
- r (6.13)
r

g


g = G Mr
--------
3
. (6.14)
r
- r , . .
.

g = GM
----------
2
GM
- = ------------------------2 , (6.15)
r ( R + h )
R , h
( , ). -
- (1 + ------- ) , , h n R,
h 2
(6.15) g = GM
----------
2
R R
(5.52); :
g = GM - (1 2 ------- ) = g0(1 2 ------- ) =
h h
----------
2
R R R

= 9,8(1 2 -------
h
) /2. (6.15, )
R

, R = 6370 , M = 6 1024 -
g0 :
GM
2
- = 9,8 /2.
g0 = ---------- (6.16)
R

294
h
, h = 15 , 2 ------- = 0,005; ,
R
15 g 0,5%.
15
15
g , ------------ = 0,15. -
6370
, ,
- ,

, g, ,

m g , -
, (
).
, -
-
= 7,3 105 /
(
23 56 ). -
(6.13) (6.14) -
-

m g
) )

. 6.8
F , -
. , - r -
m
( ).
-
r cos (. 6.8, ).
, ,
, , ,
F, F, -

F = m2rcos. (6.17)
, , , -

, . . m g, -
F F (. 6.8, ):

m g = F + F . (6.18)
,
( ) -
(

F ). g

295
; -

, g, -
( -
).
, -
. , = 0 (

= 90), F = 0, ( = 0),
F F .
, -
* , -
, .
, 6.8, :

sin F
------------------------------------------ = --------- . (1*)
sin ( ( + ) ) F

mM
, sin( ( + )) = sin( + ), F = m2r cos, F = G ----------- ,
2
r
(1*)
2 3
sin r
---------------------------- = ------------ cos. (2*)
sin ( + ) GM
r = R = 6370 , = 7,3105 /, M = 61024 .
2 3
r
, ------------ = 0,0034. ,
GM
sin = 0,0034sin( + ) cos = 0,0034(sin cos + cos sin) cos . (3*)
, sin m 0,0034.
cos d 1 sin d ( ). -
(3*) :
d 0,0034( cos + sin) cos ;
((1 0,0034 cos2 ) d 0,0034 sin cos;
d 0,0034 sin cos ;
d 0,0017 sin2. (4*)
, = 0 = 0 = 90. -

g F , -
(4*), = 45. 0,0017 = 6 ( 6
!).


. g , -
(6.17),
g = 2 R = 0,034 /2. (6.19)

296
, g = 9,83 /2, -
g = 9,78 /2. g g = 0,05 /2, -
g, (6.19).
g -
, -
. 6357 , -
6378 .
, -
,

, g
, -
(6.15). ,
R = 6370
g0 = 9,8 /2.

6.2.5.

, M
R -
, m.
r . -
, r F,
, -
.
, F = 0. F(r)
0 < r m R, -
. ,
, ,
.

6.9,
; m -
r
.
m1 m2. ,

m, F 1

F 2.

2
F1 m1 r2
------ = -----------
-.
2
(1*)
F2 m2 r1 . 6.9

297
, -
r
----1 ,
r2
2
r
----1 . (
2
r2
2
r
r); , , ----1 .
2
r2
2
m r F
(1*) ------1- = ----1 , -----1- = 1. , ,
m2 2 F2
r2
m , -
. -
, , , , -
.

, , r
, , -
3
r Mr
----------
3
- ,
R
. -
, r m R,
3
mMr
----------------
3
-
R mMr
F(r) = G ----------------
2
- = G -------------
3
-. (2*)
r R

M
M =
R, , = G ------
-
2
g0, (2*) :
R

mg r0
F(r) = --------------
-. (6.20)
R

r l R
2
R
F(r) = G mM
----------
2 2

- = mg0 ------- . (6.21)
r r
-

. 6.10 6.10.

298
6.2.6.


m1 m2, (6.5), -

m m
1 2
U = G ---------------
-. (6.22)
r
,

(U = 0 r ).
(6.22),
,
(6.5) -
. -
m1 m2
. 6.11
r . -

r m2 -
1 2,
r + r, r n r (. 6.11).

A(1 2) F r ,
, (5.4), A (1 2) =

= F
r = F r, ,
, (5.70),
A(1 2) = (U2 U1) = U(1 2),
U(1 2)
. ,
Fr = U(1 2). (1*)
, F (6.5), U (1 2)
(6.22),
m m m m
- G ---------------- .
U(1 2) = U2 U1 = G ---------------
1 2 1 2
(2*)
r + r r

m1 m 2
- 1 ---------------- .
1
(2*) , U(1 2) = G ---------------
r r
1 + ------
r
r n r, (5.52),
1 r
---------------- = 1 ------ , (2*) U(1 2) =
r r
1 + ------
r

299
m m
- r. (1*), ,
1 2
= G ---------------
2
r
F (6.5).
m1 = M , m2 = m
, r
, , R.
(6.22)
U = G Mm
----------- , (6.23)
r

m, r .
, ,
. h
, (6.23)
Mm
U = G ---------------- . (6.24)
R3 + h
, m h1
h2 (h2 > h1). ,

U = mg0h2 mg0h1. (3*)
(6.24), :

Mm Mm Mm 1 1
U = G -------------------
- + G -------------------- = G ----------- ----------------- ----------------- .
R + h2 R + h1 R h1 h
1 + ------- 1 + ------2-
R R
h1 n R h2 n R, , (5.52), -

h h
----------- [1 ------- (1 ------- )] = G ----------
U = G Mm 1 2 Mm
2
- (h2 h1).
R R R R

M
- = g0; , U = mg0(h2 h1).
, G ------
2
R
,
Mm Mm
G -------------------
- + G -------------------- = mg0h2 mg0h1. (6.25)
R + h2 R + h1

, -
! -
m, , -
, -

300
, .
, -
(6.25),
.

1 (. 6.12). -
m
, -
?
A(0 1) r = R
m ( 0) -
( 1)
. 6.12
6.10 F(r)
r = 0 r = R; , A(0 1) =
= 1--- mg0R. (5.70) A(0 1) = U(0 1) = U1 U0.
2
, , U0 = 0, U1 = A(0 1) = 1--- mg0R. ,
2
m , -
,
U = 1--- mg0R. (6.26)
2

2. m
,

?
U
F(r) 6.10, -
.
R ,
(6.26). :
U = U U0 = (U U1) + (U1 U0), (4*)
U, U1, U0 -
, , , .
(6.23)
2
mg R
0
U = -------------------
-. (6.27)
r
r = R U1 U; -
,
U U1 = mg0R. (6.28)

301
(6.28) (4*) , , (6.26), U1 U0 =
= 1--- mg0R,
2

U U0 = mg0R + 1--- mg0R = 3--- mg0R.


2 2

, m
, ,

U U0 = 3--- mg0R. (6.29)


2

6.3.

6.3.1.


. (6.16) ,
m
2
g R
0
m = -------------
-. (6.30)
G

G, -
m, g0 R.
g0 = 9,8 /2, R = 6,37 106 , G = 6,67 1011 2/2,
(6.30)
m = 5,96 1024 .
,
. M m , -
, T ,
r (
). ,
, -
(6.7):
Mm
F = G ----------
2
-. (1*)
r

302
,
2
v
------ (v
r
);
2
F = mv
----------- . (2*)
r
2
2r
(1*) (2*) , M = vr
--------- . , v = ---------- , -
G T
:
2 3
M = 4 r
--------------
2
-. (6.31)
GT
r = 1,5 1011 , T = 1 = 3,16 107 , , ,
M = 2 1030 .
, 333 000 .
(6.31) -
, ,
.
, , ,
.
.
(r = 1,88 109 )
(T = 16,7 = 1,44 106 ).
(6.31), ,
M = 1,9 1027 .
319 .
, -
. -
r = 9400 , -
= 7 47 = 2,8 104 . (6.31), -
,
M = 6,4 1023 .
, ,
r = 1,22 , -
T = 15,8 = 1,37 106 . -
(6.31), ,
M = 5,7 1026 .

-
.

303
. = 81,2 ,
= 3,68 . -
g
g0
?
,
(6.30):
2
g R
m = ---------------- , (1*)
G
m R , .
(6.30) (1*),
g0 R 2
--------- = ------- -------- .
m
m g R

m R
- = -------- = , :

--------
m R
g0
------- = ----2- = 6.
g
g = 1,62 /2.

6.3.2.

9 -
. .
ri , Ti -
i- (i = 1, 2, 3, ..., 9). (6.31),

3
ri MG
-----2- = ----------
2
-, (6.32)
Ti 4
M . ,
3 3 3 3 3
r1 r2 r3 r9 ri
-----2- = -----2- = -----2- = ... = -----2- ; -----2- = const. (6.33)
T1 T2 T3 T9 Ti
, , -
, , .
-
. ri -
-

304
(6.33).

( ),
(..):
r = 1 .. = 1,49 598 1011 = 150 .
, r1 r9,
(6.33) -
, . . .
. ( )
, , r1 = 1,22 r2 = 12,95 .
T1 = 15,8 .
-
, .

2r 9
2 1, 22 10
v1 = -----------1- = ------------------------------------
6
- / = 5,6 /.
T1 1, 37 10
(6.33) , ,
,
, ri Ti i-
.
3 3
r1 r2
-----2- = -----2- , T2 , -
T1 T2

r1v12 = r2v22,
v2 . ,
v1 r2
------ = ----- . (6.34)
v2 r1

,
(6.34): ,
,
.
, .

6.3.3.

T ,
, . . -

305
r ( 0,055,
).
,
F = G Mm
----------
2
- (M , m ). -
r
, -
2
v
------- (v
r
2
Mm mv
- = ------------- , -
). , G ----------
2
r r
2 M
, v = G -----
- , , (6.16)
r
2
2 R
v = g0 ------- . (1*)
r
2r

v = -------------
- , (1*)
T
:
2 2
g0 R T
r = 3 --------------------
2
-. (6.35)
4

-
? . ?
, -
. ,
.
,
( )
T = 29 12 44 = 42 524 = 2,55 106 .
. (6.35)
, g0 = 9,81 /2, R = 6,37 106 ,
r = 403 000 .
, , ,
T T, -
,
. -
( synodos,
, ) ( siderium, -
). T -

306
, -
T -
. (6.35)

.
, , T, T. 6.13 -
* , t1 t2, -
. , t2 t1 = T.
2r,
AB r; ,

( 2 + )r r
T = --------------------------
- = T + ---------- . (2*)
v v

. 6.13

T , v -
. -
,
r = (t2 t1) v = Tv. (3*)
(3*) , (2*) :

r v
T = T + T ------------- . (4*)
r v
r r
2 ------
- = T 2 ------ = T, ,
v v

TT
T = T + ---------- TT = TT + TT.
T
TTT, :

1 1 1 TT
--- = ----- + ------- , T = ------------------
-. (6.36)
T T T T + T

307
, T = 2,55 106 T = 365,26 = 31,56 106 . -
(6.36),
( ):
T = 2,36 106 = 27 7 43 .
, -
2 5 1 .
T = 2,36 106 (6.35), :
r = 384 000 .

6.3.4.

-

(,
, , ), ,
( , -
, ).
-
.
-

. -
,
,
, ,
. , -
, ,

, -
. ,
,
.

, ( ) -
.

.
XVIII ()

, -
.

308
-
: XIX
,
. ,
,
,
.
-
, -
.

1846 . . -
,

, -
.
,
XIX , -
. -
, () , -
. -
1930 .
: .

6.4.

6.4.1.

m r -
F = G Mm
----------
2
- (M
r
2
), v
------
r
(v ).
2
Mm mv
, G ----------
2
- = ----------- , ,
r r

GM
v = ----------- . (6.37)
r

309
(6.16)
2
g0 R
v = -------------- . (6.38)
r
g0 = 9,8 /2
, R = 6370 .
r = R (6.38),

V1 = g 0 R = 7,9 /. (6.39)
.
,
.
, -
-
. -
-
.
h -
. r = R + h, (6.38) -
:
2
g0 R R
v = ----------------- = V1 ----------------- . (6.40)
R + h R + h

h n R, , (5.52), -
(6.40) :

v = V1 1 + ------- d V1 1 ----------- .
h 1/2 h
(6.41)
R 2R

1957 -
h = 228 . (6.41)
. 7,76 /, 0,14 /
.
( h),
(. (6.40)). , , ,

. h -
, -
,
.
A, m
, -
h . , , -

310
. -
, -, -
. , ,
.
W
h:
A = W = K + U(0 h), (1*)
K U(0 h)
, .


(5.30). "
"
. A, "
A (0 h). ,
A + A (0 h) = K. (2*)
, (5.70), A (0 h) = U(0 h), (2*) "
(1*).

K
2
mv
----------
- , v -
2
(6.40); ,
R
K = 1--- mg0R ----------------
-. (3*)
2 R + h
U(0 h) ,
(6.23),

U(0 h) = U(h) U(0) = (G ----------------


Mm
- ) (G ----------- ).
Mm
R + h R
(6.16)
R
U(0 h) = mg0R(1 ----------------

- ). (4*)
R + h
(3*) (4*) (1*),
R
A = mg0R [1 -------------------------

- ]. (6.42)
2 ( R + h )
, h A,
h.
, -
A = W.

311
1. m,
R
h1 = ------- ,
10
R
h2 = ------- .
3
?


A2 A1, A1 A2 ,
h1 h2 . -
(6.42),

R 6
A1 = mg0R 1 ------------------------------
- = -----
- mg0R;
R 11
2 R + -------
10

R

A2 = mg0R 1 --------------------------
- = 5--- mg0R.
R + R 8
-------
3

7
A2 A1 = -----
- mg0R.
88
:
? -
. 6.4.3.
-
. -
. ,
,
-
.

2. ?

(6.35),
T -
. T

T = 1 = 8,64 104 .

312
-
, -
. -
(6.35) , R = 6370 ,
: r = 42 230 .
h = 42 230 6370 = 35 860
.

6.4.2.

, (, -
, , ) -

;
, .

.
-
; -
,
, -

.
, ( ) -
r1; -
2
g0 R
, (6.38), v0 = -------------- .
r1

() v
; v1 = v0 v.
A ,
.
, -
.
6.14, ( )
( ) ; A
, C . OA
A ;
r1. OC ;
r2 (r2 < r1).
r r
= ----------------
1 2
. (6.43)
r1 + r2

313
)

. 6.14

A
C v1 v2. (6.1)
v2 1+
------ = ------------ . (6.43),
v1 1

v2 r1
------ = ----- . (6.44)
v1 r2
, (6.44) ,
, -
mr1v1 = mr2v2 (. (6.2)).
C A
v2 v1.
-

. K(A C) U(A C)
A C , -
:
2 2
mv mv
K(A C) = ----------2- ----------1- ; (1*)
2 2

U(A C) + G ----------- G ----------- =


Mm Mm
r2 r1
2 1 1
= g0m R ----- ----- . (2*)
r1 r2

, , -
,
K(A C) + U(A C) = 0. (3*)

314
A C
. -
(1*) (2*) (3*),

--- m v 2 --- m v 1 + g0m R ----- ----- .


1 2 1 2 2 1 1
2 2 r1 r2

,
2 1 1
v 2 v 1 = 2g0 R ----- ----- .
2 2
(6.45)
r2 r1

v v
, v 2 v 1 = v 1 -----2- + 1 -----2- 1 -
2 2 2
v1 v1
(6.44), (6.45)
2 r
v 1 ----1- + 1 = 2g0 R ----
2 1
-. (6.46)
r2 r1

, -
, -
2
g0 R
r1 v0 = -------------- , -
r1
.
A ,
(. 6.14, ). ,
6.14, , A , -
. v1 (v1 = v0 + v)
v2 A C
, r1 r2 -
(r1 < r2).
A C (v2 < v1), -
. - -
(6.44). (6.45)

v 1 v 2 = 2g0 R ----- ----- ,
2 2 2 1 1
r1 r2

- , , -
(6.46).
, r1 -
2
g0 R
: v0 = -------------- . -
r1

315
, -
, r1 .
, -
, r1 .

, -
! v0
. v0 ,
r2 R (
, R + H, H
). r2 < R + H -

. v0
()
(), ,
. -
.
6.15 -

I VII.
-
-
v0
A, r1
. -
III
( -
2
g0 R
-------------- , A,
. 6.15 r1
).
VII ( -
-
).

1.
r1 = 3R. -

n = 1,2 , .

( ) -
.

316
r2
v2, , (6.44)
(6.46)
0 g R
r1 = 3R, v1 = nv0 = n -------------
- (4*)
3
g R0
(v0 = -------------
-
3
3R). (6.46) (4*)
:
3R
1 2
(
--- n g0R 1 + -----------
3 r2
)= 2
--- g0R,
3
,
3R 2
1 + ----------- = -----
2
.
r2 n
,
2
3n R
r2 = -----------------
2
; (5*)
2n

v r g0 R 2
1 1 2n
v2 = ----------
- = -------------- (2 n2) = V1 --------------- . (6*)
r2 3 3
n = 1,2, , ,
r2 = 7,7R = 49 000 , v2 = 0,32V1 = 2,5 /.
2. , ,
, , ,
3R,
.
(5*) , r2 ( -
-
) n = 2 . -
3R
2
g 0 R g0 R
v0 = -------------- = -------------- ,
3R 3

v, -
, :
2g 0 R 2
v = 2 v0 = ------------------ = V1 --- ,
3 3

317
V1 = 7,9 / . , v=
= 6,5 /.
3. , , ,
, R,
.

r1 = R, (4*) v1 = nv0 = n g 0 R
2
n R
(5*) r2 = --------------
- . , r2 -
2
2n
n = 2 . ,
v = 2 v0 = 2g 0 R = 2 V1 = 11,2 . (7*)

: -
! , R
v0 = 7,9 / , 2 v0 =
= 2 7,9 / = 11,2 /. -
g0 R
3R, v0 = -------------- = 4,6 /,
3
-
, 2 v0 = 2 4,6 / = 6,5 /.
: -
v0, -
, 2 v0.
, ,
.

6.4.3.

. -
2
g0 R
r1, v0 = -------------- .
r1
-
r2,
2
g0 R
v3 = -------------- ,
r2
r1 r2,

318
. 6.16, ,
,
,
, -
A C. -
r1 r2
( , -
A C ),
A C v1 v2 ,
r1 r2 -
(6.44) (6.46):
v2 r1
------ = ----- ;
v1 r2
2
(1*)
r R
2
v1 ( )
1 + ----1- = 2g0 ------- .
r2 r1
A v1, -
, v0,
,

vA = v1 v0. (2*)
C ,
, -

vC = v3 v2. (3*)
, -
A C.

. 6.16

319
, ,
v1. A C
v2, -
v3, .
-
.
(1*) (2*) (3*),
2 2
g0 R g0 R
vA vC, , v0 = -------------- v3 = -------------- .
r1 r2
2
0 2 2 2g R r
(1*) : v 1 = --------------------------
- . -
r1 ( r1 + r2)
,
2r 2
v1 = v0 ---------------- . (6.47)
r1 + r2

(2*) :
2r
vA = v0 ----------------
2
1 . (6.48)
1 + r2
r

(1*) (6.47),
2
r r 2r g0 R 2r 1
v2 = v1 ----1- = v0 ----1- ----------------
2
= -------------- ---------------- .
r2 r2 r1 + r2 r2 r1 + r2

,
2r1
v2 = v3 ---------------- . (6.49)
r1 + r2

(3*) :

2r
vC = v3 1 ----------------
1
. (6.50)
r 1 + r 2

. 6.16, -
-
2
g0 R
r1 v0 = -------------- -
r1
2
g0 R
r2 v = -------------- .
r2

320
,
A C. r1 -
, r2 . A -
-
vA = v0 v1, v1
A . -
A C v2,
v. C -
, -
vC = v2 v. ,

, -
.
v1 v2 -
(6.47) (6.49), . ,
2r 2
vA = v0 v1 = v0 1 ---------------- ; (6.51)
r1 + r2

2r
vC = v2 v3 = v3 ----------------
1
1. (6.52)
1 + r2
r


, , -
. -
r2 = R + H, H
.
r2 -
.

6.4.4.

, -
, ,
, . -

.
V2 2 :

V2 = V1 2 = 11,2 /. (6.53)
(6.53) -
3 . 6.4.2. .

321
, -
R (. . ),
g 0 R = V1. A -
v,
-
A. v, -
( ).
, v = ( 2 1)V1 -
,
, (, -
). ,
A, 2 V1,
, ;
,
. , 2 V1
.

( -
R).
, -
-
-
. -
r

U(R r) = G Mm Mm
----------- G ----------- .
R r
-
r = ;
Mm
U(R ) = G ----------
-.
R

G Mm
----------- , -
R
2
mv
----------- :
2
2
mv Mm
----------- = G ----------- . (1*)
2 R
v , ,
, , -

322
- -
. , -
V2. (1*)
2
mV 2 Mm
------------ = G ----------- . (2*)
2 R

m , GM
----------
2
- = g0, (2*):
R
2
V 2 = 2g0R.
,
V2 = 2g 0 R = 2 V1.

6.4.5.

, -
, -
,
.
r0 , m M
. U(r0 ) -

, r0 ,
mM
U(r0 ) = G --------------

.
r0
,
2
mv
------------ , v -
2
, -
r0 , -
. ,
2
mM mv
G -------------- = ------------ ,
r0 2
,
2GM
v = ------------------ = 42,2 / (6.54)
r0

( , M = 2 1030 , r0 = 1,5 1011 ).

323
m - -
r0 .
, (. . )
v0 ,
, ,
2
v
F = Ma, M , a = -----0-
r0
M M

, F = G ------------------
2
-
r0
. , -
F = Ma :
2
M M v
G ------------------
2
= M -----0- ,
r0 r0

,
GM
v0 = -------------- = 29,8 /. (6.55)
r0

, , -
v0 = 29,8 / ( -
). -

, -
,
v = v v0 = ( 2 1)v0 = 12,4 /. (6.56)

-
! . , -

-

2
m (v )
--------------------- , , ,
2
, -
2
mV
, ------------2 , V2 -
2
. , -

324
2
m (v )
, , --------------------- +
2
2
mV
+ ------------2 ; ,
2

2 2 2 2
V3 = ( v ) + V2 = ( 12, 4 ) + ( 11, 2 ) / =
= 16,7 /. (6.57)

. -
,
, -
?
V3 ,
, , -

2 2
V3 = ( v + v 0 ) + V 2 = 72,8 /.

6.4.6.
,


* , (-
, , , , ,
). -
-
, , -
. ,
. -

. -
( , , )
, -
.
.
( -), -
-
.
. 6.5.1
-2 () ,
12 ( 1977 . 1989 .).

325
, ,
, . v1 v2 , -
, ,
, . -
, -
A , B (. -
1 6.17). ( -
) r0 = 1 . . = 1,5 1011 ,
( ) r = 5,2 . . v1 v2
, :
v2 r0
------ = ------- ;
v1 r
(6.58)
r0 2GM
2
(
v 1 1 + -------
r
)
= -----------------
r0
-

(M = 2 1030 ), , ,
(6.44) (6.46). ,
2
g0 R = GM (M ), GM (
,
; , ,
, ).
(6.58), , -
(6.47):
2r

v1 = v0 ------------------
-. (6.59)
r0 + r

. 6.17

326
GM
v0 = -------------- = 29,8 / .
r0
(6.59) , , -
, , ,

v1 = 38,6 /, (6.60, )
( )
v1r
v2 = -----------0 = 7,4 /. (6.60, )
r

v1
-
, . 6.4.5. , -
,
, (6.56):
v1 = v1 v0 = (38,6 29,8) / = 8,8 /.
-
, . (6.57),
, -
:
2 2 2 2
v1 = ( v1 ) + V2 = ( v 1 v 0 ) + V 2 = 14,14 /, (6.61)
V2 = 11,2 / .
,
(. 2 6.17). (6.59),
r
r = 30,1 . .,
:

2r v r
1 0
v1 = ------------------ = 41,5 /, v2 = ----------- = 1,4 /. (6.62)
r0 + r r

.
(6.61) (6.62):

2 2
v1 = ( v 1 v 0 ) + V 2 = 16,2 /. (6.63)

t,
, 2 A
C. (6.31),

327

r
2 3
4 r
T = --------------- . (6.64)
MG

,
. T -
,
(6.64), r a:
2 3
4 a
T = ---------------- . (6.65)
MG


0r +r
a = -----------------
- = 15,5 . .
2
(6.65),
:

t = 1--- T d 109 d 32 . (6.66)


2

: , -
!
. 32 -
30 . . .

6.5.

6.5.1.

, -
,
, 32 .
16,2 / -
.
,
, -
. , ,
, ,
( -

328
), -
, , -
.
-
-
, . -

. -
XIX -
,
-
.

: ,
, ,
. -
.

XX ( 1977 1989 )
-2 , -
- (, , ).

. -
-
(
32 , 12 ),
.

. 6.18 -2.
* 800
20 1977 .
() 3-. 1979 . -2 -

. 6.18

329
71 . ,
, , , -
101 . 1981 . -
(-
107 . 1986 .),
. -2 48 .
1989 . 12 . -
( ) -
-2,
8 , 10
, 6 . -2 -
. 2005 .
76 . . = 11,4 . , . . -
, .
-2 2030 .
. -
. 20 319 -2
3,5 ,
296 036 47,4
...

-2
, - ,
(6.61), 14,14 /. , ,
, ,
(6.62), 16,2 /. -
6.19 , -
-2,
, ( -
). A -
; 38,6 / (. (6.60, )),
14,14 / . B
:
7,4 / (. (6.60, )). -

7,4 / 22 / ( B C). -
D E ( 18,1 / 24 /) -
.
( 21,5 / 23 /)
. -
-
20 /, -
.
, -
v(r) r

330
. 6.19

6.19, -
(6.45):

v2(r) v12 = 2GM 1--- ----


1
- , (6.67)
r r0

v1 = 41,5 / ( (6.62)).
1,4 /.
,
-
, , , ,
.
(. 6.20, ).
.

,
; -
-
)

GM
U(r) = -------------C- .
r )
-
,

( (6.67)). -

(, , ),
, , -
U(r) -
- . 6.20

331
r d rM
------------ , r M
M
(. 6.20, ; r -
). -
,
.
-
, , ,
, -
, 6.20, .

6.5.2.

6.21 ,
* , O -
l KN, -
v 0 ( ,
).
KN KN. -
KN v 0 -
;
v 0 = v0. NCN ( )

v
0
v 0 , . . , .
, -
O O ( ) -
. O ,
( O) OCO, CO = CO (. ). -
OB 2l
, OO = ------------- = ------------- .

cos --- cos ---
2 2
A -
; OA r.
A O O
2l
AO AO = OO 2AO = ------------- 2r.

cos ---
2
O O
, -

, OB = 2ltg --- . -
2
, ;
,
l
ltg --- = ------------- r.
2
cos ---
. 6.20 2

332
r , ,
1
-------------- = 1 + tg2 --- . :
cos --- 2 2
2
2 2
l r
tg --- = ---------------- . (1*)
2 2lr


v A.
,
mlv0 = mrv, (2*)
m ( ,
, -
). ,

2 mv 2
mv 0 Mm
----------- ----------- = G ----------- , (3*)
2 2 r

M . (2*) (3*) :
2 2
l r GM
---------------- = 2 ---------
2
-.
r v0

(1*), :

GM
tg --- = ---------- . (6.68)
2 2
lv 0


-
.

6.5.3.
.

, , -
,
. -

v 0 v 0 , 6.21,
. -
, -

v v
. -

333
,

( V ).
6.22, -
, , -
, 6.22,
, .

v = V + v 0 v = V + v 0, (6.69)

, v v.
6.22, .
-
: v > v. -
-2 , , V = 13 / ( -
), v = 7,4 /, v = 22 /, v v =
= (22 7,4) / = 14,6 /.
, 6.22, ,
, , ,
-
6.22,
, (. -
6.22, ).

) )

) )

. 6.22

334
( ) -

. -
,
,

. . 5.4.3 , -
,
v1
v 2 ( m) -
( M) ,
u1
u 2
(. 6.23). x-
(5.36),

-----
m
1 v 1x + 2v 2x
M
u1x = ------------------------------------------------- . (1*)
m
----- + 1
M

m
M . m, -----
M
(1*) u1x = v1x + 2v2x, , -
,

u1 = v1 + 2v2. (2*)

, -
2v2, . .
.
, -
2v2.

. 6.23

-
6.24, , -

335
)

. 6.24

6.24, .
-
. ,
,
, .
.

6.6. ?

6.6.1. ...

, , , ?
? -
?

. -
-
.
, -
. ,

, - , .
, -
, -

336
, ,
.
-
. , -
. -
,
.
.
, -
. -


.

.
. :
,
. , -
-
...
( ), -
, -
, -
. ,
.
, 1731 : -
, -
, . , -
, , ,
, ; , -
,
; -
, ,
... , -
, ;
, -
, ,
...
, ,
,
( XIX )
. -
. , ,
, : -

337
,
, ...
, :
, -
,
. -
, :
, -
.
XIX
,
. ,
, -
. , -

. -
, , -
, , -
, , , -
.
,

, , ,
, .

6.6.2. ( )


( ).
-
, ,

( ). , m -
M,
, m ,
M. 6.4. ( -
) -
. 6.5


.

( m1 m2)

338
. m1, -
- r1, , -
m2, -

r 2, (. (6.6)):
m 1
F 12 = G -------
3
(r 1
r 2)m2, (6.70, )
r

r = |
r 1 r 2|.
, m2
, m1
m 2
F 21 = G -------
3
(r 2
r 1)m1. (6.70, )
r
M
r -
m. , , M,
m,

F = G M
----3- r m. (6.71)
r
( (6.70, ), m1, m2,

r 1, r 2 M, m, 0,
r , .)
M , -
- r , ,
, , -

.
, , -
, r .
(6.71),

F Mr
- = G --------
 = ---- 3
. (6.72)
m r

, . -
, , -
.
( , ) , ,
(6.23),

= U M
----- = G ----- . (6.73)
m r

339
-
,
.

-
: , -

, ,

.  (A) (A) -
-
,
. 6.25 A
M1, M2, M3.
r 1, r 2,

r3 -, A
(. 6.25). (6.72)

(6.73),  (A) (A) :

1 M M
2 M
3
(A) = 1(A) + 2(A) + 3(A) = G -------
3
- r 1 G -------
3
- r 2 G -------
3
- r 3;
r1 r2 r3
(6.74)
M M M
(A) = 1(A) + 2(A) + 3(A) = G -------1- G -------2- G -------3- .
r1 r2 r3
(6.75)
, , , -
-
,
.
( ) ,
. , -
, , -
(6.72) (6.73), M ,

r -, -
.

. -
-
, .
, -
. , -

? -

340
XX , -
. ,
,
.

6.6.3.

, , .
, -
.
( -

mi). F , ,
,

F
g = ------ , (1*)
mi

g . ,
(
mg), ( -
). -

, , -

F :

F
 = -------
-. (2*)
mg

(2*) , F = mg  . F (1*),
m
g = -------g-  . (6.76)
mi

:

= .

-
. , ,
-
:
mg = mi. (6.77)
( 1971 . . . .
(6.77) 1012.)

341
(6.77) , -, -
, -
. -, (6.77) ,
, -
:

 = g. (6.78)
-
,
-
. , , -
, ,

m g , -

g ( m g -
), ,

( m g ).
.
:

-

.

, -
, -
() -
. -
(
, , )
, -
. , -
. -
, ,
. , ,
. ,
, ,
( (2.47)).

6.6.4.


* , .

342
-
. ,
, -
, -
, .
-
,
. ,
. -
, , .
, : -
, ,
, ,
, -
, .
, ,
( , -).
. -
, -
-
. -
.
,
: ,
.
, -
. -
, . -
.
( ) -
, ,
. ( )
- .
,
.
(-
) : -
, . ()
, .
-
, -
, . .
:
|| n c2, v n c, (6.79)
c .

343
,
, , , -
. -
. , -
, ,
. ,
, -
, -
. , , -
,
.
, -
. -
: ,
- . -
, -
, .

6.1. - ? 2 1030 .
- 6.6.
, -
? - 1630 ,
= 0,21. . -
6.2. 26 500
1,52 .. . -
.
? 6.7.
6.3. - n = 1,5 -
? -
2 1030 ? R =
6 1024 , - = 6370 .
1 . . = 1,5 1011 . 6.8.
6.4.
- , .
100 , - -
- -
4 106 ? ,
6 1024 . ?
6.5. - 6.9. ,
, - , -
,

344
,
r = 7370 T = 104 . -
6.10. - A.
6.15.
- , ,
R, - 0,53 R 0,11 M, R M
. . -
6.11. - , -
= 81 . g
r = 384 000 .
-
V -
, -
, g0,
,
- V1 . -
- , ,
. = 5500 /3, g0 = 9,8 /2,
6.12. V1 = 7,9 /.
( ) r = 1,88 6.16. ,
, - 0,53
T = 16,7 , - , 0,11
r = 71 400 . , ,
, -
- .
- 6.17. :
g - ) -
r . , )
. .
- M = 1,9 1027 , -
(6.31).
R = 71 400 .
6.13. -
- 6.18.
,
. , . -

T = 238 40 . T.
, - ?
. ,
.
6.14. A B - ma, m a
- -
. , .
A - 6.19. ,
V. - = 5500 /3,
B,

345
T = 24 , , 6.25. -
, v1 -
, - -
. ,
6.20. - ?
, - T ?
r =
= 9,54 .., -
n% = 2% , - r = 1 ..,
. - M = 2 1030 .
T = 12 . 6.26.
6.21. - , -
- n = 3 -
, .
,
- ? ,
? -
M= 6 1024 R = -
= 6370 , . ,
6.22. -
? -
- R = 6370 .
. 6.27. -
,
, , - m = 200 , -
?
7,34 1022 , -
1740 . 3
6.23. - , -
- ?
, - R = 6370 .
? .
= 9,1 - ,
, R = 6370 , 5
(6.42) --- mg0R; -
R = 3400 . 6
6.24. A 2 - -
B: -
RA = 1--- RB. , - ,
2 v, -
- .
. 6.28.
-
MA B r1 = 7400 . -
--------- ------ .
MB A

346
, . -
- (6.48) (6.50).
= 0,6? 6.31. : ) -
R = 6370 . ,
. - ) -
(6.46). .
6.29. 7,34 1022 1730
- .
r1 = 7400 . - 6.32.
-
, , -

v = 0,2 /. - . -
- -
, -
,

-
.
?
R = 6370 .
7,34 1022 1730 ,
. -
.
(6.46), v1 =
. -
= v0 v (v0 -
r1). ,
6.30.
- ,
r1 = 6800 . - .
6.33. -
-
-

- ,
r2 = 7500 . -
. -
v1
, -
,
- ,
( -
-
, r2)?
?
v2 M =
, - = 7,34 1022 R = 1730 ,
- .
, . -

r2? 4R R 3
- = --- .
= --------------------------
R = 6370 . 4R + R 5

347
6.34. - 6.36.
-
.
4 107 .
- ?
, 2 1030 6 1024 ,
. -

1,5 1011 3,84 108 , -
-
.
.
6.37. -
6.35.

,
r = 30 000 -
? .
M = 2 1030 , = 630 /3.
r = . -
= 1,5 1011 . (6.20).

348
7


()

7.1.

7.1.1.

.
m -
S
(. 7.1). -

P ,


m g. P
,
.
( , . 7.1
) .
(. . ) , -

, N
, -

. P N -
(. . )
.

P N 7.1.
, -
.
(. 7.2, ).
(
, , . -
7.2, ).

349
)

)
. 7.2

7.2 , . , -
, S, -
S . i- -

Pi,

. 7.1 P
S;
:

P

P = i
i

. -

Ni

N .

N = i.
i

7.2
.
, P1 = P2 = P3 = ... , , N1 = N2 = N3 = ...
(. 7.2, ).
, P1 P2 P3 ... , N1 N2 N3 ...
(. 7.2, ).

, , -
.
, , (
- ) -
. 7.3, -

; Z.

350
)

)
)

. 7.3

.
,
, -
Z (. 7.3, , -
). ,

() -
, -
. ,
-
Z , 7.3,
, -
. (,
7.3, ),
(, -
).
,
-

N , -
(. 7.3, , ,

N 1 = P 1, N 2 = P 2, N 3 = P 3, ...).
,
, , -

. ,
, -
() . , -
,
()
. ,

351
Z
,
.

(
), , -

( 7.4).
-
. 7.4 (,
A
).

, , ,
-
. , 7.4
,
A,
O1A, O2A, O3A. A
, ,
-
( -
) -
,
A.
,
,
,
, . ,
-
( , -
). , -
-
, . -

.


. ,

; -
,
.
,
? ?

352
: 1) -
( , -
); 2) ;
3) (-
, ,
).
-
. , ,
, , -

(
).
. , -
. -
, -
.

7.1.2.


P
S. , -
, -, , -
, ,

. , P -
. p -

P S -
:
P
p = -------
-. (7.1)
S

P
S
, p -

p = P
---- . (7.2)
S
, , -
, -
, , 7.1.

353
S -

F ,
,
(. 7.5), -
, ,
, ,
. 7.5 (7.2):

p = F
---- . (7.3)
S

: . -
! , -
( ,
)
.


, p A A,  
S, 
P
------- A:
S
P
pA = lim  . (7.2*)
S A S


, ( 
, , . .). 


.
,
;
. , , .
A, 
m
 , V , A,
V
m . A A
V A:
m
A = lim  . (7.4)
V AV
,
.

: , .
, -

354
, , -
.
;
( ) .
-
, . , -
, , -
, , -
.

( ) -
.

7.6.
() 1
2.
3, O
, ,
.

, . 7.6
-
.
,
, ,
. -
. , -
,
. , -
-
. , -
, ,

, , (. . 9.1.5).
, , , -
.

(7.3) ,

= .

-
; -
(: ) -

355
.
1 = 1 /2.
, 1 , -
1 2,
.
:
(1 = 105 ), (1 =
= 101 325 ),
(1 . . = 133,3 ; 1 = 760 . .).

7.1.3.

XVII ,
:

, , -
.

, -
; -
( ).
, ,
.
,
, (. 7.7).
,
, () . -
, ,
( , ,
, ).

. 7.7

356
, -
A B ,
. -
, AB, -
S. -
,
( ). -
, ; -
, -
( -
(3.10)).
AB. , , -
, , -
, -
, AB. ,

AB P A P B, -
:
PA PB = 0. (1*)
(7.1), PA = pAS PB = pBS, pA pB
A B, . -
(1*), , pA = pB.
.
,
, ,
. , , -
A C (
AC 7.7 -
). . -
, , pB = pC
(pC C). pB = pA, ,
pC = pA.
, (
, )
. -
p D, , (7.3), p = F
---- , S
S
, F ,
.
, , D -
p = F
---- ,
S

357
.
:

, , -
-
.

( )
S, , ,
E, F, G, H, K, L (. . 7.7). -
p = F
---- ,
S
,
P = pS = F S
------- -
S
(, ) . -
7.7 .
(. 7.8). -
F -
d. -
l.

-
. 7.8 .
.
, -
F
, p = ---------
- . -
2
d
---------
4
P
2
d
(pdl) p --------- . ,
4
2
d
P = p dl + --------- = F ----- + 1 .
4l
4 d

7.1.4.

. -
,
. -
; ,

358
. -
,
, -
. ,
-
.
A B, -
,
AB S
(. 7.9, ). : ,
.
x- ( AB)
. ,

( AB). ,
,
:
pAS pBS = 0 (1*)
( pA pB A B, ).
(1*) ,
pA = pB. (7.5)
1:

,
, -
.

,
, , , .

) )

. 7.9

359
.
-
.
A B,
, -
AB; AB h (. 7.9, ).
x- ( AB) , -
. -
S , pAS pBS, pA
A, pB B.
mg = hgS (m ). -
,
pAS pBS ghS = 0,
,
pA pB = gh. (7.6)

p = gh (7.7)
.
: A B -

(. 7.10). ZA
, A,
ZB B. -
h.
, ZA
ZB. B -
. 7.10 ZA C
ZA.
(7.6), pC pB = gh. pC = pA ( C A
); , pA pB =
= gh. , 2,
(7.6):


( gh)
-
, h. -
, .

360
S -
h. -
, P, :
P = hSg = pS, (2*)
P
p . (2*) , p = ---- . ,
S
p = gh ,
() P , h.

-
. , -
-
.

, , , 7.11 (14).
-
:
p(z) = gz. -
( -
, -
). ,
,
. -
S
P = ghS, (7.8)
, ,
P = Vg, (7.9)
V . ,
V = hS; -
. , -
7.11,
. , P > P 1 3,
P < P 2 4.

. 7.11

361
A h
(. 7.12). pA
, -
(7.6). pB
p0 (, -
). -

pA = p0 + gh. (7.10)

. 7.12

( -
) . -
.
, . -
h, , -
. , -
, ,
p = gh. ,
, . . -
p0.
(7.10)
p + p0 . , , -
( )
,
,
p0.
(
) .
1.
p0 = 780 . . n =
= 1,5 ?

362
(7.10), -
0p
pA = 1,5 p0. , h = ----------
-.
2g
, p0 = 780 . . = 104 000 = 1000 /3,
h = 5,31 .

2 (. 7.13). -
S = 7500 2
h1 = 80 -
.
-
s = 1 2.
V = 400 3.
-
, ?
, , . 7.13
h2 = V
---- = 4 , -
s
-
p = gh2 = 3,92 104 . -
,
p. P -
P = p, , :
= 2S + 2h1 S , = 2,9 2.
, P = 113 700 . 30 000
, !

3. d = 30
. -
,
( P) -
( P).
:
!
-
: p(z) = gz (. 7.14, z-
, z = 0 ). -

z, , -
. -

363
. 7.14

, 7.14,
gz dz. -

. -
, P = 1--- dgh2.
2

gh; , P = 1--- d2gh. ,


4

P = P. h = d
--- = 15 .
2

7.1.5.

. 7.1.4, , , -
, , -
( ,
), . , -
.
7.15 -
.
,
(
), -
-
. -

( Z )
. 7.15 A B , -

364
. , A B -
: p = p. ACDB,
. h1 h2 -
A C D B,
(. ). p pD C D.
(7.6), :
pC pA = gh1;
pD pB = gh2.
; :
(pC pA) (pD pB) = g(h1 h2). (1*)
(7.6), pC pD = g(h1 h2).
(1*), , pA pB = 0.
,

-
.
Z 7.16 -
, , -
, pA = pB. , -
, h = h.
K N - . 7.16
-
. -
p0: pK = pN = p0. (7.6),
pA pK = gh;
pB pN = gh.
; :
(pA pB) (pK pN) = g(h h). (2*)
pA pB = 0 pK pN = p0 p0 = 0, (2*) ,
h h = 0.

1 (. 7.17). -
1 2
S1 S2, -
.
. 2 , -
1 -
m. -
2
1? . 7.17

365
Z,
1,
pA pB A B, .
Z , 1 . 7.1.4,
pA = pB. (3*)
pA , mg,
; ,
pA = mg
--------- + p0, (4*)
S1
p0 . pB -
p0 gh,
h ; ,
pB = p0 + gh. (5*)
mg
- = gh, -
(4*) (5*) (3*), : --------
S1
m
, h = ---------
-.
S1

2 (. 7.18).
m -
1, -
M = 1600 , -
2?
m1 = 5 m2 = 400 .
S1 = 0,5 2
. 7.18 S2 = 50 2.
A B , pA = pB. -
pA m1, -
1 m +m
m : pA = ------------------
- + p0, p0 .
S1
pB 2,
2 m +M
: pB = ------------------- + p0. pA = pB, , ,
S2

m1 + m m2 + M
------------------- = ------------------- ,
S1 S2

S
, m = (m2 + M) -----1- m1 = 15 . , -
S2

15 !

366
,
. , ,
, -
, ,
. -
.
3 (. 7.19).
-
-
1. 2 -
S
2 V
(1 > 2). : 1) -
-
;
2) -
,

- . 7.19
1.
1. 2
h2 = V
---- . Z,
S
.
( -
1); 1 . 7.1.4.
, A B : pA = pB. -
pA = p0 + 1gh1 pB = p0 + 2gh2, , ,
h
1gh1 = 2gh2, : h1 = -----------
2 2
- .
1

V ( )
h = h2 h1 = h2(1 -----2 ) = ---------------------------
1 2
-.
1 S 1
2. Z , -
-
1
pC pD C D, Z. ,

pC = 1--- 1gh1 + p0 = 1--- 2gh2 + p0; (6*)


2 2

pD = 2g(h2 1--- h1) + p0 = 2gh2(1 1--- -----2 ) + p0. (7*)
2 2 1

367
,
2
pD pC = 1--- 2gh2 1 -----2 = ------
V
- 2g 1 ----- . (8*)
2 1 2S 1

,
. pD pC
, (8*) 1 = 2.
-
.

7.1.6.


, , ,
.
pO. -
S , (7.2), P = pOS. -
; ,
, -
105 . -
pO = 1 = 760 . . = 101 325 ,
.
, , ,
, S = 100 2, , pOS = 107 ,
106 = 1000 .
.
0,2 2; ,
, 2 104 ,
2000 .
- .
?
?
,
. ,
,
;
(. 7.20). -
-
, ,
-
, -
. 7.20 , . -

368
-
, ,
.

. -
, .
,
, ,
.
,
. , -
, , -
, .
, , -
.
, . -
, -
, -
.

- .
,
; , -
, , ,
. .
,
, .
, ,
-
-
. ,
?
, -
, -
.
-
. -
.
,

.
,
. , , -
(. 7.21). . 7.21

369
, ,
, , -
. , -
,

, , ,
. -

( ),
. ,
.
, -
-
, -
-
.
: -
-
-
. -

. -
7.22, p -
H
. 7.22 .


p(H), 7.22,
, 
e = 2,71828 ( ):
H
0 g 
p = p 0e p0 , (7.11)

p 0 0
H = 0 ( ). (7.11) 
.

?
?

, -
. 7.23.

370
-
, -
.

.
, -
h. -
Z ; ,
pA = pB, pA pB -
A B, . pA
. 7.23
gh,
. pB -
p. pA = pB, , ,
p = gh. (7.12)
h , -
, .
( -
) .
-
, , 7.6 (-
1 ). -
-.
. , -
, .
.
;
.
1 (. 7.24). ,
, -

. 7.24

371
: ( 1), (-
2), ( 3). h = 68 ,
h = 5 , h = 84 .
.

p1 , ,
, , , . -

gh = p0 p1, gh = p0 p1, gh = p0 p1,


p0 , ,
h = h = h. (7.13)
h
, = 103 /3, : = -----------

- = 13,6 103 /3,
h
h
= -----------

- = 0,81 103 /3.
h

2. , -
,
p0 = 105 , R = 6,37 106 .

, -
, . .
50 . (6.15, ),
1,5% -
g0 = 9,8 /2. -
,
g0.
p0,
,
, 4R2p0. , -
, mg0; -
, 4R2p0 = mg0, m , . .
. ,
2 p
m = 4 R -----0- ; m = 5,2 1018 . (7.14)
g0

372
7.2.

7.2.1.

7.3, , ,
.
,
. ,
, 7.3,

( N 1, N 2, ...). F
. , -
, ,
, -

, F -

m g :

F + m g = 0.

, F
, (-

) , m g, . .
.
-
.
M. ,

M g, ,
; -

F ,
, . ,

F M g, , . -

, F
, .
, ,
. ,
, , -
, ,
. -
-
. -
.

373
, -
, :

,
, ,
( , -
). ( -
), .
, . -
-
; ,
.

()
():

() -
, ()
, , -
.

-
, . -
, ,
(). ,
,

a , ,
, . -
, , -
, -

, m a , m

, . . , m g, m( g a ) (-
2.5). -

, m g,

m( g a ) (
m(g a)). ,
, , -
-
, ( ). -

, a = g, ; -
, (. .
), .

374

(. 7.25): 1 ( -
) 2,
, -
.
,
.
,
-
. , ,
. 7.25
-
.
:

, (), -
,
().

(. 7.26, ). -
1 = 2 103 /3 -
R = 28
R
---- ,
2 )
-
R
---- .
4
. )
,
, -
-
. -

. . 7.26

3
4--- R3 ------- 1g = 7--- R31g. -
R
3 8 6

4--- R32g , 2
3

. , 4--- R32g = 2 7--- R31g . -


3 6
7
, 2 = --------1- = 3,5 103 /3.
4

375
A l
O ( 7.26, ). -
3
(3.12), : lR3 = l + ---- ------
R R
-,
4 8
R
l = -----
- = 1 .
28

7.2.2.

:
, ?
.
-
, 0 .
, -
:
P P1 = 0gV (7.15)
( , -
). P ,
P1 , 0,
V . gV = P
---- ,

, (7.15) P P1 =
= 0 P
---- , ,

P
= 0 ----------------- . (7.16)
P P1

, . -
. , -
. , (7.16) -
.

. -
(
P), 1 (
P1) ( P2).
(7.15) , :
1gV = P P1; (1*)
2gV = P P2; (2*)

376
2 , V
PP
. (2*) (1*), -----2 = -----------------2 ,
1 P P1
,
PP
2 = 1 -----------------2 . (7.17)
P P1

: , -
! , .
.
.
, , -
, : , -

? ,
.
.

1. , ,
P = 12,1 ,
P1 = 11,3 . , -
, ,
, , -
. 1 = 19 320 /3,
2 = 8 920 /3.

, = 1000 /3,
P
(7.16): = ----------------- = 15 125 /3. < 1, ,
P P1
.
, ,
. m1 V1
,
, m2 V2 .

m +m m +m m
= ---------------------
1 2 1 2
= ---------------------
- = ---------------------------------
V1 + V2 m1 m2 m1 m m1
------- + ------- ------- + -------------------
1 2 1 2

(m = m1 + m2 ).

377
m
, = --------------------------------- , -
m1 m m1
------- + -------------------
1 2
:
m1 m m1 m
------- + ------------------- = ----- ,
1 2


m --------------2- = m1 -----------------
1 2
-. (3*)
1
, 1, 2, (3*):
0,41 m = 0,54 m1,
m
, -------1 = 0,76. ,
m
76% 24% .

2. -
P = 2,59 , P1 = 2,17 .
. = 8,92 103 /3.

V (7.15), 0
PP
: V = -----------------1 = 43 3.
0 g
V .
:
P = (V V)g. (4*)
(4*) (7.15), :
P ( V V )
----------------- = ----------------------------- . (7.18)
P P1 V 0
V = 0 ( ), (7.18)
, , (7.16). , -
(7.18) . : -
(4*) P + Vg = Vg
P + V g
(7.15); ---------------------------- = ------ .
P P1 0
,
( P P ) P
1 0
- ; V = 13 3.
V = ----------------------------------------------
0 g

378
7.2.3.

-
, .
,
. ,
-
. .
V
:
Vg = 1V1g, (7.19)
1 , V1 ,
( -
).
(7.19) , -
. (7.19) :
V1
------- = ----- . (7.20)
V 1

,
: = 1. -
, , -
.
V V,
(7.19) (7.20) , , :
(V V)g = 1V1g; (7.21)
V1
------- = ----- 1 ------ .
V
(7.22)
V 1 V

1. V = 2000 3,
, , -
. .
= 7870 /3, 1 = 1000 /3.

V1 .
V
, ------1- = 1--- . (7.22),
V 2
, , V :
2
1
- = 1870 3.
V = V ---------------------- (1*)
2

379
,
2000 3 130 3 ; -
. -
.
2.
P = 20 . ,
. -
. = 2,7 103 /3, -
1 = 1 103 /3.
, P
, , ,
, -
V ; , P = 1--- 1Vg. , -
2

2P
- , V = 4,110-3 3.
V = --------- (2*)
1 g
V V, -
(1*), .
(2*) (1*),
V = 3,3 103 3.

. -
, 1V1g,
, 2V2g. V1 V2
, , 1
2 . 2 n 1
(2 = 1,2 /3), 2V2g
(7.19) (7.20). -
, , ,
.
-
V,
-
-
2
1 (2 < < 1).

. 7.27

380
S (. 7.27).
V
------1- , V1 ,
V
.
Z2 (. . 7.27)
p = p0 + 2gh, p0
; ,

F2 = (p0 + 2gh)S.
Z1 -
p + p, p -
, 2gh2 + 1gh1. , -

F1 = (p + p)S = (p0 + 2gh + 2gh2 + 1gh1)S.
F1 F2 , , F:

F = (2gh2 + 1gh1)S = 2g(V V1) + 1gV1.



, -
,
.
:
Vg = 2g(V V1) + 1gV1. (7.23)
-
. . (7.23)
,
V1 2
------- = ------------------ . (7.24)
V 1 2

,
. . , 2 . -
2 n 2 n 1, (7.24) ,
, (7.20).

3. -
, -
. .
1 = 13 600 /3, 2 = 1000 /3.

381
V , V1
V
, . , ------1- = 1--- .
V 3
(7.24),
3( 2) = 1 2,

. : = 1--- (1 +
3
+ 22) = 5200 /3. -
.
, -
? ?
: . -
, , , -
. , -
, , .
, -
.

4.
. ,
?

: V ,
v , V1 , -
; 0, 1, 2, ,
, .
:
1(V v)g + 2vg = 0V1g. (3*)
, V2 -
:
1(V v)g = 0V2g.
, (3*) 0V2g + 2vg =
= 0V1g. ,

V2 = V1 v -----1 . (4*)
0

, V1.
, ,

382
V2 + v. , ,
. (4*) :
2 0
V2 + v = V1 v -----------------
-. (5*)
0
2 > 0, (5*) , (V2 + v) < V1. ,
. -
V1 (V2 + v) S
( , -
), h,
:

v -----2 1

0
h = ------------------------
-.
S

5.
. ,
?
v , 0, 1,
3 , , .
(4*), ,
, ,
:

V1 V2 = v( -----3 ). (6*)
0
:
3vg = 0v1g, (7*)
v1 , .
(7*) (6*),
V1 = V2 + v1.
, , , -
,
, , .
, .

7.2.4.

,
*
F , , -

383

F , . -
, ,
.
, , ,
,
( -
).
. , -

. F F
, O O1
(. 7.28, ). ,

= F l + F l,

. F F ,
O O1 (. 7.28, ).

, . :
(
) , -
.

) )

. 7.28

. , -
.
, . .
7.29, . -
, O1, -
M, O.
= Fl,

384
) )

. 7.29

. , l = OM sin,

. ( F F
). M , OM
.
, , -
( )
.
, . -
, , , , -
.
7.29, . ,
O1,
M, O. ,
, -
, , . -
.

7.2.5.

,
. , -
, -
.
, -
. (
convectio, , ).
, ,
0, ,
. , -

385
. -
V .
Vg -
0Vg. < 0 ( -
), 0Vg > Vg. , -
V
, (0 )Vg. -
.
, .
,
. -

. -
. -
. , -
, . ,
,
.

.
, , , (,
), -
(, ). -
-
. -
,
, , . -
-

. -
.
, ( ),
( , , -
, . .),
, .

7.3.

7.3.1.

, , -
, -

386
, -
. -
, ,
. -
-
, ,
. -
r0 -
; r0 d 109 .

, -
r0. , -
,
; -
.
r0,
, , .
, -
. -
, -
;
-
, .
7.30 ( A
B),
.

F ,
B. A
.
-

. 7.30

387
, -
. -
, -
( (5.69)). ,
, ,
,
.
, -
. , -
. -
. , ,
.
, ( -
), . -
-
; ,
-
.
, -
, , .
, -
.
,
, -
( -
). , -
. , -
U
S . -
:
U
= ------- . (7.25)
S

-
( ) , -
.
(7.25) ,

.
=


(: /2). -
1 = 1 , -

(: /).

388

(/) :
() 0,073
0,040
0,022
0,017
0,470

. -, -
.
20 C. -
( 1130 C) = 1,102 /; -
( 253 C) = 0,0021 /; -
( 269 C) = 0,00012 /. -,
- -
, -
. -, -

.
, , ,
, -
(
- ),

( ). ,
,
-
.
?
?
, -
, -
, -
. ,
, -
-
. -
, -
,

.
-
, , ,

389
. -
,
, , ,
, , -
.
, . -
, , . .
, ,
, . , -
- -
.
, -
, .
- . -
-
. - -
.

7.3.2.

-
-
. -
. ,
(), -
. -
, -
,
. , -
(
, ), ,

.
-
, ( -
7.31).
,
, -
(.
7.31, ). , -

, ,
(.

390
) )

. 7.31

7.31, ). -
. -
-
( -
) ,
.
-
.
.
a. -
.
-
, )
a h (h n a).
-
. -
- )
7.32, ;
,
.

. )
(. 7.32, ). -
,
(. 7.32, ),
-
-
() . . 7.32
. -
, -
-
.
, ,
. -
7.33, - . 7.33

391
AB
, .
,
. , , -
, -
.
F ,
l ( -
; ,
l,
F
F). F
----
l
( --------
l
) -
:

= F
---- . (7.26)
l
, -
-
. , -
, 1 ,
1 -
.
, F
----
l
(7.25) , -

l (. 7.34, ).
,
, -
,

F . -
,
F1 = 2F. 2 , -
.
F1, -
F.

) )

. 7.34

392

F 1 -
x (. 7.34, ),
S =
= 2lx. , (7.25)
Un = S =
= 2lx. ,

F 1 x; ,
2lx = F1x.
, F1= 2F, : F = l, . .
(7.26).
,
:
U
= -------n = F
---- . (7.27)
S l
, -
U
-------n -
S

---- . -
F
l
,
.

7.3.3.

7.35, ,
)
() -
. -
, -
MKNL,

)
. -
-
. ,

-
, . 7.35

393
MKNL ( 7.35, ). -
-
, -
(. 7.35, ). -

p -
, , , p0
. ( , p0 -
; , -
p, p0 = p.)
A B
. p A, -
p0 B. ,

p = p0 + p. (7.28)

: A
! . C, h
A, p0 + p, p0 + p + gh, -
.

p, -
, -

; -
. R,

p = 2
------- . (7.29)
R

, -
(. 7.36). ,
MKNL. -
)

-

,
(. 7.36, ).
) -
p

. 7.36 p = p0 p. (7.30)

394
p0 , -

, ( ,
).
(7.29), -

R.
-
-
R R
r0 (r0
). . 7.37

(. 7.37).
. , -
2R.
,
,


(. ). F -
, -
,

2R:
F = 2R.

, -
S = R2.
2
, p = F
---- = ------- .
S R

; 

, 
. , 
, 2R
. , 
F = 2R
,
2
p = ------- .
R

395
1. -
R. ,

p0 p, p0 -
, p
. .
,
R. A, -
4R2p0 4R2p -
R:
A = (p0 p)4R2R. (1*)
U . -
U , (7.25),
U = S, (2*)
S . ,

S = 4(R + R)2 4R2 = 8RR + 4(R)2.


4(R)2
, (2*)
U = 8RR. (3*)
(1*) (3*),
(p0 p)4R2R = 8RR,

, p0 p = 2
------- .
R
: (7.29), -
-
.

2 (. 7.38).
R = 4 . -
p0 = 100 250 .

= 0,04 /.
.
. 7.38 -
-
h d 108 . h n R,
, ,
R. B p0 (

396
), C p ( -
), A pA. -
A -
, (7.28),
, (7.29),

pA = p0 + 2
------- . (7.31)
R
-
, (7.30) -

pA = p 2
------- . (7.32)
R
(7.31) (7.32)

p 2 2
------- = p0 + ------- ,
R R
,

p = p0 + 4
------- . (7.33)
R
(7.33) ,
p = 100 260 .
, 4 -
0,01% .

7.3.4.


-
.
-

( F ), -

( F ). F

F , , -
( ).

F F , , -
( -
). , , -
.

397
) ) -
,

, , -

. -
, -
-
. 7.39 -

( ) (. 7.39, ).
, (. 7.39, ).
= 0, ; = ,
.


, F < F , 

7.40, ; F > F 
7.40, .
,
A, (, , 
). 

F = F + F (. ).

, < --- () > --- ().
2 2

) )

. 7.40

,
. , , -
, , . , , -
-
,
.

398
, , ,
. ,
.

7.3.5.

() -
,
( capillaris, -
).
, , -
r. -
,
-
(. 7.41, ). ,

r
R = ------------
-. (7.34)
cos
= 0 ( ) -
, R = r.

) )

. 7.41

, -
(. 7.41, ).
r
- . = , , -
, R = ------------
cos
r
, R = ------------- (7.34)
cos
. , R = r
( = ).

399
(7.28)(7.30) (7.34), -

p = p0 + 2 2
------- = p0 + ------- cos, (7.35)
R R

p = p0 2 2
------- = p0 ------- cos. (7.36)
R R
p0 ( ) .

. -
h
(. 7.42, ). pA, pB, pC -
A, B, C, . , pA = p0 (p0 -
), pB = pC + gh. (7.36), pC = p0
2
------- . A B
R

Z; pA = pB , , p0 = (p0 2
------- ) +
R
+ gh. h :
2
h = -----------
-. (7.37)
gR
, h
(. 7.42, ). h -
(7.37). ,
A, B, C. , pA = p0 pB =
= pA + gh. (7.35), pC = p0 + 2
------- . pC = pB (-
R
C B

) )

. 7.42

400
Z), , , p0 + 2
------- = pA + gh = p0 + gh,
R
2
h = -----------
-.
gR
( -
), (7.37)
2
h = ---------
-. (7.38)
gr

= 1--- ghr. , -
2
()
, -
. -
.
. -
-
, -
. -
.
.
-
.
(. 7.43). -
, r = 1 ,
.
; -
. , -
. = 1000 /3,
= 0,07 /.
.
A . 7.43
pA = p0 2
------- , (1*)
r
B
pB = p0 + 2
------- , (2*)
r
p0 . (1*) (2*)
, pB pA = gh, h
. : gh = 2 2 4
------- + ------- , , h = ---------- =
r r gr
= 2,85 .

401
7.4.

7.4.1.


( ), . -
(), . . -
. -
. -
(,

).
, -
-
( ).
, , -
, .
( ) -
,
, (
, -
). -
,
.
-
, ; -
2.3.5. -
, . -

. -
, -
. -
. -
, , ,
, -
.

.
, -
.
, -
. ( , -
-

402
-
.) ,
-
.

.

7.4.2. .

,
, .

, .
-
. -
: -
,
, ,
.

.
, .

, -

v , -
. ,
, ,

v .
-
, . 7.44

v (. 7.44).
. -
; -
. , , -
, , -
.

, ( , -
) , -

. , -
.

403


v -
. -

. -
, -
. 7.45
, . .

v -
. , -

( !),
. -
. ,
.
7.45 -
, . -

.

, (.

7.46). , v
-
. ,

-
.

-
1 2 (. 7.47).
, -
. 7.46
-

. 1 v1 -
-
1, -
S1, 2 v2 2
S2. -
t 1
1S1v1t, 2
2S2v2t.
. 7.47

404
, ,
,
1S1v1 = 2S2v2. (7.39)
. -
, 1 = 2 (7.39) :
S1v1 = S2v2. (7.40)
, (
)
, . ,
, -
.

7.4.3.

-
; ,
() .
(
)
( : ).
, -

.
-
(. 7.48).
,
.
-
AB S1 CD
S2. -
AB
v1, CD v2.
, -
,
h1 h2.
t
-
-

AB CD. t . 7.48

405
,
, AB CD, . -
:
, , -
( AB CD). -
, . -
,
t.
, -
. -
, AB,
A1 = p1S1l1, (1*)
p1 AB, l1 = AA. ,
CD,
A2 = p2S2l2, (2*)
p2 CD, l2 = CC.
,
. -
A3.
(-
. 5.4.1), , -
,
K :
A1 + A2 + A3 = K. (3*)
(-
. 5.5.2), A3 -
U
: A3 = U. A3 (3*), :
A1 + A2 = K + U. (4*)
, ,
AB CD, ;
K + U (4*) -
,
AB AB, -
CD CD. ,

K = 1--- S2l2 v 2 1--- S1l1 v 1 ,


2 2
(5*)
2 2
U = S2l2gh2 S1l1gh1. (6*)

406
(1*), (2*), (5*), (6*) (4*), , l1 = v1t l2 =
= v2t, (7.40). -
(4*) :

p1 p2 = 1--- v 2 1--- v 1 + gh2 gh1.


2 2
2 2
,

p1 + 1--- v 1 + gh1 = p2 + 1--- v 2 + gh2.


2 2
(7.41)
2 2
,
. , 1--- v2
2
gh , , -
1),
:


-
.

(7.41)
. , -
, , . .
. ,
:

-
-
.

,
(7.41) :

p1 + 1--- v 1 = p2 + 1--- v 2 .
2 2
(7.42)
2 2

7.4.4.

,
. , -

1) -

407
, (
), .
-
. ,
, 7.49.
, -
:
, , -
, . ,

, .

-
(. 7.50).

, .

, ,
. 7.49 . 7.50 ,
, -
, ,
-
.
,
,
(. 7.51). . -
( ) -
. -
-
.
.

. 7.51 . 7.52

408

,
hA;
hB (. 7.52). (7.41)
, . -
: A ( ) B (
).
p0, B
, , . -
(7.41) :

p0 + 1--- v2 + ghA = p0 + ghB ,


2

v . ,

v = 2g ( h B h A ) . (7.43)

,
hB hA.

(. 7.53).
d = 4 , -
F = 30 ?
0 = 1000 /3.
. -
.

. 7.53

,
. A -
4F
p0 , F ( ---------
2
),
d
vA. B p0, -
vB.
4F
+ 1--- 0 v A = p0 + 1--- 0 v B .
2 2
p0 + ---------
2
(1*)
d 2 2

409
, S0vB = 1--- d2vA, S0
4
. -
, (1*) :
2
4F 1 2 4S 0
---------2 = --- 0 v B 1 ---------2- . (2*)
d 2 d

8F 2
4S0 n d2, (2*) : v B = ---------------
-.
2
0 d
L , -
= 45 .
(1.48) = 45, :
2
v 8F
L = ------B = -------------------
- , L = 4,9 .
2
g 0 gd

7.4.5.


; 1732 ,
-
.
7.54. -
U- ,
. -
, -
, . -
0,
.

. 7.54

410
A p ;
C p1.
p v
, BC -
, ( : -
C ):

p + 1--- v2 = p1. (1*)


2

pD pE
D E, . ,
pD = pC + gh1 = p1 + gh1,
pE = pA + gh2 + 0gh = p + gh2 + 0gh.
pD = pE, ,
p1 + gh1 = p + gh2 + 0gh.
, h1 h2 = h, -
:
p1 = p + (0 )gh. (2*)

(1*) (2*) , p + (0 )gh = p + 1--- v2. -


2
,

2 ( 0 )gh
v = -------------------------------- . (7.44)

h, v.

.

7.4.6.

S
-
, v.
.
1 2 (. 7.55). 1

v 1, 2 v 2 (v1 = v2 = v).

411
t -
1
Svt

P 1 = Svt v 1. -
2 ,


P 1 = Svt v 2.
,

P 2 P 1 = F t, (1*)

. 7.55
F ,

.
. -
. (1*),
:

F = Sv( v 2 v 1). (7.45)

,
-

F :

F = Sv( v 1 v 2). (7.46)

F .

7.55 F ; -

v = v 1 v 2.

v ,
2 2
, : (v)2 = v 1 + v 2
2v1v2 cos . , v1 = v2 = v3,
:
(v)2 = 2v2 (1 cos). (2*)
,

F = Sv 2 ( 1 cos ) . (7.47)

= 180: Fmax = 2Sv.


7.56.

412
(7.46) (7.47)
,
, -
-
S. -
( , , ) -
. ,
-
,
, - . 7.56
.

7.5.

7.5.1. ( )

; -

. ( , -
) -
, .
, , -
-
. -
-
.
, , -
-
.
1
2 S , z (. 7.57).

. 7.57

413

1 v 1, 2 -

v 2. , -
, v1 < v2. 1 2

F 12, 2 1 F 21.

, F 12 = F 21. F F 12 F 21.
,
F v
---- f ------- , (1*)
S z

v = v2 v1. F
---- -
S

, v
------- . -
z

,
. (1*) , -
, ,
.
, -
:
F v
---- = ------- . (7.48)
S z

,
(, ). -

=
--- , (7.49)

(7.48) ,


= .

-
; . -
, -
(: ). -
,

414
1 /, 1 ,
1 .

= ,

,

=
. .

, -
(: /2).
, 1 /2 = 1 = 1 /( ).

( )
20 :

1,1 103
1,6 103
0,25 103
1,5 105
1,8 105

, 20 60 -
.
-
() .
-
-
. (-
) , . -
, ,
,
( -
).
( -
), -
, -
-
.
, , , -
,

. -

415
) )

. 7.58

,
. -
7.58, , (-
, ). B -
A . -
7.58, (,
). B -
A,

.

7.5.2. .

-
, , -
. (
lamina, ). -
. -
()
,

.

-
( turbulentus, -
, ).
-
. ,
, , .

416
, -
-
:

Re = vl
--------- , (7.50)

, v
, l -
(, ), .
Re . -
2000, .
2000, -
.
7.59 -
. -
7.59, .
.

, . 7.59,
. -
, -
.
, -

. 7.59,
. -
.

, -
. ,
(
), ,

.

) ) )

. 7.59

417
7.5.3.

, , -
( -
). ,
, : -
.
, .
, , -
, .
, -
.
XVIII . '
( '). -
. ,

.
-
XX -
. .


l
d = -----------
-, (7.51)
Re
l . , ,
l = 0,1 , Re = 1000, d = 3 ; Re = 10 000
1 . ,

. -
.

, .
, -
, .

. -
.
, -
,
. -
( ),
, .

418
,
-
-
; -
, -
.
- . 7.60
.
7.60 -
. A B -
. -
, ,
. -
.
() ,
.

,
. 2.3.5.
v ( , -
) (. (2.18)), -
v2 (.
(2.20)).

7.5.4.

, -
, , ,
, -
.
7.61 .
,

. 7.61

419
. ( , -
-
.)
,
. . -
, -

. -
,
, -
( ). -
.

-
.
() - ( -
, - ). -
, ,
. , -
. -
. . -
( -
)
. -
.
-
. , -

, -
.
.
? , ?
,
?
, ,
. -
, -
, ,
, -
, -
. , -
, , , ,

420
. -

. -
,
.
, , -
-
, . -
, -
. v + u,
v u, u
(). -

, . .
. p1 -
, p2 -
:

p1 + 1--- (v + u)2 = p2 + 1--- (v u)2.


2 2
p:

p = p1 p2 = 1--- [(v + u)2 (v u)2] = 2uv. (7.52)


2
, ,
F = pS = 2uvS, (7.53)
S .

, p, -
, , ,
-62?
m = 200 ,
2S = 500 2, p =
mg
= --------- = 4 103 . ,
2S
4% , . . -
.
u ?
p
(7.52), u = ---------- . p = 4 103 , v =
2v
= 1000 / = 280 / = 1 /3, u = 7 /.
, -
, .

421
7.5.5.

, (
* ) , (.
7.62). v ,
u .

(. ). ,
, ;
v + u,
v u. ,

, , F , -
( ).
. -
.

. 7.62

, ,
-
() .
. -
-
,
, -
.

. ,
-
, -
, , .
-
.

422

7.1. : - 7.7. -
.
? -
7.2. S = 8 2 m = 272
h. .
- . -
, = 13 600 /3.
7.8.
a, . -
? 1 -
7.3. h1 = 48
, l, 1 = 900 /3,
2
. h2 = 20 -
-
, - 2 = 800 /3.

- . -
? = 13 600 /3.
7.4. - 7.9. , -
-
, ,
F = 6 ? - , p0 = 105
S = 3 2, - = 1,2 /3, , -
S0 = 2 2. -
,
.
7.5. , .
45 - 7.10.
. -
- ?
- ?
? 7.11. 1 -
7.6.
- 2 (2 > 1).
. -
- ?
H. - 7.12.
,
? , ,
0. V. -

423
- 7.19.
. - P = 5 , -
, P1 = 4,55 . -
. -
. - = 7800 /3.
, 7.20. -
- ,
, - V = 200 3, ,
. - . -
0. ?
7.13. - = 8900 /3,
0 = 1000 /3.

.
. ? ,
7.14. - V V -
n = 3 , , , V V,
. - .
. 7.21.
0 = 1000 /3; R -
. r, -
7.15. , 1,
V = 250 3 2. -
. -
. , -
= 900 /3, -
0 = 1030 /3. ,
-
7.16. V = 0,25 3
?
, - . -
2/3 .
-
, - ; ,
. - -
= 800 /3, - .
7.22. -
0 = 1000 /3.
1
7.17. 2 , -
. 2.
? 1 -
1 = 7900 /3, - 2?
2 = 13 600 /3. 7.23.
7.18. , R -
- r P
n = 1,5 P1 .
. -
. ,

424
, - -
? -
7.24. ?
- 7.29. -
. R = 3 -
, .
?
7.25. -

- ? -
. = 0,47 /2.
, 7.30. -
? ,
. , D = 12 ?
-
. = 0,04 /2.
7.26. h = 0,6 7.31. -
S = 8 2 (-
. - ), -
, - h = 10
? -
= 900 /3, - ?
0 = 1000 /3. = 0,073 /,
. F = 1000 /3.
, 7.32. -
; -
,
r = 0,25 -
Fmax

. -
, -
-
F, . . .
F -
max
------------- . , -
2

7.27. .
3,2 , 3,8 , -
2,9 . (

, ),
. R
2600 /3, . -
1000 /3.
7.28. - r,
- -
-
- -
. . :

425
) R , R = 1737 ,

- . -
h1 = 2 ; ,
) R , . -
= 0,073 /, -
h2 = 4 ; 0 = 1000 /3,
) , -
, - G = 6,67 1011 2/2.
. 7.34.
- ; v, -
= 0,073 /, p.
0 = 1000 /3.
7.33. , - -
, ,
2 , .
r = 0,36
h = 25 . . 0.
.

426
8

8.1.

8.1.1.

, -
, . . .
-
; -
. , -
, .
.

( ,
).
-
, .
xm .
x(t) t
, .
x(t) :
x(t + n) = x(t), (8.1)
, n . -
,

x(t) = h(t)g(t), (8.2)
h(t) , g(t) -
. ,
(8.2), , .
xm

427
h(t); ,
-
xm g(t) .

, ; -
, , -
, ,
( -
. 8.1.5.)
, -
( , ),
: , -
, .

8.1.2. ()

, , ,
, , -
,
- -
.
, -
. ,
, -
.
. -
, , .
,
,
, . , , -
,
, .
, -
(
). -
,
.
. , . .
, ,
.
; -
.
.

428
, -
, -
-
, .
,
-
, .
. .
.
. .

, (. 8.1).
-

. F


F F (F = F cos;

F = F sin). T F

, F -
, ,
A.
, -
A, . 8.1
-
. AA
; , xm = 1--- AA = = l sin, l
2
.
-
, , -
, .
, -
-
. , -
, ,
, ,
.

8.1.3.

,
,

429
, -
,
.
.
, , -
,
. -
-
.
, . -
,
,
, . . - -
.
. -

, , -
,
. , -
, 0. -
,
0. 8.5, -
, -
= 0. -
-
, .

,
.
, (, )
, .
; -
, -
.
( ,

); -
.
,

(. (8.17)). -
,
, -

430
, ,
,
().

8.1.4.

-

, -
. -
, , , -
.
-
(, -
), , -

(. 8.2).
, -
.
-
.

. 8.2

-
(. 8.3).
( -
), -
,
.
.
,
.
-
. -
. ,
, . 8.3

431
A . -
A -
, B -
, -
. ,
,
. -
, -
( ). -
1657
.
-
, ,
-
, .
,
,
-
. , -

,
( ,
).
.
, , , ,
, , -
, , , -
, , ,
.

8.1.5.

, -
* , , , .
8.4, x t
( , ) :
) () ,
) ,
) ,
) ,
) ,
) .
xm .

432
) )

) )

. 8.4

, -
. ,
. -
, , -
. -
.
(. 8.4, ) -
(. 8.4, ). ,
( ), -
. -
;
, -
. , ) (8.2)
:
x(t) = xm(sin t)et. (8.3)

2
= ------ , , , -

( ).
n , (. .
). 8.4, .
.
.
, 8.4,
.

433
8.2.

8.2.1.

,
x t :

x = xmsin 2 -- + 0 .
t
(8.4)

xm , (-
), 0 -
. 2 -t- + 0 .

= 1--- , ,

. -
() :
= 2 = 2
------ . (8.5)

(8.5) (8.4) :
x = xmsin(t + 0). (8.6)
8.5, -
. -

. 8.5

434
t.
( , ) -
, , t- -
2.
0. , -
( t = 0) ;
x(0) = xm sin0 = 0. , 0 = 0 , ,
x = xmsint. (8.7)
8.5, .

.
,
t = 0,
.
, t = 0 -
, . . : x(0) = xm. ,
0 = --- , ,
2

x = xmsin t + --- = xmcos t. (8.8)


2
8.5, .
,
.
, -
t = 0 ,
.
, (8.6),
,
t = 0
.
, 0 -
t = 0. ,
t = 0, -
, 0 -
( ,
, ).
8.5, , , -
t-. , -
--- . ,
2

--- .
2

435
1
= --- ,


1
= .


(: 1)
.
, 1 -
.
()
(: 1). -
1 ,
, 2 -
.

8.2.2.

, -
xm. -
-
; , ,
v = xm a = 2xm. (8.9)

, x- (. 8.6).

. 8.6

436
x-
, , xm
.

2x m 2
-------------- = ------ = .
v

x-; .
, t = 0 -
A (. ); x- -
x = 0, . . -
. t ,
, M. AM -
, t. -
- t. ,
x-

x(t) = xmsint.
, (8.7) -
.
: 8.6 -
v(t) a(t) -
, (8.7). -

x- v a -
,
t , . -

v (t) a(t) , -
. ,
(8.9),

v (t) = xm cost, (8.10)

a (t) = 2xm sint. (8.11)
, -
xm,
x- xm
. x-
AO , A
t = 0, 0
x- .

437
8.2.3. , ,

(8.7), (8.10), (8.11), -


,
xm, 0 = 0.
8.7. -
. t- 0 2
8.6, xm. M, B,
C, D, , t-
8.7, . ,
8.6. 8.7
.

. 8.7

-, ,

--- ,
2
( . -
).
, , -
0. 8.7 ,
0 = 0. , 0 0, -
t- 0. -
(8.7), (8.10) (8.11) 0.

438
, x(t), v(t), a(t) -
:
x(t) = xmsin(t + 0), (. (8.6))
v(t) = xmcos(t + 0), (8.12)
a(t) = 2xmsin(t + 0). (8.13)

(1.8), t
t :
x ( t + t ) x ( t )
v(t) = ---------------------------------------- . (1*)
t
(1.15), t -
t :
v ( t + t ) v ( t )
a(t) = ----------------------------------------- . (2*)
t
(1*) (2*)
t 0.
x(t) = xmsint (1*) , sin( + ) = sincos +
+ cos sin ,
v ( t )t
---------------- = sin( t + t) sint =
xm
= sin tcos t + cos tsin t sin t.
t
cost = 1; sint = t. (3*)
v ( t )t
, ---------------- = t cos t. , ,
xm
(8.10), -
.
(8.10) (2*) , cos( + ) = coscos sinsin,

a ( t )t
--------------- = cos(t + t) cost =
x m
= costcost sint sint cos t.
a ( t )t
(3*), , --------------- = t sin t, . .
x m
(8.11).
, , (8.7) (8.6), (8.10) (8.11)
( (8.12) (8.13)) ,
x- ,
, -
(1.8) (1.15).

439
8.2.4.

-
x0 v0 t = 0. (8.6) ,
x0 = xmsin0, (8.14)
(8.12)
v0 = xmcos0. (8.15)
(8.14) (8.15),
x
tg0 = ---------0- . (8.16)
v0
x0 = 0, tg0 = 0 , , 0 = 0 (,
t = 0). v0 = 0, tg0 =
, , 0 = --- (, -
2
t = 0 ).
x v
(8.14) (8.15) : sin0 = ------0- cos0 = -----------
0
-.
xm x m
,
2
21 v
sin2 0 + cos2 0 = ------
2
- x 0 + ------0 .
2
xm

sin2 0 + cos2 0 = 1, -
:
2
2 v
xm = x 0 + ------0 . (8.17)
2

v
x0 = 0, xm = -----0- . v0 = 0, xm = x0.

8.3.

8.3.1.

(8.6) -
. -
.

440
, -
, , -
.

, -
. ,
. , -

. R -
, , Rx
.
. -
:
Rx = x, (8.18)
, -
. -
, Rx , -
. , ,
, -
.
m ,

Rx = max, (8.19)
ax .
(8.19) (8.18),

ax = ----
- x.
m

, x, ax
! . -
( ), (-
) ,
,
. , , -
,


ax(t) = ----
- x(t). (8.20)
m
(8.20)

(8.6), x(t) = xm sin(t + 0) , (1.8)

441
(1.15), ax(t) = 2xm sin(t + 0). -
:

= ----- , (8.21)
m
xm 0
( , , -
). (8.21), ,
-
,
ax(t) = 2x(t). (8.22)

8.3.2.

: -
1 , -
, 2 m, 3,
k (. 8.8). O
; .
A B , -
, -
; OA = OB = xm, xm . -
, -

, x. F

N , R

F , -

x . (2.3),

. 8.8

442

F = k
x . R = F , (8.18)
:
Rx = kx, (8.23)
(8.22), -
:
k
= ----- . (8.24)
m
(8.5), :
m
= 2 ----- . (8.25)
k
, m -
, -
k (. 8.9). -

F = m g -

F = kX , X
M, -
(. -
). ,
,

R = m g + F = m g kX .
x-,
Rx = mg kX. (1*)
. 8.9
x, (8.18), -
O -
,
, x0 -
. ,
x = X x0. (2*)
x0 :
kx0 = mg. (3*)
(2*) (3*) (1*) :
Rx = mg k(x0 + x) = kx.
(8.23), , (8.24) (8.25).
, 8.8, -
, 8.9
.

443

(8.24) ( (8.25)). -

x0 . , -
, -
.
) 1 (. 8.10, ).
m, -
, -

) k1 k2.
?
Rx -
. 8.10
, ,
Rx = (k1x + k2x), x
(, ).
, -

k = k1 + k2. (8.26, )
,
m
= 2 ------------------ .
k1 + k2

2 (. 8.10, ). m,
, -
1 k1 2 k2.
?
x
x = x1 + x2, (4*)
x1 x2 1 2, -
. 1 ,
2. , -

R = k2x2. (5*)
, , 2 -
1, , 1
2. ,
k2x2= k1x1. (6*)

444
(4*) (5*),
R
----- = x x1.
k2

(6*) :
R R
----- = x ----- ,
k2 k1

,
k k
1 2
R = -----------------
- x.
k1 + k2

, 2,
1, ,

k k
1 2
k = -----------------
-. (8.26, )
k1 + k2

( k + k )m
= 2 ---------------------------
1 2
-.
k1 k2

, 1 2 (k1 = k2),
k
(8.26, ) : k = -----1 .
2
2m
= 2 -------- . ,
k1
,
2 .

8.3.3.

m,
l.
, , -
, -
. -
.

445
- -
8.11,
,
t,
M;
. -

F T .

F

F(t), ,

F(t),

; F(t) = F cos(t);

. 8.11 F(t) = F sin(t). -

F(t) . -
:
F ( t )
- = g sin (t).

a(t) = ----------------- (8.27)
m
, , > 0 (

), F . A
A . -
T = F, M: T > F. (,
T F, = ma, a
M.)
, :
n 1. (8.28)
, -
57, (8.28) , -
( 8.11
).
(8.28) , sin d ;
(8.27) :
s(t )
a t d g(t) = g ---------
-, (8.29)
l
s(t) OM. (8.29) ,
(8.28)


g
= --- . (8.30)
l

446
, (8.30) (8.29)

a(t) = 2s(t), (8.31)


(8.22) -
, . ,
ax x,
a s ,
n 1
.
, n 1 -
g
= --- .
l
(8.5), ,
l
= 2 --- . (8.32)
g

: -
! -
, , , -
, (
, -
).

-
. , ,
. -
, (8.28) -

J
= 2 ----------- , (8.33)
mgl
J , m , l
. (8.33)
:

L
= 2 ---- . (8.34)
g

J
L = ------- .
ml
, ,
.

447
8.3.4.


* , 8.12. l
A
. m, -
,
k, -
. -
.
(8.28),
-
-.
, -
; -
. 8.12

2 g
1 = --- . (1*)
l

, , , -
, , 8.8. -
;
(8.24) ( (8.26, )).

2 2k
2 = ------- . (2*)
m

(1*) ,
( (8.27) (8.29)) :

m gx
R1 = ------------- ,
l

x . (2*)
, , (8.23),
;
R2 = 2kx.

m gx
R1 + R2 = ------------- 2kx.
l

g 2k
ax = --- + ------- x. (3*)
l m

448
(3*) (8.22), , -
:
g k
2 = --- + 2 ----- . (4*)
l m
, -
1 2:
2 2
2 = 1 + 2 . (8.35)

(8.35) -
: , -
, -
, -
.

8.3.5.

, - m l
, -
() -

a . ,
2.5, , -

F = m a .

F , ; -
, -

F
------ = g,
m

F +F

g = ------------------- = g a, (8.36)
m
. ,
(8.34)
l
= 2 ---------------
- . (8.37)
ga

l .

a , , | g a | = g + a.
-
, (8.37),
l
= 2 ------------- . (8.38)
g+a

449

a , -

, g,

l
= 2 ------------- . (8.39)
ga

a = g, . . , -
.

: , -
! (
), .
-
, ,
.



a , 

g ,
, .
 ; 
...

1. , v = 16 /,
.
, t = 4 . -
l = 1,8 .

?

a = v
--- .
t
. , (8.38),

lt
= 2 ---------------- ; = 3,5 .
gt v
( ) -
l
= 2 --- = 2,7 . , -
g
, , -
0,8 .

450
2 (. 8.13, ). l
,
. -
-
,
.

) )

. 8.13


, -
,
a = gsin (1*)
--- g

2
(. 8.13, ). ,
2
g = | g a | = acos 2 ( g a sin ) . (2*)
(1*) (2*),

g = | g a | = gcos. (3*)
(3*) (8.37), :
l
= 2 ----------------- . (8.40)
g cos
(1*) (3*) ;
g2 + a2 = g2(cos2 + sin2 ) = g2.
, ,
8.13, , . ,

g . -
, -
.

451
8.4.

8.4.1.

-
, -
(, , -
, , -
).
,
, -
.
(8.6) (8.12) , -
t,
x(t) = 0, . . .
sin(t + 0) = 0 , , |cos(t + 0)| = 1.
(8.12), ,
Km = 1--- m2 x m ,
2
2
, (8.24), :

Km = 1--- k xm .
2
(8.41)
2

. ,
, (8.41),
W. ,

W = 1--- k x m .
2
(8.42)
2
(8.6) (8.12)
x(t)
---------- = sin(t + 0),
xm

v(t)
------------ = cos(t + 0).
x m

; :
2 2
x (t) v (t)
------------
2
- + --------------
2 2
= 1.
xm xm

452
(8.24) (8.42) :
1 1
--- kx2(t) + --- mv2(t) = W. (8.43)
2 2
(8.43) -
U(t), ,
K(t). (8.43) -
:
U(t) + K(t) = W, (8.44)

U(t) = 1--- kx2(t), (8.45)


2

K(t) = 1--- mv2(t). (8.46)


2
, , 0 = 0 -
:
sin2 = 1--- (1 cos2); cos2 = 1--- (1 + cos2), (8.47)
2 2
(8.45) (8.46) :
U(t) = Wsin2 (t) = 1--- W(1 cos2t); (8.48)
2

K(t) = Wcos2 (t) = 1--- W(1 + cos2t). (8.49)


2
8.14.

.

. 8.14

453
U(t) K(t)
-
( -
) . t = 0
(0 = 0), -
-
2 3 n
---
- , ------ , ------ , , ------- , , -


------- 3 5
, ------- , ------- , , (------------------------
2n 1 )
- , ,
2 2 2 2

3 5 ( 2n 1 )
------- , ------- , ------- , , ------------------------- , ( n = 1, 2, 3, ...).
4 4 4 4

8.4.2.

( m l)
m ,
. -
W,
:
U() + K() = W. (8.50)
U() K() -
M,
(. 8.15, ). -
( , -
= 0),
U() = mgh() = mgl(1 cos). (8.51)

) )

. 8.15

454

A:
W = mghm = mgl(1 cosm). (8.52)
(8.51) (8.52) (8.50),
K() = mgl(cos cosm). (8.53)
8.15,
U() ( 1) K() ( 2). -
.
-
U() K() m = --- (, 3 4). -
2
, 2 4 mgl cos m. (-
, 3 4 (. . 5.34).)

.
m = 60 .
-
, -
?
(8.51) (8.53), U() = K()
: 1 cos = cos cosm, . -
,
cos = 1--- (1 + cosm). (8.54)
2

cosm = cos60 = 1--- , , , cos = 3--- . -


2 4
, : = 41 24.
, m = --- , (8.54) , = ---
2 3
(. . 5.34).
n 1, -
. . (8.51)
U() = 2mgl sin2
--- . (1*)
2
2
n 1 sin2
--- = ------ . -
2 4
s = l, (1*)
1 mg
U = 1--- mgl2 = --- --------- s2. (2*)
2 2 l

455
(8.24) (8.30) (2*)
U = 1--- ks2, (8.45) -
2
.

8.4.3.

, ,
( , -
) , -
. T()
M , -
(. 8.16, ).
T() mgcos = ma, (1*)
a -
2
v
A. a = -----
- , l , v -
l
M, (1*)
()
T() mgcos = 2K
----------------- , (2*)
l
K() M. -
(8.53) (2*),
T() = mg(3cos 2cosm), (8.55)
m . T() -
8.16, . -
T() , m = --- .
2

) )

. 8.16

456
.
,
n = 2,5 -
?
(8.55), = 0
T(0) = nmg, nmg = mg(3 2cosm). ,
n
cosm = 3------------ = 1--- . -
2 4
, m = 75 30.

8.5.

8.5.1.


m, -
x- k
(. . 8.8). , -

F(t) = kx(t). (8.56)
, (-
), -
-

k
0 = ----- . (8.57)
m
x(t) v(t) ( -
, x-) -
:
x(t) = xmsin(0t + 0); (8.58)
v(t) = vmcos(0t + 0) = 0xmcos(0t + 0), (8.59)
0 .
,
,
(8.56), F(t),
v(t) :
F(t) = v(t). (8.60)

457
(8.60) , -
.
-
. ,
-
.  -
(/).

= -------- (8.61)
2m
.
Q, ,
kxm
vm = 0xm, :
kx m k mk 0
- = ---------- = ------------- = ------ .
Q = ----------------- (8.62)
0 x m 0 2
x, -
. ( = = 0) -
.

, -
0 Q, -
xm :
xm(t) = xm(0)e0t/(2Q) = xm(0)et. (8.63)
, 8.17.
(8.63) , 1/ : ,
e d 2,7 .

. 8.17

458
8.5.2.

,
.

k
0 = ----- , -
m
.
, -

F(t) = sint, (8.64)


Fm , .

( ) .

x(t) = xmsin(t + ), (8.65)
xm () -
. (8.65) (8.58),
, (8.58), xm , -
; -
, -
Fm ,
F(t). (8.65) , -
x(t)
F(t). (8.65), , ,
. 8.2.3, v(t) a(t) , -
:
v(t) = xmcos(t + ), (8.66)
a(t) = 2xmsin(t + ). (8.67)

:
(8.64), (8.56) (8.60). -
, -
:
Fmsint kx(t) v(t) = ma(t). (8.68)
(8.65) (8.67) (8.57), -
:
Fmsint xmcos(t + ) =
2
= ( 0 2)mxmsin(t + ). (8.69)

459
8.5.3.
,

, -
, = 0. -
, , , -

0 ,
. , ,
0, -
.
, = 0.
(8.69)
Fmsint = (02 2)mxmsin(t + ). (8.70)
xm . -
(8.70) t.
: ) = 0 ( 2, 3, ), . .
: ) = ( -
, 3, 5, ), . . -
, , -
.
) sint = sin(t + ), (8.70)
Fm
xm = -----------------------------
2 2
-. (8.71)
( 0 )m

) sint = sin(t + ), (8.70) -


:
Fm
xm = -----------------------------
2 2
-. (8.72)
( 0 )m

xm j 0, , x(t) F(t) -
< 0 ( -
(8.71)), x(t) F(t) -
> 0 (
(8.72)).
8.18,
xm(), (8.71) (8.72). ,
F F
2
m
- = -------- ,
0 xm ----------- m

0 m k

460
)

. 8.18

xm . 8.18,
< 0 > 0. < 0, = 0; > 0,
= .

8.5.4.

-
, . . ,
= 0, (8.73)
, . ,
xm
. = 0
(8.71) (8.72) , .
, -
, (
, Q -
).
xm(0) (0) -
, -
= 0. (8.69) -
,
Fmsin0t = 0 xm(0) cos(0t + (0)). (8.74)

461
(8.74) t.
,

(0) = --- ( 2n --- , n = 1, 2, 3, ...), (8.75)


2 2

. . ,
--- . -
2
(8.75), sin 0t = cos (0t + (0)),
(8.74)
:

m F
xm(0) = ---------
-. (8.76)
0

(8.62) :
QFm m 0 F
xm(0) = ------------
- = --------------- . (8.77)
k 2k

-
, xm(0)
. xm(0) , -

,
. -
(8.75). . 8.2.3,
---
2
. , (8.75)
: = 0 -
. -
,

v (t) F (t) (, , -
,
). t -

F (t) v (t), -
- ,
, ,
.
-
-
.

462
, -
. -
, , -
, , . . -
, .
-

.
-

.
.
, -
(, ) -
(
, . 4.4.3).

-
, -
.
, -
.
( -
) ,
.
,
1906 .
. -
, , -
.

, .

8.5.5.

xm -

( ),
:
) ,
) ,
(8.73).

463
(8.69) , -
xm() ():

Fm
xm = ----------------------------------------------------------
-; (8.78)
2 2 2 2 2 2
( 0 ) m +


tg = -----------------------------
2 2
-. (8.79)
( 0 )m

, ,
= 0 0 (8.78) (8.71) (8.72),
0 = 0 (8.77). ,
0 = 0 (8.79) = --- .
2
xm(), (8.78), -
8.19, .
1 -
( 1 ), 2
| 2 > 1 |, 3 -
| 3 > 2 |. -
.
xm() ,

. 8.19

464
( -
); -
8.18, .

: -
! = 0
F
- . = 0
m
xm(0) = ---------
0
. -
,
; .
( ), -
0;

2
2
= 0 ----------- .
2
(8.80)
2m

= (8.78), :

m F
xm() = -------------------------------- . (8.81)
2
2
0 -----------
2
4m

xm(0) < xm().


8.19, -
(8.79) () : 1, 2, 3.
1 < 2 < 3. 0 0,
= --- = 0.
2

-
* -
: -
(,
, )
. , -
, -
. 8.20 -

.
1, -

465
. 8.20

. -
: 1, 2, 3, , 9.

8.5.6.

* , , -
, . -
, 0 -
. , -
.
, -
. (. (8.63)), -
et, ,
0
Q = ------- (. (8.62)).
2Q

, . ,
T -
, . .
1 2Q
T d --- = ------- . (8.82)
0
Q , -
Fm QF m
xm(0). , (8.76), --------- = -----------
-.
0 k
(8.82), ,
xm ( 0) Fm 0
------------------ -------------- .
T 2k

466
,
-
. , -
. -
, ,
. -
, -
.
(8.82) , -
, . ,
, e d 2,7 .

8.6.

8.6.1.

m F1 = k1x
F2 = k2x. -
x1 = x1msin (1t + 1)
k1 k2
x2 = x2msin (2t + 2), 1 = ----- , 2 = ----- . -
m m
, -
.
-
. , k1 = k2 = k
, , 1 = 2 = . -
.
Xm .
,
, :
x1 = x1msin(t + 1) x2 = x2msin(t + 2). (8.83)

X = Xmsin(t + ). (8.84)
( x-); ,

X = x1+ x2.
(8.83) (8.84),
Xmsin(t + ) = x1msin(t + 1) + x2msin(t + 2).

467
t. t = 0

Xmsin = x1msin1 + x2msin2, (1*)

t = ------- :
2

Xmcos = x1mcos1 + x2mcos2. (2*)

(1*) (2*),

1m x sin + x sin
1 2m 2
tg = -----------------------------------------------------------
-. (8.85)
x 1m cos 1 + x 2m cos 2

(1*) (2*) ,
2 2 2
X m = x 1m + x2m + 2x1mx2m(cos 2 cos 1 + sin 2 sin 1).


cos( ) = cos cos + sin sin, (8.86)

2 2 2
Xm = x 1m + x 2m + 2x1mx2m cos(2 2). (8.87)

.
1: 2 1 = 2n, n (. 8.21, ).
cos(1 2) = 1.

. 8.21

468
(8.85) (8.87) ,
= 0; Xm = |x1m + x2m |. (8.88)
2: 2 1 = (2n + 1) (. 8.21, ).
cos(2 1) = 1. (8.85) (8.87) ,
= 0; Xm = |x1m x2m|. (8.89)

3: x1m = x2m = xm, 2 1 = --- + 2n (. 8.21, ).


2
cos(2 1) = 0. (8.85) (8.87) ,

= --- ; Xm = 2 xm. (8.90)


4

8.6.2.
()

-
,

x1(t) = xmsin1t x2(t) = xmsin2t.

1 = + , 2 = , n , (8.91)

sin + sin = 2sin +


------------- cos ------------- , (8.92)
2 2

1 2 + 1 2
X(t) = x1(t) + x2(t) = 2xmsin ------------------- t cos ------------------- t.
2 2
(8.91)
X(t) = 2xmsint cos(t). (8.93)
X(t) 8.22.
.
1 2 ,

= 2
------ (8.94)

469
. 8.22


(t) = 2xm|cos(t)|. (8.95)
.
, -
, .
;

0 = -------
-. (8.96)

1 1 2
= ----
- = ------------------- (8.97)
0 2
.
-
. -
, ,
.

8.6.3.

-
: 1 = , 2 = 2,
3 = 3. , ,
x1m, x2m, x3m,
. X(t)
:
X(t) = x1(t) + x2(t) + x3(t) =
= x1msint + x2msin2t + x3msin3t. (8.98)

470
-
, ; 2
, ;
3 ( ).
x1(t), x2(t), x3(t)
( x-),
X(t) . -
, -
. 8.23, -
(8.98). -
( 1
, 2 , 3
).

)
)

. 8.23

,
! (), --
X(t)
. 2
------

( ). , -
-
; -
. 8.23 -
= 2
------

: , 2, 3. 8.23, -
(8.98).
j xjm
(j = 1, 2, 3).

471
8.6.4.

XIX
, f(t)
, , -
, :
f(t) = a0 + a1sin(t + 1) + a2sin(2t + 2) +
+ a3sin(3t + 3) + , (8.99)

= 2
------ . (8.99)

f(t) , , . -
(8.99) -
, -

( ).
X(t), -
(. 8.24, ).
, -
8.24, , () ,
8.24, . ,
-
X(t).
X(t) 8.24 8.23, -
. f(t) -
(8.99)

. 8.24

472
. , f(t)
, -
.
,
() -
.
, ,
0 , .
,
f(t) . -
, -

1 = 2 4 6 8
------ , 2 = 21 = ------ , 3 = 31 = ------ , 4 = 41 = ------ , .

-
, -
(
). , f(t) -
, -
1, 2, 3, 4, 0
.
( -
,
0).
, ,
.

8.25
X(t)
-
( ). . 8.25
-
, , ,
. , , ,
, , , -
(8.42), ,
. -
. -

. , -
, -
.

473

8.1. 8.6. -
- -
. 1 -
5 , 2 = 2 1 xm =
8 . - = 20 .
. : ) -
- , ) -
? 10
8.2. - ?
8.7. -
. -
, - ?
, - 8.8. ,
,
-
?
-
8.3. - -

-
, , -

-
, -
30 ,
?
2

-
, -
: ) .
, ) , ) 8.9. -
- , -
? -
8.4. v(t) -
- .
- 8.10.
, ,
-
? -
8.5. - , -
: -
20 .
x(t) = xmsin1t cos2t + 8.11. t -
+ xmsin2t cos1t. m, -
-
,

, v(t) = vmcos t + --- .
, 6
, - : )
. t = 0; ) ;

474
) - 8.17. ,
t; ) - ,
t = 0. - ,
,
. m,
8.12. - ,
, - ?
, : ) 8.18. m
- -
; ) - -
. k .
8.13. - .
m = 60 - 8.19. m
x(t) = xmsin(t + 0), -
k
xm = 0,6 , = 1,5 1, 0 = --- . . -
4
: ) - .
; ) 8.20. -
; ) -
. = 2,46 -
8.14. - . -
, R = 6370 ,
, 2,4 , n = 81
5 , - .
.
--- . , - 8.21.
2
1,2 - -
: )
, ) , ) - h = 50
. , -
8.15. , - -
- l0 = 4 ,
k, , - ?
m. R = 6370 .
- 8.22.

? h = 2
8.16. - . -
, . .
m, - T = 8,64 104 ?
- R = 6370 .
- .
k1, k2 0
T 1 ----- , 0 1 -
(. . 8.25). 1
.

475
h, - = 0gSx, S
. . F
8.23. - x, -
- k = 0gS.
l, 8.25. -
, - -
a?
8.24. -
h = 12 ,
= 650 /3 . -
(. 8.26).
, l.
- , ,
- .
, . .
- :
. - mg
k = --------- , m .
0 = 1000 /3. - l
. 8.26.
-
, -
-
xm
,
: ) -
, ) -
. 8.26

. , --- .
2
F -
,
-
x: F = .

476
9

9.1.

9.1.1.


, , . -
, .
, -
,
, ,
-
, , ,

, ,
. .
-
( ) -
, -
. -


. ,
, .
, -
,
.
(),
, -

. , -

477
, -
, , , -
, ,
.
,

105 ( ).
()
.
( -
,
),
. ,
,
, , , -
, -
, . -
, -
.
-
.

, -
, , , -
, , . . -
-
.
(, )
; .
-
.
, ,
. , -
.
() .
.
, -
.

9.1.2.

: -

478
.
.
, -
? ?
-
,
(. 9.1, ). 1 2,
1,
12 . 2 -
3.
.
4,
5.

. 9.1


9.1, .
O .
v , -
-
(, -
).
(. 9.1, ).
, ,
:
= v. (9.1)
d 0,1 , d 2 , v d 0,2 /.

479
, -
! O . -
, ,
, . ,
-
,
,
.
-
(
, ). -
; -
v , , -
.
; -
, -
. , -
,
,
, . .
.
,
( ) ,
.

v v . -

. v v -
: -
,
. .
, -

, .

v v -
;

. -
( -
). .
, -
, .
,
.

480
, -
. -
.
, -
, , ,
, , -
. -
;
. -

.

------ .
v

9.1.3. :

, , -
, , (. 9.2, ).
, -
, . -
, , .
,
,

. , , -
, .
.
,
, -
. -
: , , -

) )

. 9.2

481
(
, ). -
, ,
.
,
, , -
, . -

, ,
(. 9.2, ). , , -
. , , -
, ,
. -
, -
() -
. ,
.

, , -
, ( -
-
). -
. ,
-
. , -
, -
,
.
. -
, .
-
9.2, ,
, -
, .
-
, -
, -

, . . , -
, .

, -
.
.

482
9.1.4.

,
, -
, .
,
, -
.
,
(. 9.3).
:
,
.

. 9.3

, -
, , ,
. , -
x-. -
( )
, , -. , 8
x ,
x-
. 9.3, x-
-. -, x-, -
, .
1 -
(. . ), -
1 2 , 2 -
, -. -
2 1. -
,
, .

483
1 -
( 1
). 2 -
1 ,
. 3
2 ,
1, .
v.
, 9.4, -
, ( ,
). -
17 , 9.3. -
v
- = ------ .
x- ---------
16 16
( 17 ; , ,
v
- = --- , -
---------
n n
n + 1 .)
9.4 t = 0.
1
(. ). -
t = --- . 1
4

. 9.4

484
m, . -
2, 3, 4, 5 t = --- (
4

). t = --- , -
2

t = 3
------ , t = . ,
4
, ,
= v.
9.4 ,

(
). -
( ), -
, , , -
2n, n . v
(
), x- -
( ). , -
.
, -
! -
: 1) ( -
-; 2)

( x-). 9.5
t.

x-. , t + t , -
,
t x = vt.

. 9.5

485
. 9.6

, -
, -
. ,
x- (. 9.6). -
1 ,
. , -

( -
). , -
9.4, , - ( , -
) 90 -
x-.
x-, -
, v.

9.1.5. ;

,
k
.
: (2.13)
k:
F = kl,
,
:

U = 1--- kl2, (9.2)


2
(5.74),
. l (2.13) (9.2) -
( ) (-
).

486
,

. -
:

-
-
.

:
p = Z, (9.3)
p , -
, Z ,
. , -
( ). -
(/2), . .
(). , -
.
-
, -
() .
-
9.7, , )

. l, -
S = l2. AD -
, BC -

F , -

() .
)
-
:

p = F
---- . (1*)
S

F -
. 9.7
CC = l.

= l
------ (2*)
l

(
) . Z -

487
,
; E.
(1*) (2*) (9.3), -
():
F l
---- = E ------ . (9.4)
S l
(9.4) (2.13) , -
l F = F, -
,
k,
:

k = ES
-------- . (9.5)
l
(9.5) (9.2),
() :
1 ES
U = --- -------- l2. (9.6)
2 l
V = lS (2*),
-
() V:

U = 1--- E2V. (9.7)


2
9.7, .

, F -
BC ,
. -
p = F
---- .
S
CC = l.
:

= l
------ = tg = (9.8)
l
( l n l, tg = , ). -
Z ; -
G ( -
!). (9.8),
:
F
---- = G. (9.9)
S

488
(9.7) -
V
U = 1--- G2V = 1--- G2V. (9.10)
2 2
E G
:
E, G,
2 1011 8 1010
1 1011 4 1010
0,7 1011 2,61010
0,8 1011 3 1010

: -
! .
9.7 , , -
.
(-
), ()
-
() , -
.

p = K V
-------- , (9.11)
V

V
-------- ()
V
p, K -
.

, K =
= 1,42 105 , 100 C = 0,98 105 ,
K = 2,2 109 , K = 1 109 , K =
= 3 109 .

9.1.6.

,
, (9.1), -
v. , -

489
, -
. v, -

v.
9.3 9.6
v k
m . , m -
k,
, , -
.

(, ), , -

v , -
, ,
E ( m -
, k
E; (9.5)). -
:
E
v = ---- . (9.12)

(v
E )
(9.12) , -

v . ,
(9.12)
.

v ,
, , (9.12),
G:
G
v = ---- . (9.13)

, G < E. , -
-
.
( -
) , (9.12)
(9.13), K:
K
v = ---- . (9.14)

490
, ,
! (9.12) (9.14), -
( , ),
-
. , .
v -
* :

g 2
v = ------- + ----------- , (9.15)
2
. -
, (9.15).
,
:

g
v = ------- . (9.16)
2
,
(9.15). -
, :

2
v = ----------- . (9.17)

,
.
h (
), :

v = gh . (9.18)
, -
.

9.2.

9.2.1.

, -
,
.
(
x-, ).

( , );

491
9.5. x- ,
v. -
x,
t,
v .
(t, x).
, x = 0 ,
,
t = 0 . -
:
(t, 0) = msint. (1*)
x -
x = 0 ,
, x, . . , x/v.
, (t, x) t x ,
t:
(t, x) = (t, 0), (2*)

x
t = t ------ . (3*)
v

(1*) (3*), (t, 0) = msin t ------


x
. (2*), -
v

(t, 0) = msin t ------


x
. (9.19)
v

(9.19) .
, .
v (9.13),
(9.12). m -
, .
, ,
- x-, -
-
. 9.8, 9.8, -

A B -
xA xB,
. 9.8,
( x-), 9.8, -
(
x-). -

492
)

. 9.8

; xA xB
.
(9.19) , -
x-. , -
, :

(t, x) = msin t + ------


x
. (9.20)
v

, v = --- =
------- , (9.19) (9.20)
2

(t, x) = msin t ------x .


2
(9.21)

(t, x) = msin t + ------x .


2
(9.22)

,
t x,
k = 2
------ , (9.23)

(
!):
(t, x) = msin(t kx) (9.24)

(t, x) = msin(t + kx). (9.25)


2v
= v = -------------- ,

:

k = ------ . (9.26)
v

493
9.2.2.
?

-
, , v, -
. (9.24),
v(t, x) :
v(t, x) = mcos(t kx). (9.27)
(9.24) (9.27) (8.6)
(8.12), x .

(9.24) (9.27) ,
. (1.8) t :
( t + t, x ) ( t, x )
v(t, x) = ---------------------------------------------------- . (1*)
t
t 0
v(t, x). (9.24), :
(t + t, x) = m sin[(t + t) kx] = m sin[(t kx) + t]. (2*)

sin( + ) = sin cos + cos sin ,
(2*) :
(t + t, x) = m sin(t kx) cost + mcos(t kx)sint.
t , cos t = 1 sint = t.
(1*) (9.27).

(9.27) t, -
( x-) .
--- -
4
(. 9.9,

. 9.9

494
1 , 2
). , -
, -
( = 0, |v| = m), -
(|| = m, v = 0).
-----------------------------------------------------
( t, x + x ) ( t, x )

x
t,
, . . x. -
,
t x:

(t, x) = -----------------------------------------------------
( t, x + x ) ( t, x )
(9.28)
x
( x 0).
(t, x) (9.24), (9.28)

(t, x) = mkcos(t kx). (9.29)


( (9.24) (9.29) t -
(9.24) (9.27) x.)

(9.29), (9.24) (9.28):


m m
(t, x) = ------
- sin[(t kx) kx] ------- sin(t kx). (3*)
x x

sin( ) = sincos cos sin,
(3*) :
m
------
- sin[(t kx) kx] =
x

m m
= ------
- sin(t kx)cos kx ------- cos(t kx) sinkx.
x x
(3*) , x
cos kx = 1 sinkx = kx. (t, x) (9.29).

(9.26), (9.29)

(t, x) = m ------ cos(t kx). (9.30)
v

t -
t.
9.9 3, -

495
--- 2. ,
2
,
, -

( = 0, |v| = m, || = m ------ ),
v

(|| = m, v = 0, = 0).
,
-
.
9.9 -
. , -
x- v.
, , -
(
), -
(
). , , -

, ( -
, ).

.

9.2.3.

S
, (9.24). -
Sx x
x + x, ,
-
. , -
x n .
K(t, x) U(t, x), , -

t. -
,
K(t, x) = 1--- v2(t, x)Sx. (9.31)
2
, , (9.10)
(9.13),
U(t, x) = 1--- G2(t, x)Sx = 1--- v2(t, x)Sx. (9.32)
2 2

496
, , (9.7) (9.12),

U(t, x) = 1--- E2(t, x)Sx = 1--- v2(t, x)Sx. (9.33)


2 2
, U(t, x)
, , v

, v
().
K ( t, x ) U ( t, x )
(t, x) = ---------------------
- u(t, x) = ----------------------- , -
Sx Sx
,
x t.
(9.31) (9.27), -
-
:

(t, x) = 1--- m 2cos2 (t kx).


2
(9.34)
2
(9.32) (9.30), -
-
:

u(t, x) = 1--- m 2cos2 (t kx).


2
(9.35)
2
(9.34) (9.35) , w(t, x)

2
w(t, x) = (t, x) + u(t, x) = m 2cos2 (t kx). (9.36)

cos 2 = 1--- (1 +
2
+ cos2), (9.36) :

w(t, x) = 1--- m 2[1 + cos2(2t 2kx)].


2
(9.37)
2
9.10 1 -
-
; -
. 2

. ,
. ,

,

497
. 9.10

. ,
x- v.
,

(. 8.14); , -
-
-
(. . 8.4.1).
. -
. -
,
. -
-
. -
, ,
.
9.10 , <w>

<w> = 1--- m 2.
2
(9.28)
2
, , -
,
1--- m 2 2
2
------ .
2

9.2.4. ,
( )
, .
,
.

498
. , -
.
,
, -
, -
. -
t t -
W,

= W
---------- . (9.39)
t

, t .
t 0 (9.39) -
t. .
.

j:

j =
-------- , (9.40)
S

-
S.

,

,
- . 9.11

. j <w>, . .
.
S, x- ( ,
(. 9.11). t -
, Svt, -
W = <w>Svt. , -
W
---------- = <w>Sv, -
t
W
-------------- = <w>v. ,
tS
, ,
j = <w>v. (9.41)

499
(), , (9.38),

j = 1--- m 2v.
2
(9.42)
2
, ,
. (9.42) ,
, -
.

v -
.
:
1 2
j = --- m 2 v . (9.43)
2
1874 . -
, -

. j .
(9.39) ,



= .
(-
); 1 = 1 /.
(9.40) .

= .

(-
) (-
: /2). 1 = 1 /, -

.
-
, 1 ,
-
1 2.

9.3.

9.3.1.

,
-

500
. -
m , -
. (
x-) (t, x) =
= msin(t kx), (t, x) =
= msin(t + kx + ).
. (- -
x-).
:
(t, x) = (t, x) + (t, x) =
= msin(t kx) + msin(t + kx + ). (9.44)

sin + sin = 2sin + +


------------- cos -------------
2 2

, cos() = cos, (9.44)

(t, x) = 2mcos kx +
--- sin t + --- . (9.45)
2 2

(9.45) .
9.12 -
(9.45) -

.

x = x + ------ . -
2k

. 9.12

501
: cos kx +
--- = cos kx. (, x)
2
; -

(, x) ------ .
2k
-

:

t1 = -------
2
( 1, ),

t2 = t1 + --- = -------

------- ( 2),
8 4 2

t3 = t2 + --- = -------

------- ( 3),
8 2 2

t4 = t3 + --- = -------
3
------- ( 2),
8 4 2


t5 = t4 + --- = ---
- ------- ( 1),
8 2

t6 = t5 + --- = -------
5
------- ( 4),
8 4 2

t7 = t6 + --- = -------
3
------- ( 5),
8 2 2


t8 = t7 + --- = -------
7
------- ( 4),
8 4 2

t9 = t8 + --- = 2
------ ------- ( 1).
8 2

t1, t5, t9 sin t +


--- (9.45) -
2
.
; . -
t2 t4 sin t +
--- = ------- , t6 t8 sin t + --- = ------- .
2 2
2 2 2 2
-
2 m. t3 sin t +
--- = 1, -
2

502
t7 sin t +
--- = 1.
2
2m.

, , t1 = ------- , -
2
/8 , ,
,
: 1 2 3
2 1 4 5 -
4 1.

: x- ,
!
;
. x- -
3 5
; 9.12 : x = ------ , ------ , ------ .
2k
2k 2k
7 9
, , : ------ , ------ , .
2k 2k
2
, k = ------ x = x ------ = x ------- , -
2k 4
x- :

x = --- (2n 1)
------- (n ). (9.46)
4 4

(, cos kx +
--- = 0.)
2
x- , -
, 2m. -
.
x-: 0, --- , 2 3
------ , ------ , .
k k k
x- :

x = --- 2n
------- (n ). (9.47)
4 4
(,
cos kx +
--- = 1.) ,
2

--- .
2

--- .
4

503
9.3.2.

?
?
, -
(, , , -
), ( , -
, ) . ,
, . -

.

-
. : 1) =
( ,
) 2) , . . = 0 (
, ,
).
1
, , 2
,
.
-
(9.45), = . :
(t, x) = 2msinkx cost. (9.48)
x- (9.46)
= :
x = --- 2n (n ). (9.49)
4
(9.47) = :
x = --- (2n 1) (n ). (9.50)
4

(9.45), = 0:
(t, x) = 2mcoskx sint. (9.51)
, -
, (9.46) (9.47) = 0:

x = --- (2n 1) (n ). (9.52)


4

x = --- 2n (n ). (9.53)
4

504
, -

.

9.3.3.

, -
.
(9.51),
(t, x) t x -
, -
. (t, x), v(t, x)
(t, x) . . 9.2.2
, (t, x) v(t, x) (t, x) -
.
.
(t, x) (9.51), :
( t + t, x ) ( t, x )
v(t, x) = lim ---------------------------------------------------- =
t 0 t
sin ( t + t ) sin t
= 2mcoskx lim ------------------------------------------------------------
- = 2mcoskx cost;
t 0 t

(t, x) = lim -----------------------------------------------------


( t, x + x ) ( t, x )
=
x 0 x
cos ( kx + kx ) cos kx
= 2msint lim -------------------------------------------------------------- = 2mk sint sinkx.
x 0 x
,
v(t, x) = 2mcoskx cost, (9.54)
(t, x) = 2mksinkx sint. (9.55)
:

t1 = 0, t2 = --- , t3 = --- ,
8 4

t4 = 3 5
------ , t5 = --- , t6 = ------ ,
8 2 8

t7 = 3 7
------ , t8 = ------ , t9 = .
4 8
9.13, -
x-

505
. 1,
2, 3, 4, 5, 6, 7, 8, 9.
(9.51). , (, 1, 5, 9) -
. 9.13,
v x- -
; -
, 9.13, 1 9.
(9.54). , -
9.13,
x-
; -
1 9.
(9.55).

. 9.13

-
, ,
t1 = 0 ( 1). t1 = 0

( 2m), -
x- . ,

506
, ,
t2 = --- ( 2).
8
-
, ,
t3 = --- (, 3).
4
x- , -
( 2m) ,
( 2mk)
.
,
! ,
.
9.13.


. ,
, . 
. 9.1.1 , 
, 
.
,
, . ,
, .

9.3.4.

((t, x) u(t, x)
) t x , -
, , (9.31)
(9.33), :

(t, x) = 1--- v2(t, x), (9.56)


2

u(t, x) = 1--- v22(t, x). (9.57)


2
, (9.56)
(9.57) (9.27) (9.29), .
, ,
(9.56) (9.54), (9.57)

507
(9.55). -
:

(t, x) = 1--- m 2cos2 kx cos2 t


2
(9.58)
2

u(t, x) = 1--- m v k2sin2 kx sin2 t,


2 2
2

, (9.26),

u(t, x) = 1--- m 2sin2 kx sin2 t.


2
(9.59)
2
, (9.58) (9.59) -
, .
(9.58) , , coskx = 0,
,
(9.59) , , sinkx = 0, -
u. coskx = 0, kx =
= --- (2n 1), . . x = --- (2n 1) (n ); x
2 4
x- (. (9.52)).
sinkx = 0, kx = n, . . x = --- (2n 1); x -
4
x- (. (9.53)).
,
,
.

v.

-
? ?

9.14 9.15.
9.14 u -
:
t1 = 0 ( ), t2 = --- ( ), t3 = --- ( ),
8 4
9.15 w
-
(w = + u). -
,
.

508
)

. 9.14

-
:

(t, x) = 1--- m 2cos2 t (1 + cos2kx),


2
(9.60)
4

u(t, x) = 1--- m 2sin2 t (1 cos2kx),


2
(9.61)
4
, , (9.58) (9.59), -
cos2 = 1--- (1 + cos2x)
2

sin2 = 1--- (1 cos2x).


2

. 9.15

509
t1 = 0 cos2 t1 = 1 sin2 t1 = 0. -
(9.60) (9.61):
1 2
= --- m 2 (1 + cos2kx);
4
u = 0; (9.62)
1 2
w = + 0 = --- m 2 (1 + cos2kx).
4
, t1 = 0 u
x- ;

, -
.
t3 = --- cos2 t3 = cos2 --- = 0 sin2 t3 = sin2 --- = 1.
4 2 2
(9.60) (9.61):
= 0;
1 2
u = --- m 2 (1 cos2kx);
4 (9.63)
1 2
w = 0 + u = --- m 2 (1 cos2kx).
4

, t3 = --- x-
4
;

,
.
t2 = --- cos2 t2 = cos2 --- = 1--- sin2 t2 =
8 4 2

= sin2 1
--- = --- . (9.60) (9.61):
4 2

1 2
= --- m 2 (1 + cos2kx);
8
1 2
u = --- m 2 (1 cos2kx); (9.64)
8
1 2
w = + u = --- m 2.
4

, t3 = --- (. . -
8
t1 t3)

510
, -

.
, ,

,
, -
, -
.
, , -
. -
, -
, ,
-
.
, ,
, -
. (, ,
)
,
, -
-
, .
( ) --- .
4
, -

, -
.

9.3.5.

, -
.
, .
l. ,
,
. , ,
. , -
, . -
, , -
l -
.

511
( ).
.
1. .
. -
--- , , , -
2


--- n = l, n = 1, 2, 3, . (9.65)
2
,
2v
= v = -------------- , (9.66)

(9.62) :
v
= ---------- n, n = 1, 2, 3, ... . (9.67)
l
, , l -
, ,
(9.67).
2. . -
, -
. -
--- , -
2

, , --- + --- , --- + 2 --- , --- + 3 --- ,


4 2 4 2 4 2

--- + 4 --- , , , , --- (1 + 2), --- (1 + 4), --- (1 + 6), ... .
4 2 4 4 4
, l

--- (1 + 2m) = l, m = 0, 1, 2, 3, ...


4


--- (2n 1) = l, n = 1, 2, 3, , (9.68)
4
, (9.66),
v
= ---------- (2n 1), n = 1, 2, 3, . (9.69)
2l
, , ,
, . ,

512
) )

) )

) )

. 9.16 . 9.17

, ( ),
.
9.16, ,
. -
, -
( ), ,
( ), , ( ).
9.17,
.
, ,
.

9.4.

9.4.1.

1
2.
1(t, r )

r t 1 -
2,
2(t, r ) -
2 1.
,
(t, r ) -

1 + 2:

(t, r ) = 1(t, r ) + 2(t, r ). (9.70)

513
, , ,
, .
-
.
,
() , -
.
,
,
, .
, ,
.


XV -
. , -
, -

, ,
, , -
, , ,
,
...

, 1 2, -
, -
: 1) ; 2)
( r ) -
; 3)
1 2 -
, , -
(9.70) . ,
, (
cohaesus, , -
).

, -

. , , -
,
( ) -
-

514
. -
,
.
,
,

.
. , -
.
1m 2m -
, (
r ) r ,
m(
r ) r

m(
2 2
r) = 1m + 2m + 2 1m 2m cos ( r) . (9.71)

,
cos(
r ) = 1, -
, , (9.71),
max = 1m + 2m. (9.72)

, cos(
r ) = 1, -

,
min = |1m 2m|. (9.73)

( 1m = 2m min = 0;
.)
-
. -
, -
. , -
.
,
, -
. , -
, -
. -
,
. ,

515
-
-
. , -

, -
.
, -
, -
, , -
, . -
v
--- = ---------- ;
4 2
.
-
, 2m,
.
. -
, -
-
, 180. ,
, , -
,
.

9.4.2. :

-
O1 O2, d . -
, ,
( -
). -
P (. 9.18) ,
-
-
, O1
O2.
-
, ,
. 9.18 P O1 O2,

516
l . -
l = l1 l2
. , l -

= 2l
------------- . (9.74)

(9.71), , P -
( -
), (9.74)
: cos = 1, . .
= n, n . (1*)
(1*) (9.74),
:
l = n (n ). (9.75)
,
, , -
, .
-
P ( ); -
.
: cos = 1, . .

= --- (2n 1) (n ), (2*)


2
P .
(2*) (9.74), :

l = --- (2n 1) (n ). (9.76)


2
,
, , -
, .
P
, . .
; -
.
, (9.75) (9.76) -
. O1 -
0 O2,
= 2l/
(9.71)
= 0 + 2 l
------------- . (9.77)

517
9.4.3. :

-
? ,
-
?
, - , -
.
,
, -
O1O2 . -
S0. - , -
-
,
, -
O1 O2. ,
O1 O2 -
, . . -
. ,
, -

.

, -
O1 O2 .
(
), ( -
). -
,

, , -
-
. , -
-
,
O1 O2.
9.19
. , -

. 9.19 MN (

518
S0 ). 1 2 -
. ,
O1O2, l -
O1 O2 -
P:

cos = l
------ , (9.78)
d

O1 O2.
O1O2 s ,

s = 1--- ( l). (9.79)


2

(9.79) , ,
, = (s + l) + s.
(9.78). O1PO2,
O1P = l + l O2P = l O1P O1O2
(. 9.19). O1PO2

l2 = (l + l)2 + d2 2(l + l)d cos . (1*)

. 2(l + l)d cos = 2ll + (l)2 + d2 , ,

2 2
2ll + ( l ) + d
cos = -------------------------------------------- . (2*)
2 ( l + l )d

, 9.19, cos = lim cos .


l

lim cos , 2l -
l
(2*):

2 2
( l ) + d
l + --------------------------
2l
cos = -------------------------------------- . (3*)
1 + l ------ d
2l

, (3*) l
l l
------ . , cos = lim cos = ------ ,
d l d
.

519
, -
, (9.78) (9.79)
l = , 2, 3... -

l < d. (4*)
, -
, (9.78) (9.79) l = --- , 3 --- ,
2 2

5 --- ... (4*).


2
-
, , -

d = 5--- . (5*)
2

(4*), , -
-
( l = l = 2;
1 2, ) -
( l = --- l = 3 --- 3 4, -
2 2
). (5*) (9.78) (9.79),
1, 2, 3, 4:


s1 = 1--- (d ) = -----
3 2
- d, cos1 = --- = --- , 1 = 66 25;
2 10 d 5

s2 = 1--- (d 2) = -----
1 2 4
- d, cos2 = ------ = --- , 2 = 36 50;
2 10 d 5

s3 = 1--- d --- = 2--- d, 1


- = --- , 3 = 78 30;
cos3 = ------
2 2 5 2d 5

s4 = 1--- d 3
------ = --- d,
1 3 3
- = --- , 4 = 53 10.
cos4 = ------
2 2 5 2d 5

-
(. 9.20).
MN 1 2. -
3 4 -

520
. 9.20

, O1 O2. -
-
,
O1 O2, , -
.

, -
! , -
, -
,
-
.

,
9.20
, O1 O2. -
-
, -
.

S0, , -
.
, -
, 9.21, . ,

521
)

. 9.21

O1O2, MN,
, -
(. 9.21, ).

9.4.4.

9.3 -
* ,
. -
,
(, ). -
;
1787 .

- (, ) -

( -
, ) ,
(. 9.22). -
-
, -
, , -
.
. 9.22 . -
(
), , ,
, ,
.
9.23, , .
, -

522
) )

. 9.23

, A
( ).
, B B. -
, , .
, -
.

9.5.

9.5.1.

-
(, ). -
, -
. ,
, ,
= 1--- 16 20 000 .

() -
. 16
, 20 -
. 109
.
-
.
, , , .
-
, :

= 1--- = 2
------

523
(
) (. (9.42))
j = 1--- m 2v.
2
2

,
(/2);
.

(); , -

-
. , -
;
( ). ,
( -
), .
-
, . .
, .
. -
, -
, -
. -
,
.

9.24 -
* ( 1)
( 2). -

. 9.24

524

, . ,
( 1012 /2), -
( 1 / 2), -
1000 . j-, -
1 2, . -
,
.
: , A ( A--
), , B-, ,
, ,
B- , -
. A- C-,
A- C-, -
. , ,
-
. ( )
.

.
;
. -
() -
, 2, 3, ... 1m, 2m, 2m, ... . -
, -
.
. ( ) -
.
, -
, -
.
, -
, .
)
9.25, ,
-
, (
) ( )
, 0,01 . - )
, -

-
-
,
. . 9.25

525

.
(),
().

9.5.2.

-
. , -
. -
,
. , -
, -
.
, -
.
-
. -

, . . .
, -
.
16 , . -
. -
, ,
.
() . -
, , , -
, .
.
. -
, (, ,
) (, , ).
, -
, .
,
, , . -
-
. -
,
,

. ,

526
, -
.
, -
.
. -
. -
. ,

. 10

.

0 331 / ( (9.14),
,
1,42 105 ). -
, 0,6 /
. , , 400
520 /.
, . ,
20 1480 / ( -
2,2 109 ).
100 1540 /.
. , -
(9.12)
3340 / , 5090 / , 5000 / .
1,5 ,
.
,
, .
-
, . -
, .

9.5.3.

, -
, ,
( -
).
, -
, .

527
, , , -
, -
,
(. 9.26).
. -

-
-
-
,
.
. 9.26 , , ,
, . -
-
.
( -
),
. ,
, ,
.
,
, -
. --
,
-
.

9.5.4.

, , -
.
:
,
, -
. , -
j , x-,
x -
:
j(x) = j(0)e2x. (9.80)
j(0) x = 0 ( , -
), -
, e = 2,71828 .

528
,
1
. x0 ------
- ,
2

(9.80) j(x0) = j(0) 1--- . ,


e
1
x0 = ------
- (9.81)
2
, -
e d 2,7 .
, , , . .
, -

( ). -
, . . -
-
.
:
2
2
= --- ----------
3
, (9.82)
3v

, .
()
, -
.
.
, 20 -
60 , . -
,
. (
), -
, .
2
(9.82).
, . , -
(,
), -
, -
-
.
(9.82), , -
* , 20 . -
, 5 v,

529
830 , 60 . -
, 125 830/60 d 1700 -
, . . , -
100 , -
170 ?
, . ,

, . . . , -
, ( ), -
, . . ,

-
( 4r2, r ).
, , , . . -
.
, = 0, . . , .
(r0) r0
(r) -
r. , (9.40), (r0) =
2
= 4 r 0 j(r0) (r) = 4r2j(r), (r) = (r0) ,

r0 2
j(r) = j(r0) ----- . (9.83)
r

,
1
j(r) f ---- . , , -
2
r
,
:
1
j(r) f ---- e2r. (9.84)
2
r
, -
(9.84). -
, e2r ,
1
---- ,
2
r
.

9.5.5.

( j 20 ) -
. -
, . -

530
.
. 9.5.6.
,
-
( -
),
. -
, -
. -
, -
.
:
1) ,
.

(105106) /2; 2)
, , -
v
. = ------ ,

,
= (105107) = (0,330,0033)
= (1,50,015) . -
-
.
-
. (
105 /2) -
,
, -
. -
-
, , -
, .
( (102103) /2) -
( -
, , -
, ), ( -
,
), (
,
, ).
( k 16 )
.

531
,
; -
. -
. -
( 100 )
, -
, -
.
, -
.

9.5.6.


* ? , -
. .
1794 . ,
, ,
, - ;
. ,
, . -

. : -
. , -
- .
- .
,
, .
, , -
.
, ,
XX . 1938 . -
. . , (-
), ,
, ,
. , -
, -
. -
.
,
, , ( -
sound, ). -
.
, , -
.

532
= (4080) .
v
= ------ = (48) .

, -
. T ,
L . = 40 , ,
= 8 , n = 40 , :
n
T = --- = 103 , L = n d 30 . -

.
- ,
. -
, . ,
.
, , . -
, , -
.
, -
. XX . -
80 100 . -
; -
.
-,
. .
, -
. .
,
( -
). -
. , -
, , . , -
, ; ,
.

(102104 ).
, -
.
, -
, .
.
, .
- , ,
, . -
(
). -
- .

533
9.6.

9.6.1.


, , ,
,
. , -
, -
. 1842 -
:

-
,
( , -
), , -
.

.
, ,
,
.
,
.
-
, , ,
,
.
, . -
,
. = 1--- = ----
1
- , , ,

, ,
, -
.
, -
,
1 2. , 1 -
t = 0 , , 2 -
t = . t1 t2 -
1 2, ; , t2 = t1 + . -
l1 l2

534
t1 t2. , l1 -
, 1 -
, . . t1. , l2 -
, 2
, . . t2 .
v; ,
l1 = t1v, l2 = (t2 )v. (9.85)

.

9.6.2. ,

, -
, ()
v. l2l1 , . .
1 2; ,
l2 l1 = v. (1*)
(9.85) (1*),
(t2 )v t1v = v. (2*)
, t2 = t1 + , (2*) ( )v =
= v. ,
v
1 --------

- = .
v

1 1
= ----
- = --- ,

v
= 1 --------

- . (9.86)
v

,
, v, (1*)

l1 l2 = v, (3*)
(9.85) -
( )v = v. ,
v
= 1 + --------

- . (9.87)
v

535
, , -
, < ,
, -
> .

9.6.3. ,

, -
, v.
, (9.86) (9.87)
v v.

-
? ,
,
, . , -
,
; (9.85)
, -
.
, () -
, v. -
l2 l1 , . . -
1 2; -
,
l2 l1 = v. (1*)
(9.85) (1*),
(t2 )v t1v = v. (2*)
t2 = t1 + , (2*) ( )v = v.
,
v
= 1 + ---------

- .
v


1
= ------------------------------
-. (9.88)
1 + v v


v, (1*) :
l1 l2 = v. (3*)

536
:
1
= ------------------------------
-. (9.89)
1 v v

1. -
v = 120 /. -
-
,

, ?
v = 330 /.
v
(9.89)
----- = 1 + --------- ,
v

v
1
- = -------------------- .
1 + --------
v v
1 ---------
-
v

-
v v
v = ---------------------
- = 88 /.
v + v
-
v +v
---------------------
- = 1,36 . ,
v
-
120 /
88 /.

2. = 6200
, -
v = 5 /.
, ?
v = 330 /.

. , -
. -
(9.86) -

= 1 ------
v
= 6106 .
v

537
--
,
, v.
, (9.88):
1
= ---------------
- = 6015 .
v
1 + ------
v

,

(6200 6015) = 185 .

9.6.4. ,

, -
v, -
,
v. l2 l1
v, 1
2, v, -
1 -
2; ,
l2 l1 = v + v. (1*)
(9.85) (1*), :
(t2 )v t1v = v + v. (2*)
t2 = t1 + , (2*)
( )v = v + v.
,
v v
1 --------

- = 1 + ----------

v v

, ,
1v v
= ------------------------------

-. (9.90)
1 + v v


, , v v, (1*) -
l2 l1 l1 l2.

538
:
1+v v
= ------------------------------

-. (9.91)
1 v v

1. : ) -
v
, ------ (v
2
); ) ,
v
------ ; )
2
v
------ .
4

, . -
-
,
.
, , ,
(9.86), (9.88), (9.90) :
1 2 3
----- = --- ( ), ----- = --- ( ), ----- = --- ( ).
2 3 5
= -
: = 1--- = 1--- : = 2--- =
2 2 3

= 1--- .
3

2.
v = 60 /.
, -
N = 1,5 -
, ?
v = 330 /.

(9.91),
----- = N. -

, -

v = v(N 1) Nv = 25 /.

539
3. = 10 000 .

. -
, -
? = 330 /.

, -
, . .
= 20 000 . v -
(9.91),
----- = 2,

1 + v v
----------------------- = 2.
1 v v

, v = 3--- v = 110 / d 400 /.


5

9.6.5. ,

, ,

1+V v
= -------------------------------

-, (9.92)
1 + V v

V V v v
x-, -
, v v ( v)
(. (9.86)(9.91)).
, 9.27,
( , ).
, :
V = v, V = 0 ;
V = v, V = 0 ;
V = 0, V = v ;
V = 0, V = v ;
V = v, V = v ;
V = v, V = v .

540
)

. 9.27

(9.92),
, , , , , (9.86); (9.87); (9.88); (9.89); (9.90) (9.91).
9.28 (
). -
x-,
x-. : V = v V = v ;
(9.92),
1+v v
= ------------------------------

-. (9.93)
1 + v v
: V = v V = v ; (9.92),
1v v
= ------------------------------

-. (9.94)
1 v v
, v = v,
( = ).
9.28, ,

, v v

. 9.28

541
x-. -
,
, v, v . ,
V(t) = vcos(t) V(t) =
= vcos(t). (9.92),
:
1 + v cos ( t ) v
(t) = ---------------------------------------------------

-. (9.95)
1 v cos ( t ) v

.
, ,
. ,
, -
, .
cos = cos = 0 -
. , ,

.

9.7.

9.7.1.


1690 , -
.
. , ,
, ,
. -
, .
t, -
t + t, , -
, -
. -
t + t
, -
vt.

542
9.29, 9.29, - ) )
-
t, -
-
t + t,
.
, -
-
(. 9.29, ), -
(. 9.29, ).
. 9.29

, , , -
. , -
, .
: , -
, , ,
.

9.7.2.

-
(. 9.30). -
AC ,
MN, A1A B1B
, (
).
MN .

. 9.30

543
.
AC -
. -
B , A,

t = CB
--------- ; (1*)
v

, , -
A, -

AD = vt. (2*)
, ,
DB;
, -
AA2 BB2 -
. -
.
,
AD = ABsin, BC = ABsin. (3*)
(1*) (2*) , AD = BC. ,
(3*):
= . (9.96)
, .
, -
,
. -
.

9.7.3.

( 1 2), -
. v1 v2
1 2, .
, , v1 > v2. -
1 -
, .
( -
), -
2, . -

544
. 9.31

,
(. 9.31).
AC -
; . -
MN. B
, A, -

CB
t = --------
-. (1*)
v 1

,
, A 2

AD = v2t. (2*)
, -
2, , ,
, ,
( ).
DB, AA2 BB2
.

.
(1*) (2*) ,
CB v 1
--------- = --------- . (3*)
AD v 2

545
, CB = AB sin AD = AB sin; -

CB sin
--------- = ------------- . (4*)
AD sin
(3*) (4*), :
sin v 1
------------- = --------- . (9.97)
sin v 2

v1 > v2 , > . (9.97) -


,

.
(9.97) 9.31 ,
:


, ;

-
.


, ,
, 
.
130 , 
, 
. 1823
( 
).
.

9.7.4.

, v -
v : v > v.
.
,
, . . , v > v.
(v < v),

546
, .

. , ,
. .
.
, . -
, v = 4v. -
,
t (. 9.32). A, B,
C, D. , AB = BC = CD = vt = 4vt. 9.32,
B. -
,
A, r = vt. -
9.32, C.
, A, 2r,
B r. 9.32,
D. A
3r, B 2r,
C r.

. 9.32

. ,
A D. -
,
. -
;

547

, , -
, .

, -
9.32, . -
, .
, . -
,
v
sin = ------ . (9.98)
v

(. 9.33). -
-
v.

T ,
. -
? v.


. 9.33 A . ,
B.
, H = vTtg.
tg sin (9.98);
sin 1
- = vT ----------------------------- .
H = vT ----------------------------
1 sin2 v 2
1 ------
v
,
. -
,
.

9.8.

9.8.1.

seis-
mos, -
. ,

548
,
. -
.
-
, , , -
. ,
.
,
-
. -
; -
, ,
. -
, -
, . -
, -
,
.
.
-
. -
.

9.8.2.

,
, , . -
, -
, -
;
-
, , ,
,
,
. ,
,
.

-
? ? -
?

549
,
-
. , () -
, -
( ). ,
, ( ) -
, -
-
. ,
; -
. .
( -
) , 5 /, -
. ,
, , -
, , -
.
9.34 : 1 ,
* 2 , 3 , 4 ,
5 , 6 -, 7 -
, 8 , 9 , 10 , 11 , 12
, 13 , 14 , 15 , 16 -
, 17 , 18 .

. 9.34

550
, XX ;
. -
.
(, 5 4 1 8) ,
,
. (-
, 8 5 6 3) ,
.

9.34 -
, . . ,
. -

: ( -
) ( -
-
).
, -
-
,
. -
, -
.
-
.

.
,
.
-
.
- ( ) -
, -
,
( ).
. -
. -
,
,
(. 9.35, ).
, -
(. 9.35, ).

551
)

. 9.35

; -
(. 9.35, ).
, -
-
,
. -
, , . -
(, -
, ), .

,
,
-
. -
,
.
, (-
) .
,
.
, ,
. , , -
, -
. 700 . -
() 700 300 -
, 300 70 ,
70 .

552
9.8.3. ,

-
.
. -
, -, , -, -
,
, , -
.
.
,
P- (. 9.36, ), -
, S- ( 9.36, ). (
) E = 81010 ,

. 9.36

553
G = 3 1010 , = 2600 /3.
E
P- ---- = 5,5 /, -

G
S- ---- = 3,4 /. -

, P- S- -
.

, -
. ,
, . -
.
P- S- 9.38, (. . 9.8.6).

,
! 2900 P- 1,61,8 -
S-. ,
P-
S-. -
. P- -
, ,
. , -
, S-,
-
, . , -
S-, P-.
t -
P- S-, H -
. v1 P-, v2
S-,
v v
- t.
1 2
H = ----------------- (9.99)
v1 v2


,
. , (-
, )
;
-
.
(
L-) (.
9.36, ) (. 9.36, ).

554
, .
9.36, , -
-
, . 9.36, -
, , , -
;
-
. -
, ,
.

9.8.4. :

, - ,
:
7,5 . ,
. ,
,
. , -
-
, . -
, (, -
1935
) , -

( ; - magnitudo
). ( ) -
, -
; , -
12- -
( MSK-64).
,
: 7,5
: 7
MSK-64. .
.
. -
-
, -
. -
,
,

555
.
, -
.
12-
MSK-64 (. 1).
1

I C .

II , .

III .

IV .

V , -
, .

VI ,
, .

VII , -
, ,
.

VIII , , ,
,
.

IX ,
,
.

- X ,
, , -
, .

XI
( ) -
,
.

XII , -
, -
, ,
.

,
. -
(, VIII ) -

556
-
. - -
, .
, ,
VII . -
,
VI . .
-
.
( ).

, (
). W
M :

M = 2--- lgW 3. (9.100)


3

lg , W -
. ,
, -, , -
,
, , -,
P- S- ( -
-
). (9.100),
.
(9.100) , , , M = 3,0 -
lgW = 9 , , W = 109 . (9.100),
:
< M < . M = 0,
9
--- 9
W = 10 2 = 10 d
d 3 104 .
500 6 . -
.
M < 0. -
M = 9,5.
W d 1019 . ,
1019 , -
, 9,5.
, ,
1019 ,
.

557
, 0 m M m 9,5.
M1 = 2--- lgW1 3 M2 = 2--- lgW2 3. ,
3 3
W
M2 M1 = 2--- lg --------2 . (1*)
3 W1

W
, M2 M1 = 1. (1*) lg --------2 = 3---
W1 2
W 3
, , -------2- = 10 d 30. ,
W1
30 -
.
,
8,0, -
7,0 7,9, 6,0 6,9,
5,0 5,9.

9.8.5. ,

,
,
9.37.
-
: A -
( ),
B , -
-
, C
.
-
,

A, -
, C,
. 9.37 -
,
B , -
.
, W -
M = 5,0. (9.100),

558
W = 1012 . , , -
, ,
1946 . ,
5,0 .
-
-
. 9.37, -
:
IX 3 ( A);
VII 10 ( B);
II 200 ( C).
, 200 ,
, -
, .
3 , .

1. : 3,1,
3 ; : 6,2,
100 .
.
9.37 ,
VI ( 4), -
V ( 5).
-
.

2. , L = 100
, t = 24
P- S-.

, , -
M = 7,4. P- v1 = 5,5 /,
S- v2 = 3,4 /.
.
H . t,
2 2
, . . H + L .
(9.99) H;
2 2 v v t
, H +L 1 2
= ------------------
- = 214 . L = 100 , ,
v1 v2
, H = 190 . H -
M, 9.37 -
; VI ( 6).

559
9.8.6.

6370 -
. 6370 5100 -
12 000 /3.
5100 2900 .
, 2900 300 -
, 300 100
, ( -
). 50
, , -
( -
).


.
, , ,
-
, -
, -
.
-
: , -
-
, , -
.
9.38 -
P- S- -

. 9.38

560
.
,
.
,
.
,
2900 13 / P-
7 / S-.
, S-,
, , P-
8 /.
P- -
5100 , -
.
:
!
, , -
( !) , ,
. -
.

9.1. , - -
?
, - 9.4.
, - 9.3, -
3 -
. , -
9.2. , - .
- 9.5.
- 9.3, -
- , , -
. 4 103 /3.
9.3. 9.6. -
- , -

: x-, ,
(t, x) = 5 106 sin(1800t 6x). x = 0
- t = 0
, - m. -

561
, k.
, ,
k.
9.7. -
x- , -
, : ) ------ ; ) ------ ;
- 8k 4k
, -
) 3
------ ?
8k
9.6? . -
9.8. - 9.10.
- 9.12. -
, -
- ,
.
? v. -
9.9. , - -
?
E, - 9.13.
-
m. -
2m.
. :
.
) ; ) -
? -
9.10. -
, - , :
m, - ) --- ; ) --- ; ) -----

-?
, 6 8 12
k. . -
9.12.
|v|
|| 9.14. -

9.13,
: ) ------ - -
4k
- -
,
; ) ------ , -
6k
: ) -----
- ; ) --- ; ) --- ; ) --- .
- 12 8 6 4
? . -
. - 9.13, .
9.9. 9.15. t = 0 -
9.11.
- -
, - 0.
m, - . -

562
9.19.
- -
: L = 850 . -
3
) ------- ; ) ------- ; ) ------- ; ) ------- ?
8 4 8 2 .
. - -
9.14. ,
9.16. - -
, -
, - t = 2,4 .
2m
. t = 0 v = 330 /. -
- , -
-
1--- m 2.
2 .
2 9.20. -
3 =
: ) ------- ; ) ------- ;
2 4 = 26 .

) ---
-? -

. - , -
9.15. ? -
9.17. - ,
, - , 0 = 16 .
v =
- = 330 /.
, - 9.21. -
, ,
- -
, - , -
. -
d = 7
------
2 -
? - ,
- ?
. 9.22. t = 0 -
9.18. -

a ,
.
,
, ;
, - v .
. -

563
, - 9.24. -
? v = 990 /
9.23. - , -
= 20 .
A B
,
C (. T = 4
9.39); AD = DB = DC. , -
. -
,
-
?
v = 330 /.
9.25. -
. 9.39 ,
-
,
: . -
) A; ) D; , -
) B? .

564

.1.

, , , -

, :
(sin ) ,
, ;
(cos )
;
(tg)
;
(ctg )
.
.1, :

sin = a b a b
--- ; cos = --- ; tg = --- ; ctg = --- .
c c b a
(.1)
(.1) + = a2
- b2 c2
:
. A.1
sin2 + cos2 = 1; (.2)
sin cos
tg = ------------
- ; ctg = ------------- ;
cos sin
tg ctg = 1. (.2)
.1 (.1),
:


sin = cos --- , cos = sin --- ,
2 2
(.3)

tg = ctg --- , ctg = tg --- .
2 2

565
, , . . -
, , ;
90 --- .
2
-
. ( -
, -
.)

0 30 45 60 90


0 --- --- --- ---
6 4 3 2

1 2 3
sin 0 --- ------- ------- 1
2 2 2

3 2 1
cos 1 ------- ------- --- 0
2 2 2

tg
3
0 ------- 1 3
3

ctg
3
3 1 ------- 0
3

.2,

( ):
sin ( + ) = sin cos + cos sin; (.4)
cos( + ) = cos cos sin sin. (.5)

. A -
AE AB. B -
. A.2 , -, AE ( BD)
, -, ( BC).
,
AE = Rsin( + ) = AD + DE;
AD = ABcos = (Rsin )cos;
DE = BC = OBsin = (Rcos)sin .
AD DE AE, (.4).

566
.2 ,
OE = Rcos ( + ) = OC EC;
OC = OBcos = (Rcos )cos ;
EC = DB = ABsin = (Rsin )sin .
OC EC OE, (.5).

.3,

( ):
sin ( ) = sin cos cos sin ;
(.6)
cos ( ) = cos cos sin sin .
(.7)

. A - . A.3
AE AB. C
CD, E -
EG. E , OC, -
AB; F .
,
AB = Rsin ( ) = AF BF;
AF = AEcos = (Rsin)cos.

BF, OCD OEG. , -


GE CD
--------- = --------- . GE = BF, OE = Rcos , CD = Rsin, OC = R,
OE OC
BF R sin
: ------------------- = ------------------ .
R cos R
, BF = Rcos sin . AF BF AB, -
(.6).
.3 ,
OB = Rcos( ) = OG + GB;
OG = OEcos = (Rcos)cos;
GB = EF = AEsin = (Rsin)sin.
OG GB OB, (.7).

(.4), (.5) , ,
sin 2 = 2sin cos ; (.8)
cos 2 = cos2 sin2 . (.9)

567
, sin2 + cos2 = 1, (.9):

cos2 = 1--- (1 + cos 2); sin2 = 1--- (1 cos 2). (.10)


2 2
(.8), (.9) sin 2 + cos 2,
:
2tg
sin 2 = ----------------------
2
; (.11)
1 + tg
2
1 tg
cos 2 = ----------------------
2
. (.12)
1 + tg
(.4), (.5), (.2),
:
sin ( + ) tg + tg
tg ( + ) = ---------------------------
- = ---------------------------- , (.13)
cos ( + ) 1 tg tg

(.6), (.7), (.2)


:
sin ( ) tg tg
tg ( ) = ---------------------------
- = ------------------------------- . (.14)
cos ( ) 1 tgtg
(.13) ,
2tg
tg 2 = ----------------------
2
. (.15)
1 tg
, (.11)
(.12).
: sin + sin . ,
:

= + +
------------- + ------------- ; = ------------- ------------- .
2 2 2 2

, (.4) (.6):

sin = sin +
------------- + ------------- =
2 2

+ +
= sin ------------
- cos ------------- + cos ------------- sin ------------- ; (1*)
2 2 2 2

sin = sin +
------------- ------------- =
2 2

= sin + +
------------- cos ------------- cos ------------- sin ------------- . (2*)
2 2 2 2

568
(1*) (2*),
:

sin + sin = 2sin +


------------- cos ------------- . (.16)
2 2
(2*) (1*), :

sin sin = 2cos +


------------- sin ------------- . (.17)
2 2

cos = sin ( --- ) cos = sin ( --- ), ,


2 2

(.16), cos + cos = 2sin ---------------------------


( + )
cos
------------- . (.3), -
2 2

cos + cos = 2cos +


------------- cos ------------- . (.18)
2 2
,

cos cos = 2sin +


------------- sin ------------- . (.19)
2 2

.2.

(.1) , -
sin , cos , tg , ctg ,
--- .
2
, :
0 m m --- y = sin y = cos ,
2

0 m < --- y = tg ,
2

0 < m --- y = ctg .


2
-
--- .4.
2
-
-
--- . +.
2

569
. A.5

. A.4

.5, 0 +,
O OA ,
. -
OA 0
. OA O A -
, B
N M M N. y = sin , y = cos , y =
= tg , y = ctg - (.1)
, a = AB, b = OB, c = OA. AB > 0,
A MN, AB < 0 -
; OB > 0, B y-,
OB < 0 .
,
-
+. , -
,
. .1.
-
, -
. -
, x, -
. y = sin , y = cos ,
y = tg , y = ctg .6. --- , , 2
2
. . , -
.

570
. A.6

y = sin x y = cos x, -
.7.
x-; 2;
1 m y m 1, . .
.

. A.7

, y = cos x ,
y = sin x x- --- ,
2

cos x = sin x + --- .


2

571
, y = sin x , -
y = cos x x- --- ,
2

sin x = cos x --- .


2



sin + --- = cos . .1,
2
. (.4),

= --- , , sin --- = 1, cos --- = 0, :
2 2 2

sin + --- = sin cos --- + cos sin --- = cos .
2 2 2

(.7) = --- , :
2

cos --- = cos cos --- + sin sin --- = sin .
2 2 2

y = sin x ,
:
sin (x) = sin x. (.20)
,
sinx = 0, x = m;

sinx = 1, x = --- + 2m;
2 (.21)

sinx = 1, x = --- + 2m.
2

m .
y = cos x ,
:
cos (x) = cos x. (.22)
,

cosx = 0, x = --- + m;
2
cosx = 1, x = 2m; (.23)
cosx = 1, x = (2m + 1).

m .

572

 , 
(.6) (.4), (.7) (.5):
sin ( ) = sin [ + ()] = sin cos () + cos sin () =
= sin cos + cos sin .
cos ( ) = cos [ + ()] = cos cos () + sin sin () =
= cos cos + sin sin .

.1.

, ,
, . -
,
, -

, . a -

, a | a | . a b

| a = b | , -
.
, -
, , .
, -
. ,
.

, a , -

. .1

x-, y-, z-.

ax, ay, az .
,

ax = asin cos ;
ay = asin sin ;
ax = acos ; (.1)
2 2 2
a= a x + ay + az . . .1

573
,
-
.

a , a , -

a .
: ax, ay, az.

a , , -

, a , a.

, a a ,
a.

.2. , ,

)
-

(. .2, ). a

b , c ,
) . -

.
.2, -
) :

a b -

A a
-

b . ,
. .2
A b , -
(. .2, ):

c = a + b.

A b

a ;
(. .2, ). , -
( -
):

a + b = b + a. (.2)

d a , b , c ,

a b c

574
(. .3). ,

, a -

b + c ; ,
-
( ):

( a + b) +
c = a + ( b + c ). (.3)
, .3 -
. -

c d - . .3

a b ;
.4.
-
-
:
-
, -
,
-
.
,
, -
, -
.
, . .4
-
,
. , , -

.
, ,

a b , a

b . .5, . -
.5,

) )

. .5

575
: -

b a , a

b A

c (

b )

( a ). c -
. .6
a b .
.6 ,

a b . a + b ,

a b .
,
:

(1 2) a = 1 a 2 a ; (.4)

( a b ) = 1 a 2 b . (.5)
-
, ,

.

, a ,
, , -
, . -

a .7;
MN PQ

, b c ,

a . -

, b
c

a
, MN PQ.
( -
(, )
.)
. .7 -

, a :

a = ax i + ay j + az k . (.6)

i , j , k
( ), , ,
x-, y- z-.

576


a + b +
c = d (1*)
. ,

a , b ,
c . , -
, . -
xy-.
(1*) :

ax + b x +
c x = d x;
(2*)
ay + b y +
c y = d y.
, ,

xy-. , , a

b , -
( x-)
; (2*) :
ax + bx + cx = dx;
cy = dy. (3*)

a , b , c , -
, (1*) -
:
ax + bx + cx = dx ;
ay + by + cy = dy ; (4*)
az + bz + cz = dz .

.3.
,
, -
, -
, .

a b
,
:

a b = ab cos ( a , b ). (.7)
-

, a b ( a , b ).

a b , a b = ab; ,

a a = a2. a b , a b = ab.

a b , a b = 0.

577
-
:

a b = b a; (.8)

a ( b c ) = a b a c ; (.9)

a b = ( a b ); (.10)
m n m n

a b = a b .

i j i j (.11)

i=1 j=1 i = 1j = 1

a b -

a b . (.6) (.11), :

a b = (ax
i + ay j + az k ) (bx i + by j + bz k ) =

= axbx(
i i ) + axby( i j ) + axbz( i k ) + aybx( j i ) +

+ aybz( j k ) + azbx( j k ) + azbx( k
i ) + azby( k j ) + azbz( k k ). (1*)
,

i i = j j = k k = 1;


i j = j i = j k = k j = i k = k i = 0. (.12)

(1*)
i j , j i , j k , k j , i k , k i
. (1*):

a b = axbx + ayby + azbz. (.13)
-
, , -



.
, -

a , b ,
c , -

c = a + b (. .8).
. .8 ,
c2 = a2 + b2 2abcos . (.14)

c = a + b -
;

c2 = ( a + b ) ( a + b ) = a a + 2 a b + b b =

= a2 + b2 + 2 a b .

578

(.7), a b = abcos ( a , b ). ,

( a , b ) a b ;
c2 = a2 + b2 + 2abcos ( ).
cos ( ) = cos cos + sin sin = cos , , -
, c2 = a2 + b2 2ab cos , .
: -
.
.6,

a b .

a + b , a b . -
:

( a + b ) ( a + b ) = a2 + 2 a b + b2;

( a b ) ( a b ) = a2 2 a b + b2.
,

( a + b )2 + ( a b )2 = 2(a2 + b2). (.15)
(.15) -

a + b a b , . . -
,
. .
, -
:

( a + b ) ( a b ) = a a b b = a2 b2. (.16)
(.16) , -
, a = b, . . -
.

.4.

a b
c , , -

a b ,

. .9, .

a b
.

b -
. :

c = a b.

579
) , -

a b (. -
.9, ).
a
, ,

, bsin ( a , b ),

)
( a , b ) a b .
,

a b

c = absin ( a , b ). (.17)

a b

. .9 , a b = 0;

, a a = 0 ( -

). a b
, c = ab.
, ,
,
.
-
:

b a = ( a b ). (.18)
-
:

( a b ) = a b = a b ; (.19)

a b = ( a b ); (.20)

a ( b + d) = a b + a d; (.21)
m n m n

a b .

a i b j = i j (.22)

i=1 j=1 i = 1j = 1


( a b )x, ( a b )y, ( a b )z,

a b , x-, y-, z-, .

a b . , -
-
. z-
, x- y-
. .10, -

580
) )

. .10

, .10,
. z-, -
x- y- -
.


.
, , 

, , . 
,

a b , , 

, a b , 
, 

a b
, x y z.

.10, ( )
i,

j , k . ,

i j = k; j k = i ; k i = j;
(.23)
j i = k; k j =
i; i k = j .
:


i i = 0; j j = 0; k k = 0. (.24)
(.6), :


a b = (ax i + ay j + az k ) (bx i + by j + bz k ) =

= axbx(
i i ) + axby( i j ) + axbz( i k ) +

+ aybx( j
i ) + ayby( j j ) + aybz( j k ) + azbx( k i ) +

+ azby( k j ) + azbz( k k ). (1*)

581
(.23) (.24), -
(1*) :

a b = (aybz azby) j + (azbx axbz) j + (axby aybx) k .
(.25)
,

( a b ) x = a y b z a zb y ;

( a b )y = azbx axbz; (.26)

( a b )z = axby aybx.

, -
, -
- -
, . -

, a , b , c ,


c = a + b ( -
.8). ,
b sin
--- = ------------ . (.27)
c sin

a
c = a + b ;

c a = a a + b a.

a a = 0,
c a = b a . -

c a b a :

ca sin (
c , a ) = ba sin ( b , a ). (2*)
(2*) :
c sin = b sin ( ). (3*)
, sin ( ) = sin cos cos sin = sin ,
(3*) (5.27), .

a b (. .6).

a + b a b :

( a + b ) ( a b ) = a a + b a a b b b = 2 a b .

( a + b ) ( a b ) 2 a b ,
, , , , -
,
, -
.

582

1.13.
1.1. --- .
2 v 0,
5 2v
1.2. ------ . ---------0 .
3 t1
1.3. .
1.14. x0 2v0t1.
1.4.
v0 - 1.15. x0 + 2v0t1.
1.16. , -
, .
1.5. . -
- .
, 1.18. .
2
v 0 + 2gh + v 0
2v0. 1.19. ---------------------------------------
-.
g
2
2v
1.6. --------0- . 1.20.
2
v 0 + gH .
R
1.7. x1 = v0t1 = 15 . gH
1.21. --------- .
1.8. x1 1,2 - 2
, . 2
v
1.9. 10 /2. 1.22. ------0- .
2g
1.10. - 1.23. , . .
(a1 = 3 /2), 2
v 0 sin
(a2 = 3 /2). ------------------- . -
2g
1.11. - -
- v0cos .
, - 1.24. -
. v0cos .
2 2 1.25.
( n 1 )v
1.12. -------------------------0- = 9 /2. 6 ,
2x .

2
2.3. . 2.5. (, -
2.4. - ).
m1 + m2 m m2 2.6. -
: --------------------- -------1 + ------- .
V1 + V2 V1 V2 tg .

583
2.7. . m1 ( g + a )
mg 2.18. m2 = -------------------------- = 4,8 ;
2.8. ) ------------- ; ) mgcos . ga
cos
P = m1(g + a) = 27,6 .
2.9. a = g(sin cos ); N =
= mgcos . 2.19. 10,3 -
2.12. , 9,8 -
, -
- , 8,8 -
. . ,
2.13. C -
, .

2.20. .

. gtg
2.22. -------------------------- .
m 2 m 1 ( sin + cos ) R + l sin
2.14. a = g ---------------------------------------------------------------
-;
m1 + m2 mv 0 cos
2.24. ------------------------- .
m1 m2 g M
T = --------------------- (1 + sin + cos ).
m 1 + m2 2.25. v
2.15. , . 1 2 2
2.16. ) Mg; ) - --- v 1 + v 2 , v
2
Mgm
-------------------- ; ) - v 1
2M + m
. v
tg = -----2- .
P v1
2.17. m1 = ------------- = 1,3 ; m2 =
g+a
mv 0 2H
P 2.26. ----------------- -------- .
= ------------- = 1,9 . M+m g
ga

3.1. , . 1 1
3.6. ) --- aF1 = 0,12 ; ) --- aF2 =
3.2. , 2 2
-
1
. = 0,24 ; ) aF1 --- aF2 = 0.
2
3.3. ) 0; ) ; )
cos . mga mga
3.7. ) ------------- ; ) 0; ) ------------- .
1 2 2
3.4. ) --- aF2 = 0,24 ;
2
mga mga
1 3.8. ) ------------- ; ) 0; ) 0; ) ------------- .
) a --- F 2 + F 1 = 0,48 ; ) 0.
2 2 2
2
1
3.5. ) 0; ) 0; ) --- aF2 = 0,24 ;
3.9. ---
2 6
1 -
) --- aF2 = 0,12 .
4 .

584
l 1
3.11. < ------- . 3.19. --- a(N 1).
2h 2
3.12. 2 . 3.20. 40 .
1 2 a2 l
3.13. TA = --- mg; TB = Mg + --- mg. 3.21. --------------------
- d 1
3 3 2 2
r a
3.14. m1 m2 . , -
.
1
3.15. --- mg ctg . 41r
2 3.22. xC = --------- = 10,25 ; yC =
44
1
3.16. l --- . 13r
3 = --------- = 13 .
3.17. , - 11
, 3.23. xC = 7,5 ; yC = 3,75 .
3.24. -
- .
. 3.25. 8Mg = 2350 .
r 1
3.18. min = ------------------ = 0,25. 3.26. ----------------- .
R sin 2 sin

4.1. A: 2na = 5,02 /; 7


4.14. --- mr2.
B = 2 2 na = 7,1 /; - 5
2
4.15. --- mr2 (-
2 na = 3,55 /.
5
4.2. ) ; ) . ) mR ( 2
4.3. 4,29 ; 246. -
4.4. T = 200 /; T2/(4) = ).
= 79,6 .
m 2 m1
2
4.5. 50 /2. 4.17. g --------------------- .
4.6. . m 1 + m2
v
4.7. ---- ; 0. ( m2 m 1 )
2
R 4.18. g --------------------------------------------------------------------- .
1
R + R
4.8. 2(R + R); ----------------- . ( m 1 + m 2 ) m 1 + m 2 + --- m
2
R
4.9. , . 4.19. B:
4.10. ) 2ma2; ) ma2; ) 4ma2; ) ml
2

3ma2. -------------- ,
2
1
4.11. --- ma2. ml
2
6 z-. z-: -------------- .
2
2
4.12. --- ma2. 4.20. B:
3
2
ml
4.13. m ---r + ---l .
4 2 12 -------------- , 135
5 2 2 2

585
2 M + 2m
ml
z-. C: -------------- , 4.28. -------------------- .
4 M
z-. 4.29. 5,4104 2.
2 1
mr 4.30. --- gsin = 2,45 /2;
4.21. --------------- = 10 . 2
2T
> --- tg = 0,29 .
1
h ( 2r h ) 2
4.22. mg ----------------------------- .
rh 2m 2 g
2nJ nT 4.31. ) ---------------------------------------
-;
4.23. -------------- = 94,2 ; ------- = 11 750 - 3m 1 + 2m 2 + m
2
3m 1 m 2 g
. ) ---------------------------------------
-,
4.24. ) 2ma2; ) ma2. 3m 1 + 2m 2 + m
3m 1 + m
4.25. --- MR + --- ma .
1 2 1 2
m2g ---------------------------------------
-;
2 3 3m 1 + 2m 2 + m

( M + 2m )R
2 m2
4.26. ---------------------------------- . ) > ---------------------------------------
-.
2 2 3m 1 + 2m 2 + m
MR + 2mr
4.32. . -
2mvr
4.27. ---------------------------------- . ,
2 2
MR + 2mr .

5.1. ) mgs sin ; ) 0; 5.13. 25 .


) mgs cos . 1
5.2. - 5.14. --- mgl( 3 1);
2
. .
5.3. ) ; ) ; ) ; ) ; 5.16. h(1 ctg )/ = 8 .
) . 5.17. Fscos = 10,4 .
5.5.
2
v 0 2gh . mgs
5.18. ------------------------ = 8,1 .
1 + tg
5.6. 3 .
5.7. mgh. 5.19. v0 = gh .
1 5.20. 1 : 3.
5.8. --- mv2 mgh = 80,1 .
2 2mg ( h + s )
5.21. ------------------------------- = 6 .
5.9. s
2
-
5.22. -

sin (1 + cos ) = 0,27; -
gh
5.10. ( 5 2) ------- . 1
----------------------- = 0,54.
2 1 + cos
2
mv N
5.11. ----------- = 6,4102 . 5.23. ---------------------------------------------------- =
5 mg ( sin + cos )
5.12. ) 6; ) 3. = 18,9 / = 68 /.

586
T mg v 02
5.24. l ------------------- d 1 . 5.29. -------------------------------
- d 20 .
2mg 2
2g ( n + 1 )
5.25. 6mg. +m
5.30. 5gl M
----------------- .
m
5.26. 5mg2t2sin2 ------ .
14 1
5.31. mg(h h) --- mv2 = 2,33 .
2
v
5.27. -----0- ,
5.32. mgl cos --- cos .
2 2
mv 2
5.33. mgl cos --- cos + --- tg
-----------0- . 1
4 2 2

sin --- tg --- sin --- = 0,97mgl.
5mgl 1
5.28. --------------- .
4 2 2 2

6.1. (1 + )( 1 ) = 1,53 . 6.15. g = 0,4g0 = 3,9 /2; =


6.2. 1,87 . = 0,74 = 4,1103 /3; V =
6.3. 3,541022 . = 0,46V1 = 3,63 /.
6.4. 2500 .
6.5. 0,6 . 6.16. 2,55 .
6.6.
GM
100 - 6.17. ) ---------------- = 42 /;
. R
6.7. ( n 1)R d 2GM
d 1430 . ) ------------------- = 59 /.
R
6.8. 4 .
2 3 6
4 r
6.9. --------------- = 6,1 1024 . 6.18. ----------- .
2
GT
2 GT

4 300
6.10. --- GR. 6.19. --------------- = 0,34%.
3 2
GT
r
6.11. ---------------
- = 38 400 . 300
6.20. --------------- = 3,8103 /3.
1+ 2
nGT
3 r 3
6.12. = -----------2 --------- = 1,24103 /3; R
GT R 6.21. 2 -------------- = 5050 = 1,4 .
GM
3
r
g = 42 ---------------
- = 25 /2. 6.22. 6 .
2 2
T R
6.23. R R d 3890 .
24
6.13. ----------- = 5,51 103 /3.
2
GT M B 1
6.24. --------B- = 4; ------ = --- .
6.14. 2V. MA A 2

587
2GMr R 2r 1
6.25. v1 = ----------------------------- = 40,1 /; v2 = V1 ------- 1 ---------------- = 357 /
r ( r + r ) r2 r1 + r2
3 (V1 = 7,9 / -
( r + r )
T = --- ------------------------- d 6,3 . ).
2 2MG
6.31. ) 1,68 /; ) 2,38 /.
2g 0 R GM
6.26. v = ------------------ = 6,45 /. 1
6.32. --- ( 2 1) --------------- = 350 /.
n R
2
1
6.27. --- mg0R(5 + ( 2 1)2) = GM
6.33. --- ----------
6 1 1
--------------- = 310 /.
2 10 R
= 10,76109 .
2 6.34. 10 /.
g0 R
6.28. ( 1 + 1) -------------- = GM
r1 6.35. -------------- = 5,9103 /2.
2
r
= 1,94 /.
6.29. 6640 . 6.36. -
-
R 2r 2
6.30. v1 = V1 ------- ---------------- 1 = 2,2 .
r1 r1 + r2
4
= 375 /; 6.37. ------ Gr = 5,28 /2.
3

7.1. ,
. 7.11. -----2 .
1
(: -
). 7.12. -
7.2. ah. -
7.3. 2l. . -
-
FS 20Vg.
7.4. ----------0- = 0,04 .
S 7.13.
7.5. 1075 . , -
2gH -
7.6. ----------------------0- . .
+ 0
0 n
m 7.14. ------------ = 1500 /3.
7.7. ------- = 2,5 . n1
S
V 0
h 1 1 h2 2 7.15. --------------
- = 1980 3.
7.8. ------------------------------
- = 2,1 . 0

2
p 7.16. --- V -----0 = 0,21 3.
7.9. ------0- = 8,5 . 3
g

7.10. , - 7.17. -----1 = 0,58.
. 2

588
n 1130 /3. -
7.18. ------------ = 3. 5,2 .
n1
PP 3
7.28. ------- d 1,44 .
7.19. -----------------1 = 700 /3. - 3 9
P
.
2
Vg 7.29. 4R2 ------- 1 = 1,38105 .
7.20. --------------------0 = 2,21 . 4
3
0
3 7.30. 2D2 = 3,6103 .
R
7.21. 1 + (2 1) ------- . 2
3
r 7.31. -------------- = 0,0015 .
0 gh
7.22. 2 .
3 2
PR
7.23. ---------------------------------------------- . 7.32. ) -------------- = 0,74 ;
3 3 h 1 g
( P P1) ( R r )
7.24. . 2 2
) -------------- = 0,37 ; ) ---------- = 6 .
7.25. - - h 2 g rg
.
3
2 7.33. = ------------------------------ = 3340 /3.
7.26. --- S0gh2 1 ----- = 141 .
1 2GR 0 rh
2 0
2
7.27. 700 /3. 2v 5 0 v
7.34. ------- ; p + ---------------
-.
- 3 18

8.1. 3x m
. 8.6. ) xm = 0,4 /; ) ------------------- =
2
8.2. -
, - = 0,35 /.
, - 8.7. ,
4 . -
8.3. , - ,
. ,
. :
) , ) - .
, ) , - 8.8. x(t) = 0,3sin t x(t) =

3 = 0,3cos t --- .
------- . 2
2 8.9. v(t) = 0,3cos t.
3 8.10. :
8.4. --- .
4 9 1042 cos2 t; :
8.5. x(t) x(t) = 91042; :
= xmsin (1 + 2)t; xm, 9 1042 sin2 t.
1 + 2, - 1 vm 1 2
8.11. ) --- ------- ; ) --- m v m ;
. 2 2

589
mv m 2
m
1 2
( )
) --- m v m sin2 t + --- ; ) ------------
2 6 8
-. 8.18. 4 ----- .
k
8.12. - m
t, - 8.19. 2 ------ .
3k

( )
sin t + --- = 0, . . -
6
R
8.20. ----------
- = 1740 .
n

n ---
8.21. l0 1 2 ------- = 3 94 .
6 h
t = ---------------- , - R

t, Th
8.22. ------- = 27 .

( )
sin t + --- = 1, . .
6
R
l
8.23. 2 ---------------------- .
n + --- 2 2
3 g +a
t = ---------------- ( n
h
). 8.24. 2 ---------- = 0,6 .
0 g
8.13. ) 0,24 /; ) 0,36 /2,
) 1,73103 . l
8.25. 2 ------ .
8.14. ) 0,05 ; ) 0; ) 0,34 /2. g
2mg 8.26. )
8.15. ------------- . -
k
, -
m
8.16. 2 ------------------ .
k1 + k2 ; ) x(t) = 2 xmsin t + --- , ( )
4
8.17. 2m. .

9.1. , 2 E E
3 , 9.9. ) 22 m ------------ ; ) ---- .

2
9 .
9.2. -
. 9.10. ) |v| = 2 m
---- ,
2
9.3. 5104 ,
300 /, || = 2 m k--- ; ) |v| = 1--- m,
3,5 103 . 2 2
9.4. 52,5 . 1
|| = --- mk.
9.5. 48,6 /2. 2
9.6. (t, x) = mcos (t kx).
9.7. v(t, x) = msin (t kx).
1
2
2
(
2
9.11. ) --- m 2 1 + ------- ;
2
)
2 2

9.8. ---------------- .
4
2
1
2
2 1
2
2 2
(
) --- m 2; ) --- m 2 1 ------- .
2
)
590
v v L
9.12. ---------- . 9.19. -----------------------
- = 4840 / d 5 /.
L v t

9.13. ) 3 m; ) 2 m; ) m.
9.20. v --------------0- = 206 /.
0
2 3m 2 2m
9.14. ) ---------------------- ; ) ---------------------- ;
9.21. v --------------0- = 127 /.

2 m
) -------------
- ; ) 0.
9.22. (t) = 1 ------ .
at
v

9.15. a) 0 cos2 --- ;
8 9.23. ) --------------------- ; ) ; ) --------------------- .
2v 2v
3 1 ----------- 1 + -----------
) -----0- ; ) 0 cos2 ------ ; ) 0. 2v 2v
2 8
v T
1 2 1 2
9.16. ) 0; ) --- m 2; ) --- m 2. 9.24. ------------------------------------------ = 1490 d
4 2 v 2
cos 1 ------

9.17. 3. v
d 3d 5d d 1,5 .
9.18. ------ , ------- , ------- .
14 14 14 9.25. 1400 .

591

. .
. . , . .
. .
. .
$ 
27.08.2008. 6090 1/16.
. .
. . . 37,00. 3000 . .

$005$93, 2; 953005

.
105082, , . , . 7, . 1.
: 117418, , / 26.
: . (499) 619$02$20, 619$31$88.
$: www.onyx.ru

.
. . 05088 18.06.2001.
109193, , . 5$ , . 13, . 1.
./ (495) 120$51$47, 129$09$60, 742$43$54.
E$mail: mir$obrazovanie@onyx.ru

You might also like